User login
A 66-year-old man with abnormal thyroid function tests
A 66-year-old man presented to the emergency department with increasing shortness of breath and productive cough, which had begun 5 days earlier. Three years previously, he had been diagnosed with chronic obstructive pulmonary disease (COPD).
One week before the current presentation, he developed a sore throat, rhinorrhea, and nasal congestion, and the shortness of breath had started 2 days after that. Although he could speak in sentences, he was breathless even at rest. His dyspnea was associated with noisy breathing and cough productive of yellowish sputum; there was no hemoptysis. He reported fever, but he had no chills, night sweats, chest pain, or paroxysmal nocturnal dyspnea. The review of other systems was unremarkable.
His COPD was known to be mild, in Global Initiative for Chronic Obstructive Lung Disease (GOLD) grade 1, group A. His postbronchodilator ratio of forced expiratory volume in 1 second (FEV1) to forced vital capacity (FVC) was less than 0.70, and his FEV1 was 84% of predicted. Apart from mild intermittent cough with white sputum, his COPD had been under good control with inhaled ipratropium 4 times daily and inhaled albuterol as needed. He said he did not have shortness of breath except when hurrying on level ground or walking up a slight hill (Modified Medical Research Council dyspnea scale grade 1; COPD Assessment Test score < 10). In the last 3 years, he had 2 exacerbations of COPD, 1 year apart, both requiring oral prednisone and antibiotic therapy.
Other relevant history included hypertension and dyslipidemia of 15-year duration, for which he was taking candesartan 16 mg twice daily and atorvastatin 20 mg daily. He was compliant with his medications.
Though he usually received an influenza vaccine every year, he did not get it the previous year. Also, 3 years previously, he received the 23-valent pneumococcal polysaccharide vaccine (PPSV23), and the year before that he received the pneumococcal conjugate vaccine (PCV13). In addition, he was immunized against herpes zoster and tetanus.
The patient had smoked 1 pack per day for the past 38 years. His primary care physician had advised him many times to quit smoking. He had enrolled in a smoking cessation program 2 years previously, in which he received varenicline in addition to behavioral counseling in the form of motivational interviewing and a telephone quit-line. Nevertheless, he continued to smoke.
He was a retired engineer. He did not drink alcohol or use illicit drugs.
PHYSICAL EXAMINATION
On physical examination, the patient was sitting up in bed, leaning forward. He was alert and oriented but was breathing rapidly and looked sick. He had no cyanosis, clubbing, pallor, or jaundice. His blood pressure was 145/90 mm Hg, heart rate 110 beats per minute and regular, respiratory rate 29 breaths per minute, and oral temperature 38.1°C (100.6°F). His oxygen saturation was 88% while breathing room air. His body mass index was 27.1 kg/m2.
His throat was mildly congested. His neck veins were flat, and there were no carotid bruits. His thyroid examination was normal, without goiter, nodules, or tenderness.
Intercostal retractions were noted around the anterolateral costal margins. He had no chest wall deformities. Chest expansion was reduced bilaterally. There was hyperresonance bilaterally. Expiratory wheezes were heard over both lungs, without crackles.
His heart had no murmurs or added sounds. There was no lower-limb edema or swelling. The rest of his physical examination was unremarkable.
Results of initial laboratory testing are shown in Table 1.
Assessment: A 66-year-old man with GOLD grade 1, group A COPD, presenting with a severe exacerbation, most likely due to viral bronchitis.
INITIAL MANAGEMENT
The patient was given oxygen 28% by Venturi mask, and his oxygen saturation went up to 90%. He was started on nebulized albuterol 2.5 mg with ipratropium bromide 500 µg every 4 hours, prednisone 40 mg orally daily for 5 days, and ceftriaxone 1 g intravenously every 24 hours. The first dose of each medication was given in the emergency department.
The patient was then admitted to a progressive care unit, where he was placed on noninvasive positive pressure ventilation, continuous cardiac monitoring, and pulse oximetry. He was started on enoxaparin 40 mg subcutaneously daily to prevent venous thromboembolism, and the oral medications he had been taking at home were continued. Because he was receiving a glucocorticoid, his blood glucose was monitored in the fasting state, 2 hours after each meal, and as needed.
Two hours after he started noninvasive positive pressure ventilation, his arterial blood gases were remeasured and showed the following results:
- pH 7.35
- Partial pressure of carbon dioxide (Paco2) 52 mm Hg
- Bicarbonate 28 mmol/L
- Partial pressure of oxygen (Pao2) 60 mm Hg
- Oxygen saturation 90%.
HOSPITAL COURSE
On hospital day 3, his dyspnea had slightly improved. His respiratory rate was 26 to 28 breaths per minute. His oxygen saturation remained between 90% and 92%.
At 10:21 pm, his cardiac monitor showed an episode of focal atrial tachycardia at a rate of 129 beats per minute that lasted for 3 minutes and 21 seconds, terminating spontaneously. He denied any change in his clinical condition during the episode, with no chest pain, palpitation, or change in dyspnea. There was no change in his vital signs. He had another similar asymptomatic episode lasting 4 minutes and 9 seconds at 6:30 am of hospital day 4.
Because of these episodes, the attending physician ordered thyroid function tests.
THYROID FUNCTION TESTING
1. Which thyroid function test is most likely to be helpful in the assessment of this patient’s thyroid status?
- Serum thyroid-stimulating hormone (TSH) alone
- Serum TSH and total thyroxine (T4)
- Serum TSH and total triiodothyronine (T3)
- Serum TSH and free T4
- Serum TSH and free T3
There are several tests to assess thyroid function: the serum TSH, total T4, free T4, total T3, and free T3 concentrations.1
In normal physiology, TSH from the pituitary stimulates the thyroid gland to produce and secrete T4 and T3, which in turn inhibit TSH secretion through negative feedback. A negative log-linear relation exists between serum free T4 and TSH levels.2 Thus, the serum free T4 level can remain within the normal reference range even if the TSH level is high or low.
TSH assays can have different detection limits. A third-generation TSH assay with a detection limit of 0.01 mU/L is recommended for use in clinical practice.3
TSH testing alone. Given its superior sensitivity and specificity, serum TSH measurement is considered the best single test for assessing thyroid function in most cases.4 Nevertheless, measurement of the serum TSH level alone could be misleading in several situations, eg, hypothalamic or pituitary disorders, recent treatment of thyrotoxicosis, impaired sensitivity to thyroid hormone, and acute nonthyroidal illness.4
Free vs total T4 and T3 levels
Serum total T4 includes a fraction that is bound, mainly to thyroxin-binding globulin, and a very small unbound (free) fraction. The same applies to T3. Only free thyroid hormones represent the “active” fraction available for interaction with their protein receptors in the nucleus.8 Patients with conditions that can affect the thyroid-binding protein concentrations usually have altered serum total T4 and T3 levels, whereas their free hormone concentrations remain normal. Accordingly, measurement of free hormone levels, especially free T4, is usually recommended.
Although equilibrium dialysis is the method most likely to provide an accurate serum free T4 measurement, it is not commonly used because of its limited availability and high cost. Thus, most commercial laboratories use “direct” free T4 measurement or, to a lesser degree, the free T4 index.9 However, none of the currently available free T4 tests actually measure free T4 directly; rather, they estimate it.10
Commercial laboratories can provide a direct free T3 estimate, but it may be less reliable than total T3. If serum T3 measurement is indicated, serum total T3 is usually measured. However, total T3 measurement is rarely indicated for patients with hypothyroidism because it usually remains within the normal reference range.11 Nevertheless, serum total T3 measurement could be useful in patients with T3 toxicosis and in those who are acutely ill.
Accordingly, in acutely ill hospitalized patients like ours, measuring serum TSH using a third-generation assay and free T4 is essential to assess thyroid function. Many clinicians also measure serum total T3.
CASE CONTINUED: LOW TSH, LOW-NORMAL FREE T4, LOW TOTAL T3
The attending physician ordered serum TSH, free T4, and total T3 measurements, which yielded the following:
- TSH 0.1 mU/L (0.5–5.0)
- Total T3 55 ng/dL (80–180)
- Free T4 0.9 ng/dL (0.9–2.4).
2. Which best explains this patient’s abnormal thyroid test results?
- His acute illness
- Central hypothyroidism due to pituitary infarction
- His albuterol therapy
- Subclinical thyrotoxicosis
- Hashimoto thyroiditis
Since euthyroid patients with an acute illness may have abnormal thyroid test results (Table 2),5–7 thyroid function testing is not recommended unless there is a strong indication for it, such as new-onset atrial fibrillation, atrial flutter, or focal atrial tachycardia.1 In such patients, it is important to know whether the test abnormalities represent true thyroid disorder or are the result of a nonthyroidal illness.
In healthy people, T4 is converted to T3 (the principal active hormone) by type 1 deiodinase (D1) mainly in the liver and kidneys, whereas this reaction is catalyzed by type 2 deiodinase (D2) in the hypothalamus and pituitary. Type 3 deiodinase (D3) converts T4 to reverse T3, a biologically inactive molecule.12 D1 also mediates conversion of reverse T3 to diiodothyronine (T2) (Figure 1).
Thyroid function testing in patients with nonthyroidal illness usually shows low serum total T3, normal or low serum TSH, and normal, low, or high serum free T4. However, transient mild serum TSH elevation can be seen in some patients during the recovery period.16 These abnormalities with their mechanisms are shown in Table 2.5–7 In several commercial kits, serum direct free T4 can be falsely decreased or increased.8
THE DIFFERENTIAL DIAGNOSIS
Our patient had low serum TSH, low-normal serum direct free T4, and low serum total T3. This profile could be caused by a nonthyroidal illness, “true” central hypothyroidism, or his glucocorticoid treatment. The reason we use the term “true” in this setting is that some experts suggest that the thyroid function test abnormalities in patients with acute nonthyroidal illness represent a transient central hypothyroidism.17 The clinical presentation is key in differentiating true central hypothyroidism from nonthyroidal illness.
In addition, measuring serum cortisol may help to differentiate between the 2 states, as it would be elevated in patients with nonthyroidal illness as part of a stress response but low in patients with true central hypothyroidism, since it is usually part of combined pituitary hormone deficiency.18 Of note, some critically ill patients have low serum cortisol because of transient central adrenal insufficiency.19,20
The serum concentration of reverse T3 has been suggested as a way to differentiate between hypothyroidism (low) and nonthyroidal illness (high); however, further studies showed that it does not reliably differentiate between the conditions.21
GLUCOCORTICOIDS AND THYROID FUNCTION TESTS
By inhibiting D1, glucocorticoids can decrease peripheral conversion of T4 to T3 and thus decrease serum total T3. This effect depends on the type and dose of the glucocorticoid and the duration of therapy.
In one study,22 there was a significant reduction in serum total T3 concentration 24 hours after a single oral dose of dexamethasone 12 mg in normal participants. This effect lasted 48 hours, after which serum total T3 returned to its pretreatment level.
In another study,23 a daily oral dose of betamethasone 1.5 mg for 5 days did not significantly reduce the serum total T3 in healthy volunteers, but a daily dose of 3 mg did. This effect was more pronounced at a daily dose of 4.5 mg, whereas a dose of 6.0 mg had no further effect.
Long-term glucocorticoid therapy also decreases serum total T4 and total T3 by lowering serum thyroid-binding globulin.24
Finally, glucocorticoids can decrease TSH secretion by directly inhibiting thyrotropin-releasing hormone.25,26 However, chronic hypercortisolism, whether endogenous or exogenous, does not cause clinically central hypothyroidism, possibly because of the negative feedback mechanism of low thyroid hormones on the pituitary and the hypothalamus.27
Other drugs including dopamine, dopamine agonists, dobutamine, and somatostatin analogues can suppress serum TSH. As with glucocorticoids, these drugs do not cause clinically evident central hypothyroidism.28 Bexarotene, a retinoid X receptor ligand used in the treatment of cutaneous T-cell lymphoma, has been reported to cause clinically evident central hypothyroidism by suppressing TSH and increasing T4 clearance.29
BETA-BLOCKERS, BETA-AGONISTS AND THYROID FUNCTION
While there is general agreement that beta-adrenergic antagonists (beta-blockers) do not affect the serum TSH concentration, conflicting data have been reported concerning their effect on other thyroid function tests. This may be due to several factors, including dose, duration of therapy, the patient’s thyroid status, and differences in laboratory methodology.30
In studies of propranolol, serum total T4 concentrations did not change or were increased with daily doses of 160 mg or more in both euthyroid participants and hyperthyroid patients31–33; serum total T3 concentrations did not change or were decreased with 40 mg or more daily34; and serum reverse T3 concentrations were increased with daily doses of 80 mg or more.31 It is most likely that propranolol exerts these changes by inhibiting D1 activity in peripheral tissues.
Furthermore, a significant decrease in serum total T3 concentrations was observed in hyperthyroid patients treated with atenolol 100 mg daily, metoprolol 100 mg daily, and alprenolol 100 mg daily, but not with sotalol 80 mg daily or nadolol (up to 240 mg daily).35,36
On the other hand, beta-adrenergic agonists have not been reported to cause significant changes in thyroid function tests.37
SUBCLINICAL THYROTOXICOSIS OR HASHIMOTO THYROIDITIS?
Our patient’s thyroid function test results are more likely due to his nonthyroidal illness and glucocorticoid therapy, as there is no clinical evidence to point to a hypothalamic-pituitary disorder accounting for true central hypothyroidism.
The other options mentioned in question 2 are unlikely to explain our patient’s thyroid function test results.
Subclinical thyrotoxicosis is characterized by suppressed serum TSH, but both serum free T4 and total T3 remain within the normal reference ranges. In addition, the serum TSH level may help to differentiate between thyrotoxicosis and nonthyroidal illness. In the former, serum TSH is usually suppressed (< 0.01 mU/L), whereas in the latter it is usually low but detectable (0.05– 0.3 mU/L).38,39
Hashimoto thyroiditis is a chronic autoimmune thyroid disease characterized by diffuse lymphocytic infiltration of the thyroid gland. Almost all patients with Hashimoto thyroiditis have elevated levels of antibodies to thyroid peroxidase or thyroglobulin.40 Clinically, patients with Hashimoto thyroiditis can either be hypothyroid or have normal thyroid function, which is not the case in our patient.
CASE CONTINUED
An endocrinologist, consulted for a second opinion, agreed that the patient’s thyroid function test results were most likely due to his nonthyroidal illness and glucocorticoid therapy.
3. In view of the endocrinologist’s opinion, which should be the next step in the management of the patient’s thyroid condition?
- Start levothyroxine (T4) therapy
- Start liothyronine (T3) therapy
- Start N-acetylcysteine therapy
- Start thyrotropin-releasing hormone therapy
- Remeasure thyroid hormones after full recovery from his acute illness
It is not clear whether the changes in thyroid hormone levels during an acute illness are a pathologic alteration for which thyroid hormone therapy may be beneficial, or a physiologic adaptation for which such therapy would not be indicated.41
However, current data argue against thyroid hormone therapy using T4 or T3 for patients with nonthyroidal illness syndrome (also called euthyroid sick syndrome).42 Indeed, several randomized controlled trials showed that thyroid hormone therapy is not beneficial in such patients and may be detrimental.41,43
Therapies other than thyroid hormone have been investigated to ameliorate thyroid hormone abnormalities in patients with nonthyroidal illness. These include N-acetylcysteine, thyrotropin-releasing hormone therapy, and nutritional support.
Some studies showed that giving N-acetylcysteine, an antioxidant, increased serum T3 and decreased serum reverse T3 concentrations in patients with acute myocardial infarction.44 Nevertheless, the mortality rate and length of hospitalization were not affected. Further studies are needed to know whether N-acetylcysteine therapy is beneficial for such patients.
Similarly, a study using a thyrotropin-releasing hormone analogue along with growth hormone-releasing peptide 2 showed an increase in serum TSH, T4, and T3 levels in critically ill patients.45 The benefit of this therapy has yet to be determined. On the other hand, early nutritional support was reported to prevent thyroid hormonal changes in patients postoperatively.46
Measuring thyroid hormone levels after full recovery is the most appropriate next step in our patient, as the changes in thyroid hormone concentrations subside as the acute illness resolves.47
CASE CONTINUED
The patient continued to improve. On hospital day 6, he was feeling better but still had mild respiratory distress. There had been no further episodes of arrhythmia since day 4. His blood pressure was 136/86 mm Hg, heart rate 88 beats per minute and regular, respiratory rate 18 breaths per minute, and oral temperature 37.1°C. His oxygen saturation was 92% on room air.
Before discharge, he was encouraged to quit smoking. He was offered behavioral counseling and medication therapy, but he only said that he would think about it. He was discharged on oral cefixime for 4 more days and was instructed to switch to a long-acting bronchodilator along with his other home medications and to return in 1 week to have his thyroid hormones checked.
One week later, his laboratory results were:
- TSH 11.2 mU/L (reference range 0.5–5.0)
- Free T4 1.2 ng/dL (0.9–2.4)
- Total T3 92 ng/dL (80–180).
Clinically, the patient was euthyroid, and examination of his thyroid was unremarkable.
4. Based on these last test results, which statement is correct?
- Levothyroxine therapy should be started
- His serum TSH elevation is most likely transient
- Thyroid ultrasonography is strongly indicated
- A radioactive iodine uptake study should be performed
- Measurement of thyroid-stimulating immunoglobulins is indicated
During recovery from nonthyroidal illness, some patients may have elevated serum TSH levels that are usually transient and modest (< 20 mU/L).48 Normalization of the thyroid function tests including serum TSH may take several weeks49 or months.50 However, a systematic review found that the likelihood of permanent primary hypothyroidism is high in patients with serum TSH levels higher than 20 mU/L during the recovery phase of their nonthyroidal illness.51
Ultrasonography is useful for evaluating patients with thyroid nodules or goiter but is of little benefit for patients like ours, in whom the thyroid is normal on examination.
Similarly, a radioactive iodine uptake study is not indicated, as it is principally used to help differentiate between types of thyrotoxicosis. (Radioactive iodine is also used to treat differentiated thyroid cancer.)
Thyroid-stimulating immunoglobins are TSH receptor-stimulating antibodies that cause Graves disease. Nevertheless, measuring them is not routinely indicated for its diagnosis. However, their measurement is of significant help in the diagnosis of Graves disease if a radioactive iodine uptake study cannot be performed (as in pregnancy) and in atypical presentations such as euthyroid Graves ophthalmopathy.52 Other indications for thyroid-stimulating immunoglobin measurement are beyond the scope of the article. Our patient’s test results are not consistent with hyperthyroidism, so measuring thyroid-stimulating immunoglobins is not indicated.
CASE CONCLUSION: BETTER, BUT STILL SMOKING
The patient missed his 1-month clinic follow-up, but he visited the clinic for follow-up 3 months later. He was feeling well with no complaints. Test results including serum TSH, free T4, and total T3 were within normal ranges. His COPD was under control, with an FEV1 88% of predicted.
He was again encouraged to quit smoking and was offered drug therapy and behavioral counseling, but he declined. In addition, he was instructed to adhere to his annual influenza vaccination.
KEY POINTS
- In patients with acute illness, it is recommended that thyroid function not be assessed unless there is a strong indication.
- If thyroid function assessment is indicated for critically ill patients, serum TSH and free T4 concentrations should be measured. Some clinicians also measure serum total T3 level.
- Thyroid function testing in critically ill patients usually shows low serum total T3, normal or low serum TSH, and normal or low serum free T4.
- Many drugs can alter thyroid hormone levels.
- Thyroid hormone therapy is not recommended for critically ill patients with low T3, low T4, or both.
- During recovery from nonthyroidal illness, some patients may have mild elevation in serum TSH levels (< 20 mU/L).
- Thyroid hormone levels may take several weeks or months to return to normal after the acute illness.
- Patients with serum TSH levels higher than 20 mU/L during the recovery phase of their nonthyroidal illness are more likely to have permanent primary hypothyroidism.
- Lamb EJ, Martin J. Thyroid function tests: often justified in the acutely ill. Ann Clin Biochem 2000; 37(pt 2):158–164. doi:10.1258/0004563001899159
- Spencer CA, LoPresti JS, Patel A, et al. Applications of a new chemiluminometric thyrotropin assay to subnormal measurement. J Clin Endocrinol Metab 1990; 70(2):453–460. doi:10.1210/jcem-70-2-453
- Ross DS, Ardisson LJ, Meskell MJ. Measurement of thyrotropin in clinical and subclinical hyperthyroidism using a new chemiluminescent assay. J Clin Endocrinol Metab 1989; 69(3):684–688. doi:10.1210/jcem-69-3-684
- Koulouri O, Moran C, Halsall D, Chatterjee K, Gurnell M. Pitfalls in the measurement and interpretation of thyroid function tests. Best Pract Res Clin Endocrinol Metab 2013; 27(6):745–762. doi:10.1016/j.beem.2013.10.003
- Lechan RM, Fekete C. Role of thyroid hormone deiodination in the hypothalamus. Thyroid 2005; 15(8):883–897. doi:10.1089/thy.2005.15.883
- Chopra IJ, Hershman JM, Pardridge WM, Nicoloff JT. Thyroid function in nonthyroidal ilnesses. Ann Intern Med 1983; 98(6):946–957. doi:10.7326/0003-4819-98-6-946
- Chopra IJ, Solomon DH, Hepner HW, Mortenstein AA. Misleadingly low free thyroxine index and usefulness of reverse triiodothyronine measurement in nonthyroidal illnesses. Ann Intern Med 1979; 90(6):905–912. doi:10.7326/0003-4819-90-6-905
- Pontecorvi A, Robbins J. The plasma membrane and thyroid hormone entry into cells. Trends Endocrinol Metab 1989; 1(2):90–94. pmid:18411097
- Hennemann G, Krenning EP. Pitfalls in the interpretation of thyroid function tests in old age and non-thyroidal illness. Horm Res 1987; 26(1–4):100–104. doi:10.1159/000180688
- Baloch Z, Carayon P, Conte-Devolx B, et al; Guidelines Committee, National Academy of Clinical Biochemistry. Laboratory medicine practice guidelines. Laboratory support for the diagnosis and monitoring of thyroid disease. Thyroid 2003; 13(1):3–126. doi:10.1089/105072503321086962
- Lum S, Nicoloff JT, Spencer CA, Kaptein EM. Peripheral tissue mechanism for maintenance of serum triiodothyronine values in a thyroxine-deficient state in man. J Clin Invest 1984; 73(2):570–575. doi:10.1172/JCI111245
- Ortiga-Carvalho TM, Chiamolera MI, Pazos-Moura CC, Wondisford FE. Hypothalamus-pituitary-thyroid axis. Compr Physiol 2016; 6(3):1387–1428. doi:10.1002/cphy.c150027
- de Vries EM, Fliers E, Boelen A. The molecular basis of the non-thyroidal illness syndrome. J Endocrinol 2015; 225(3):R67–R81. doi:10.1530/JOE-15-0133
- Chopra IJ, Huang TS, Beredo A, Solomon DH, Teco GN, Mean JF. Evidence for an inhibitor of extrathyroidal conversion of thyroxine to 3, 5, 3'-triiodothyronine in sera of patients with nonthyroidal illnesses. J Clin Endocrinol Metab 1985; 60(4):666–672. doi:10.1210/jcem-60-4-666
- Peeters RP, Debaveye Y, Fliers E, Visser TJ. Changes within the thyroid axis during critical illness. Crit Care Clin 2006; 22(1):41–55. doi:10.1016/j.ccc.2005.08.006
- Spencer C, Eigen A, Shen D, et al. Specificity of sensitive assays of thyrotropin (TSH) used to screen for thyroid disease in hospitalized patients. Clin Chem 1987; 33(8):1391–1396. pmid:3301067
- Adler SM, Wartofsky L. The nonthyroidal illness syndrome. Endocrinol Metab Clin North Am 2007; 36(3):657–672. doi:10.1016/j.ecl.2007.04.007
- Persani L. Central hypothyroidism: pathogenic, diagnostic, and therapeutic challenges. J Clin Endocrinol Metab 2012; 97(9):3068–3078. doi:10.1210/jc.2012-1616
- Kidess AI, Caplan RH, Reynertson RH, Wickus GG, Goodnough DE. Transient corticotropin deficiency in critical illness. Mayo Clin Proc 1993; 68(5):435–441. doi:10.1016/s0025-6196(12)60188-8
- Lamberts SW, Bruining HA, De Jong FH. Corticosteroid therapy in severe illness. N Engl J Med 1997; 337(18):1285–1292. doi:10.1056/NEJM199710303371807
- Burmeister LA. Reverse T3 does not reliably differentiate hypothyroid sick syndrome from euthyroid sick syndrome. Thyroid 1995; 5(6):435–441. doi:10.1089/thy.1995.5.435
- Duick DS, Warren DW, Nicoloff JT, Otis CL, Croxson MS. Effect of single dose dexamethasone on the concentration of serum triiodothyronine in man. J Clin Endocrinol Metab 1974; 39(6):1151–1154. doi:10.1210/jcem-39-6-1151
- Gamstedt A, Järnerot G, Kågedal B. Dose related effects of betamethasone on iodothyronines and thyroid hormone-binding proteins in serum. Acta Endocrinol (Copenh) 1981; 96(4):484–490. doi:10.1530/acta.0.0960484
- Wartofsky L, Burman KD. Alterations in thyroid function in patients with systemic illness: the “euthyroid sick syndrome.” Endocr Rev 1982; 3(2):164–217. doi:10.1210/edrv-3-2-164
- Wilber JF, Utiger RD. The effect of glucocorticoids on thyrotropin secretion. J Clin Invest 1969; 48(11):2096–2103. doi:10.1172/JCI106176
- Nicoloff JT, Fisher DA, Appleman MD Jr. The role of glucocorticoids in the regulation of thyroid function in man. J Clin Invest 1970; 49(10):1922–1929. doi:10.1172/JCI106411
- Surks MI, Sievert R. Drugs and thyroid function. N Engl J Med 1995; 333(25):1688–1694. doi:10.1056/NEJM199512213332507
- Haugen BR. Drugs that suppress TSH or cause central hypothyroidism. Best Pract Res Clin Endocrinol Metab 2009; 23(6):793–800. doi:10.1016/j.beem.2009.08.003
- Sherman SI, Gopal J, Haugen BR, et al. Central hypothyroidism associated with retinoid X receptor–selective ligands. N Engl J Med 1999; 340(14):1075–1079. doi:10.1056/NEJM199904083401404
- Murchison LE, How J, Bewsher PD. Comparison of propranolol and metoprolol in the management of hyperthyroidism. Br J Clin Pharmacol 1979; 8(6):581–587. doi:10.1111/j.1365-2125.1979.tb01048.x
- Faber J, Friis T, Kirkegaard C, et al. Serum T4, T3 and reverse T3 during treatment with propranolol in hyperthyroidism, L-T4 treated myxedema and in normal man. Horm Metab Res 1979; 11(1):34–36. doi:10.1055/s-0028-1092678
- Kristensen BO, Weeke J. Propranolol-induced increments in total and free serum thyroxine in patients with essential hypertension. Clin Pharmacol Ther 1977; 22(6):864–867. doi:10.1002/cpt1977226864
- Murchison LE, Bewsher PD, Chesters MI, Ferrier WR. Comparison of propranolol and practolol in the management of hyperthyroidism. Br J Clin Pharmacol 1976; 3(2):273–277. doi:10.1111/j.1365-2125.1976.tb00603.x
- Lotti G, Delitala G, Devilla L, Alagna S, Masala A. Reduction of plasma triiodothyronine (T3) induced by propranolol. Clin Endocrinol 1977; 6(6):405–410. doi:10.1111/j.1365-2265.1977.tb03322.x
- Perrild H, Hansen JM, Skovsted L, Christensen LK. Different effects of propranolol, alprenolol, sotalol, atenolol and metoprolol on serum T3 and serum rT3 in hyperthyroidism. Clin Endocrinol (Oxf) 1983; 18(2):139–142. pmid:6133659
- Reeves RA, From GL, Paul W, Leenen FH. Nadolol, propranolol, and thyroid hormones: evidence for a membrane-stabilizing action of propranolol. Clin Pharmacol Ther 1985; 37(2):157–161. doi:10.1038/clpt.1985.28
- Walker N, Jung RT, Jennings G, James WP. The effect of a beta-receptor agonist (salbutamol) on peripheral thyroid metabolism in euthyroid subjects. Horm Metab Res 1981; 13(10):590–591. doi:10.1055/s-2007-1019346
- Melmed S, Geola FL, Reed AW, Pekary AE, Park J, Hershman JM. A comparison of methods for assessing thyroid function in nonthyroidal illness. J Clin Endocrinol Metab 1982; 54(2):300–306. doi:10.1210/jcem-54-2-300
- Docter R, Krenning E, De Jong M, Hennemann G. The sick euthyroid syndrome: changes in thyroid hormone serum parameters and hormone metabolism. Clin Endocrinol (Oxf) 1993; 39(5):499–518. pmid:8252737
- Mariotti S, Caturegli P, Piccolo P, Barbesino G, Pinchera A. Antithyroid peroxidase autoantibodies in thyroid diseases. J Clin Endocrinol Metab 1990; 71(3):661–669. doi:10.1210/jcem-71-3-661
- De Groot LJ. Non-thyroidal illness syndrome is a manifestation of hypothalamic-pituitary dysfunction, and in view of current evidence, should be treated with appropriate replacement therapies. Crit Care Clin 2006; 22(1):57–86. doi:10.1016/j.ccc.2005.10.001
- Jonklaas J, Bianco AC, Bauer AJ, et al; American Thyroid Association Task Force on Thyroid Hormone Replacement. Guidelines for the treatment of hypothyroidism: prepared by the American Thyroid Association Task Force on Thyroid Hormone Replacement. Thyroid 2014; 24(12):1670–1751. doi:10.1089/thy.2014.0028
- Kaptein EM, Beale E, Chan LS. Thyroid hormone therapy for obesity and nonthyroidal illnesses: a systematic review. J Clin Endocrinol Metab 2009; 94(10):3663–3675. doi:10.1210/jc.2009-0899
- Vidart J, Wajner SM, Leite RS, et al. N-acetylcysteine administration prevents nonthyroidal illness syndrome in patients with acute myocardial infarction: a randomized clinical trial. J Clin Endocrinol Metab 2014; 99(12):4537–4545. doi:10.1210/jc.2014-2192
- Van den Berghe G, Wouters P, Weekers F, et al. Reactivation of pituitary hormone release and metabolic improvement by infusion of growth hormone-releasing peptide and thyrotropin-releasing hormone in patients with protracted critical illness. J Clin Endocrinol Metab 1999; 84(4):1311–1323. doi:10.1210/jcem.84.4.5636
- Langouche L, Vander Perre S, Marques M, et al. Impact of early nutrient restriction during critical illness on the nonthyroidal illness syndrome and its relation with outcome: a randomized, controlled clinical study. J Clin Endocrinol Metab 2013; 98(3):1006–1013. doi:10.1210/jc.2012-2809
- Economidou F, Douka E, Tzanela M, Nanas S, Kotanidou A. Thyroid function during critical illness. Hormones (Athens) 2011; 10(2):117–124. doi:10.14310/horm.2002.1301
- Hamblin PS, Dyer SA, Mohr VS, et al. Relationship between thyrotropin and thyroxine changes during recovery from severe hypothyroxinemia of critical illness. J Clin Endocrinol Metab 1986; 62(4):717–722. doi:10.1210/jcem-62-4-717
- Iglesias P, Diez JJ. Thyroid dysfunction and kidney disease. Eur J Endocrinol 2009; 160(4):503–515. doi:10.1530/EJE-08-0837
- Spencer CA. Clinical utility and cost-effectiveness of sensitive thyrotropin assays in ambulatory and hospitalized patients. Mayo Clin Proc 1988; 63(12):1214–1222. doi:10.1016/s0025-6196(12)65408-1
- Attia J, Margetts P, Guyatt G. Diagnosis of thyroid disease in hospitalized patients: a systematic review. Arch Intern Med 1999; 159(7):658–665. pmid:10218744
- Barbesino G, Tomer Y. Clinical review: clinical utility of TSH receptor antibodies. J Clin Endocrinol Metab 2013; 98(6):2247–2255. doi:10.1210/jc.2012-4309
A 66-year-old man presented to the emergency department with increasing shortness of breath and productive cough, which had begun 5 days earlier. Three years previously, he had been diagnosed with chronic obstructive pulmonary disease (COPD).
One week before the current presentation, he developed a sore throat, rhinorrhea, and nasal congestion, and the shortness of breath had started 2 days after that. Although he could speak in sentences, he was breathless even at rest. His dyspnea was associated with noisy breathing and cough productive of yellowish sputum; there was no hemoptysis. He reported fever, but he had no chills, night sweats, chest pain, or paroxysmal nocturnal dyspnea. The review of other systems was unremarkable.
His COPD was known to be mild, in Global Initiative for Chronic Obstructive Lung Disease (GOLD) grade 1, group A. His postbronchodilator ratio of forced expiratory volume in 1 second (FEV1) to forced vital capacity (FVC) was less than 0.70, and his FEV1 was 84% of predicted. Apart from mild intermittent cough with white sputum, his COPD had been under good control with inhaled ipratropium 4 times daily and inhaled albuterol as needed. He said he did not have shortness of breath except when hurrying on level ground or walking up a slight hill (Modified Medical Research Council dyspnea scale grade 1; COPD Assessment Test score < 10). In the last 3 years, he had 2 exacerbations of COPD, 1 year apart, both requiring oral prednisone and antibiotic therapy.
Other relevant history included hypertension and dyslipidemia of 15-year duration, for which he was taking candesartan 16 mg twice daily and atorvastatin 20 mg daily. He was compliant with his medications.
Though he usually received an influenza vaccine every year, he did not get it the previous year. Also, 3 years previously, he received the 23-valent pneumococcal polysaccharide vaccine (PPSV23), and the year before that he received the pneumococcal conjugate vaccine (PCV13). In addition, he was immunized against herpes zoster and tetanus.
The patient had smoked 1 pack per day for the past 38 years. His primary care physician had advised him many times to quit smoking. He had enrolled in a smoking cessation program 2 years previously, in which he received varenicline in addition to behavioral counseling in the form of motivational interviewing and a telephone quit-line. Nevertheless, he continued to smoke.
He was a retired engineer. He did not drink alcohol or use illicit drugs.
PHYSICAL EXAMINATION
On physical examination, the patient was sitting up in bed, leaning forward. He was alert and oriented but was breathing rapidly and looked sick. He had no cyanosis, clubbing, pallor, or jaundice. His blood pressure was 145/90 mm Hg, heart rate 110 beats per minute and regular, respiratory rate 29 breaths per minute, and oral temperature 38.1°C (100.6°F). His oxygen saturation was 88% while breathing room air. His body mass index was 27.1 kg/m2.
His throat was mildly congested. His neck veins were flat, and there were no carotid bruits. His thyroid examination was normal, without goiter, nodules, or tenderness.
Intercostal retractions were noted around the anterolateral costal margins. He had no chest wall deformities. Chest expansion was reduced bilaterally. There was hyperresonance bilaterally. Expiratory wheezes were heard over both lungs, without crackles.
His heart had no murmurs or added sounds. There was no lower-limb edema or swelling. The rest of his physical examination was unremarkable.
Results of initial laboratory testing are shown in Table 1.
Assessment: A 66-year-old man with GOLD grade 1, group A COPD, presenting with a severe exacerbation, most likely due to viral bronchitis.
INITIAL MANAGEMENT
The patient was given oxygen 28% by Venturi mask, and his oxygen saturation went up to 90%. He was started on nebulized albuterol 2.5 mg with ipratropium bromide 500 µg every 4 hours, prednisone 40 mg orally daily for 5 days, and ceftriaxone 1 g intravenously every 24 hours. The first dose of each medication was given in the emergency department.
The patient was then admitted to a progressive care unit, where he was placed on noninvasive positive pressure ventilation, continuous cardiac monitoring, and pulse oximetry. He was started on enoxaparin 40 mg subcutaneously daily to prevent venous thromboembolism, and the oral medications he had been taking at home were continued. Because he was receiving a glucocorticoid, his blood glucose was monitored in the fasting state, 2 hours after each meal, and as needed.
Two hours after he started noninvasive positive pressure ventilation, his arterial blood gases were remeasured and showed the following results:
- pH 7.35
- Partial pressure of carbon dioxide (Paco2) 52 mm Hg
- Bicarbonate 28 mmol/L
- Partial pressure of oxygen (Pao2) 60 mm Hg
- Oxygen saturation 90%.
HOSPITAL COURSE
On hospital day 3, his dyspnea had slightly improved. His respiratory rate was 26 to 28 breaths per minute. His oxygen saturation remained between 90% and 92%.
At 10:21 pm, his cardiac monitor showed an episode of focal atrial tachycardia at a rate of 129 beats per minute that lasted for 3 minutes and 21 seconds, terminating spontaneously. He denied any change in his clinical condition during the episode, with no chest pain, palpitation, or change in dyspnea. There was no change in his vital signs. He had another similar asymptomatic episode lasting 4 minutes and 9 seconds at 6:30 am of hospital day 4.
Because of these episodes, the attending physician ordered thyroid function tests.
THYROID FUNCTION TESTING
1. Which thyroid function test is most likely to be helpful in the assessment of this patient’s thyroid status?
- Serum thyroid-stimulating hormone (TSH) alone
- Serum TSH and total thyroxine (T4)
- Serum TSH and total triiodothyronine (T3)
- Serum TSH and free T4
- Serum TSH and free T3
There are several tests to assess thyroid function: the serum TSH, total T4, free T4, total T3, and free T3 concentrations.1
In normal physiology, TSH from the pituitary stimulates the thyroid gland to produce and secrete T4 and T3, which in turn inhibit TSH secretion through negative feedback. A negative log-linear relation exists between serum free T4 and TSH levels.2 Thus, the serum free T4 level can remain within the normal reference range even if the TSH level is high or low.
TSH assays can have different detection limits. A third-generation TSH assay with a detection limit of 0.01 mU/L is recommended for use in clinical practice.3
TSH testing alone. Given its superior sensitivity and specificity, serum TSH measurement is considered the best single test for assessing thyroid function in most cases.4 Nevertheless, measurement of the serum TSH level alone could be misleading in several situations, eg, hypothalamic or pituitary disorders, recent treatment of thyrotoxicosis, impaired sensitivity to thyroid hormone, and acute nonthyroidal illness.4
Free vs total T4 and T3 levels
Serum total T4 includes a fraction that is bound, mainly to thyroxin-binding globulin, and a very small unbound (free) fraction. The same applies to T3. Only free thyroid hormones represent the “active” fraction available for interaction with their protein receptors in the nucleus.8 Patients with conditions that can affect the thyroid-binding protein concentrations usually have altered serum total T4 and T3 levels, whereas their free hormone concentrations remain normal. Accordingly, measurement of free hormone levels, especially free T4, is usually recommended.
Although equilibrium dialysis is the method most likely to provide an accurate serum free T4 measurement, it is not commonly used because of its limited availability and high cost. Thus, most commercial laboratories use “direct” free T4 measurement or, to a lesser degree, the free T4 index.9 However, none of the currently available free T4 tests actually measure free T4 directly; rather, they estimate it.10
Commercial laboratories can provide a direct free T3 estimate, but it may be less reliable than total T3. If serum T3 measurement is indicated, serum total T3 is usually measured. However, total T3 measurement is rarely indicated for patients with hypothyroidism because it usually remains within the normal reference range.11 Nevertheless, serum total T3 measurement could be useful in patients with T3 toxicosis and in those who are acutely ill.
Accordingly, in acutely ill hospitalized patients like ours, measuring serum TSH using a third-generation assay and free T4 is essential to assess thyroid function. Many clinicians also measure serum total T3.
CASE CONTINUED: LOW TSH, LOW-NORMAL FREE T4, LOW TOTAL T3
The attending physician ordered serum TSH, free T4, and total T3 measurements, which yielded the following:
- TSH 0.1 mU/L (0.5–5.0)
- Total T3 55 ng/dL (80–180)
- Free T4 0.9 ng/dL (0.9–2.4).
2. Which best explains this patient’s abnormal thyroid test results?
- His acute illness
- Central hypothyroidism due to pituitary infarction
- His albuterol therapy
- Subclinical thyrotoxicosis
- Hashimoto thyroiditis
Since euthyroid patients with an acute illness may have abnormal thyroid test results (Table 2),5–7 thyroid function testing is not recommended unless there is a strong indication for it, such as new-onset atrial fibrillation, atrial flutter, or focal atrial tachycardia.1 In such patients, it is important to know whether the test abnormalities represent true thyroid disorder or are the result of a nonthyroidal illness.
In healthy people, T4 is converted to T3 (the principal active hormone) by type 1 deiodinase (D1) mainly in the liver and kidneys, whereas this reaction is catalyzed by type 2 deiodinase (D2) in the hypothalamus and pituitary. Type 3 deiodinase (D3) converts T4 to reverse T3, a biologically inactive molecule.12 D1 also mediates conversion of reverse T3 to diiodothyronine (T2) (Figure 1).
Thyroid function testing in patients with nonthyroidal illness usually shows low serum total T3, normal or low serum TSH, and normal, low, or high serum free T4. However, transient mild serum TSH elevation can be seen in some patients during the recovery period.16 These abnormalities with their mechanisms are shown in Table 2.5–7 In several commercial kits, serum direct free T4 can be falsely decreased or increased.8
THE DIFFERENTIAL DIAGNOSIS
Our patient had low serum TSH, low-normal serum direct free T4, and low serum total T3. This profile could be caused by a nonthyroidal illness, “true” central hypothyroidism, or his glucocorticoid treatment. The reason we use the term “true” in this setting is that some experts suggest that the thyroid function test abnormalities in patients with acute nonthyroidal illness represent a transient central hypothyroidism.17 The clinical presentation is key in differentiating true central hypothyroidism from nonthyroidal illness.
In addition, measuring serum cortisol may help to differentiate between the 2 states, as it would be elevated in patients with nonthyroidal illness as part of a stress response but low in patients with true central hypothyroidism, since it is usually part of combined pituitary hormone deficiency.18 Of note, some critically ill patients have low serum cortisol because of transient central adrenal insufficiency.19,20
The serum concentration of reverse T3 has been suggested as a way to differentiate between hypothyroidism (low) and nonthyroidal illness (high); however, further studies showed that it does not reliably differentiate between the conditions.21
GLUCOCORTICOIDS AND THYROID FUNCTION TESTS
By inhibiting D1, glucocorticoids can decrease peripheral conversion of T4 to T3 and thus decrease serum total T3. This effect depends on the type and dose of the glucocorticoid and the duration of therapy.
In one study,22 there was a significant reduction in serum total T3 concentration 24 hours after a single oral dose of dexamethasone 12 mg in normal participants. This effect lasted 48 hours, after which serum total T3 returned to its pretreatment level.
In another study,23 a daily oral dose of betamethasone 1.5 mg for 5 days did not significantly reduce the serum total T3 in healthy volunteers, but a daily dose of 3 mg did. This effect was more pronounced at a daily dose of 4.5 mg, whereas a dose of 6.0 mg had no further effect.
Long-term glucocorticoid therapy also decreases serum total T4 and total T3 by lowering serum thyroid-binding globulin.24
Finally, glucocorticoids can decrease TSH secretion by directly inhibiting thyrotropin-releasing hormone.25,26 However, chronic hypercortisolism, whether endogenous or exogenous, does not cause clinically central hypothyroidism, possibly because of the negative feedback mechanism of low thyroid hormones on the pituitary and the hypothalamus.27
Other drugs including dopamine, dopamine agonists, dobutamine, and somatostatin analogues can suppress serum TSH. As with glucocorticoids, these drugs do not cause clinically evident central hypothyroidism.28 Bexarotene, a retinoid X receptor ligand used in the treatment of cutaneous T-cell lymphoma, has been reported to cause clinically evident central hypothyroidism by suppressing TSH and increasing T4 clearance.29
BETA-BLOCKERS, BETA-AGONISTS AND THYROID FUNCTION
While there is general agreement that beta-adrenergic antagonists (beta-blockers) do not affect the serum TSH concentration, conflicting data have been reported concerning their effect on other thyroid function tests. This may be due to several factors, including dose, duration of therapy, the patient’s thyroid status, and differences in laboratory methodology.30
In studies of propranolol, serum total T4 concentrations did not change or were increased with daily doses of 160 mg or more in both euthyroid participants and hyperthyroid patients31–33; serum total T3 concentrations did not change or were decreased with 40 mg or more daily34; and serum reverse T3 concentrations were increased with daily doses of 80 mg or more.31 It is most likely that propranolol exerts these changes by inhibiting D1 activity in peripheral tissues.
Furthermore, a significant decrease in serum total T3 concentrations was observed in hyperthyroid patients treated with atenolol 100 mg daily, metoprolol 100 mg daily, and alprenolol 100 mg daily, but not with sotalol 80 mg daily or nadolol (up to 240 mg daily).35,36
On the other hand, beta-adrenergic agonists have not been reported to cause significant changes in thyroid function tests.37
SUBCLINICAL THYROTOXICOSIS OR HASHIMOTO THYROIDITIS?
Our patient’s thyroid function test results are more likely due to his nonthyroidal illness and glucocorticoid therapy, as there is no clinical evidence to point to a hypothalamic-pituitary disorder accounting for true central hypothyroidism.
The other options mentioned in question 2 are unlikely to explain our patient’s thyroid function test results.
Subclinical thyrotoxicosis is characterized by suppressed serum TSH, but both serum free T4 and total T3 remain within the normal reference ranges. In addition, the serum TSH level may help to differentiate between thyrotoxicosis and nonthyroidal illness. In the former, serum TSH is usually suppressed (< 0.01 mU/L), whereas in the latter it is usually low but detectable (0.05– 0.3 mU/L).38,39
Hashimoto thyroiditis is a chronic autoimmune thyroid disease characterized by diffuse lymphocytic infiltration of the thyroid gland. Almost all patients with Hashimoto thyroiditis have elevated levels of antibodies to thyroid peroxidase or thyroglobulin.40 Clinically, patients with Hashimoto thyroiditis can either be hypothyroid or have normal thyroid function, which is not the case in our patient.
CASE CONTINUED
An endocrinologist, consulted for a second opinion, agreed that the patient’s thyroid function test results were most likely due to his nonthyroidal illness and glucocorticoid therapy.
3. In view of the endocrinologist’s opinion, which should be the next step in the management of the patient’s thyroid condition?
- Start levothyroxine (T4) therapy
- Start liothyronine (T3) therapy
- Start N-acetylcysteine therapy
- Start thyrotropin-releasing hormone therapy
- Remeasure thyroid hormones after full recovery from his acute illness
It is not clear whether the changes in thyroid hormone levels during an acute illness are a pathologic alteration for which thyroid hormone therapy may be beneficial, or a physiologic adaptation for which such therapy would not be indicated.41
However, current data argue against thyroid hormone therapy using T4 or T3 for patients with nonthyroidal illness syndrome (also called euthyroid sick syndrome).42 Indeed, several randomized controlled trials showed that thyroid hormone therapy is not beneficial in such patients and may be detrimental.41,43
Therapies other than thyroid hormone have been investigated to ameliorate thyroid hormone abnormalities in patients with nonthyroidal illness. These include N-acetylcysteine, thyrotropin-releasing hormone therapy, and nutritional support.
Some studies showed that giving N-acetylcysteine, an antioxidant, increased serum T3 and decreased serum reverse T3 concentrations in patients with acute myocardial infarction.44 Nevertheless, the mortality rate and length of hospitalization were not affected. Further studies are needed to know whether N-acetylcysteine therapy is beneficial for such patients.
Similarly, a study using a thyrotropin-releasing hormone analogue along with growth hormone-releasing peptide 2 showed an increase in serum TSH, T4, and T3 levels in critically ill patients.45 The benefit of this therapy has yet to be determined. On the other hand, early nutritional support was reported to prevent thyroid hormonal changes in patients postoperatively.46
Measuring thyroid hormone levels after full recovery is the most appropriate next step in our patient, as the changes in thyroid hormone concentrations subside as the acute illness resolves.47
CASE CONTINUED
The patient continued to improve. On hospital day 6, he was feeling better but still had mild respiratory distress. There had been no further episodes of arrhythmia since day 4. His blood pressure was 136/86 mm Hg, heart rate 88 beats per minute and regular, respiratory rate 18 breaths per minute, and oral temperature 37.1°C. His oxygen saturation was 92% on room air.
Before discharge, he was encouraged to quit smoking. He was offered behavioral counseling and medication therapy, but he only said that he would think about it. He was discharged on oral cefixime for 4 more days and was instructed to switch to a long-acting bronchodilator along with his other home medications and to return in 1 week to have his thyroid hormones checked.
One week later, his laboratory results were:
- TSH 11.2 mU/L (reference range 0.5–5.0)
- Free T4 1.2 ng/dL (0.9–2.4)
- Total T3 92 ng/dL (80–180).
Clinically, the patient was euthyroid, and examination of his thyroid was unremarkable.
4. Based on these last test results, which statement is correct?
- Levothyroxine therapy should be started
- His serum TSH elevation is most likely transient
- Thyroid ultrasonography is strongly indicated
- A radioactive iodine uptake study should be performed
- Measurement of thyroid-stimulating immunoglobulins is indicated
During recovery from nonthyroidal illness, some patients may have elevated serum TSH levels that are usually transient and modest (< 20 mU/L).48 Normalization of the thyroid function tests including serum TSH may take several weeks49 or months.50 However, a systematic review found that the likelihood of permanent primary hypothyroidism is high in patients with serum TSH levels higher than 20 mU/L during the recovery phase of their nonthyroidal illness.51
Ultrasonography is useful for evaluating patients with thyroid nodules or goiter but is of little benefit for patients like ours, in whom the thyroid is normal on examination.
Similarly, a radioactive iodine uptake study is not indicated, as it is principally used to help differentiate between types of thyrotoxicosis. (Radioactive iodine is also used to treat differentiated thyroid cancer.)
Thyroid-stimulating immunoglobins are TSH receptor-stimulating antibodies that cause Graves disease. Nevertheless, measuring them is not routinely indicated for its diagnosis. However, their measurement is of significant help in the diagnosis of Graves disease if a radioactive iodine uptake study cannot be performed (as in pregnancy) and in atypical presentations such as euthyroid Graves ophthalmopathy.52 Other indications for thyroid-stimulating immunoglobin measurement are beyond the scope of the article. Our patient’s test results are not consistent with hyperthyroidism, so measuring thyroid-stimulating immunoglobins is not indicated.
CASE CONCLUSION: BETTER, BUT STILL SMOKING
The patient missed his 1-month clinic follow-up, but he visited the clinic for follow-up 3 months later. He was feeling well with no complaints. Test results including serum TSH, free T4, and total T3 were within normal ranges. His COPD was under control, with an FEV1 88% of predicted.
He was again encouraged to quit smoking and was offered drug therapy and behavioral counseling, but he declined. In addition, he was instructed to adhere to his annual influenza vaccination.
KEY POINTS
- In patients with acute illness, it is recommended that thyroid function not be assessed unless there is a strong indication.
- If thyroid function assessment is indicated for critically ill patients, serum TSH and free T4 concentrations should be measured. Some clinicians also measure serum total T3 level.
- Thyroid function testing in critically ill patients usually shows low serum total T3, normal or low serum TSH, and normal or low serum free T4.
- Many drugs can alter thyroid hormone levels.
- Thyroid hormone therapy is not recommended for critically ill patients with low T3, low T4, or both.
- During recovery from nonthyroidal illness, some patients may have mild elevation in serum TSH levels (< 20 mU/L).
- Thyroid hormone levels may take several weeks or months to return to normal after the acute illness.
- Patients with serum TSH levels higher than 20 mU/L during the recovery phase of their nonthyroidal illness are more likely to have permanent primary hypothyroidism.
A 66-year-old man presented to the emergency department with increasing shortness of breath and productive cough, which had begun 5 days earlier. Three years previously, he had been diagnosed with chronic obstructive pulmonary disease (COPD).
One week before the current presentation, he developed a sore throat, rhinorrhea, and nasal congestion, and the shortness of breath had started 2 days after that. Although he could speak in sentences, he was breathless even at rest. His dyspnea was associated with noisy breathing and cough productive of yellowish sputum; there was no hemoptysis. He reported fever, but he had no chills, night sweats, chest pain, or paroxysmal nocturnal dyspnea. The review of other systems was unremarkable.
His COPD was known to be mild, in Global Initiative for Chronic Obstructive Lung Disease (GOLD) grade 1, group A. His postbronchodilator ratio of forced expiratory volume in 1 second (FEV1) to forced vital capacity (FVC) was less than 0.70, and his FEV1 was 84% of predicted. Apart from mild intermittent cough with white sputum, his COPD had been under good control with inhaled ipratropium 4 times daily and inhaled albuterol as needed. He said he did not have shortness of breath except when hurrying on level ground or walking up a slight hill (Modified Medical Research Council dyspnea scale grade 1; COPD Assessment Test score < 10). In the last 3 years, he had 2 exacerbations of COPD, 1 year apart, both requiring oral prednisone and antibiotic therapy.
Other relevant history included hypertension and dyslipidemia of 15-year duration, for which he was taking candesartan 16 mg twice daily and atorvastatin 20 mg daily. He was compliant with his medications.
Though he usually received an influenza vaccine every year, he did not get it the previous year. Also, 3 years previously, he received the 23-valent pneumococcal polysaccharide vaccine (PPSV23), and the year before that he received the pneumococcal conjugate vaccine (PCV13). In addition, he was immunized against herpes zoster and tetanus.
The patient had smoked 1 pack per day for the past 38 years. His primary care physician had advised him many times to quit smoking. He had enrolled in a smoking cessation program 2 years previously, in which he received varenicline in addition to behavioral counseling in the form of motivational interviewing and a telephone quit-line. Nevertheless, he continued to smoke.
He was a retired engineer. He did not drink alcohol or use illicit drugs.
PHYSICAL EXAMINATION
On physical examination, the patient was sitting up in bed, leaning forward. He was alert and oriented but was breathing rapidly and looked sick. He had no cyanosis, clubbing, pallor, or jaundice. His blood pressure was 145/90 mm Hg, heart rate 110 beats per minute and regular, respiratory rate 29 breaths per minute, and oral temperature 38.1°C (100.6°F). His oxygen saturation was 88% while breathing room air. His body mass index was 27.1 kg/m2.
His throat was mildly congested. His neck veins were flat, and there were no carotid bruits. His thyroid examination was normal, without goiter, nodules, or tenderness.
Intercostal retractions were noted around the anterolateral costal margins. He had no chest wall deformities. Chest expansion was reduced bilaterally. There was hyperresonance bilaterally. Expiratory wheezes were heard over both lungs, without crackles.
His heart had no murmurs or added sounds. There was no lower-limb edema or swelling. The rest of his physical examination was unremarkable.
Results of initial laboratory testing are shown in Table 1.
Assessment: A 66-year-old man with GOLD grade 1, group A COPD, presenting with a severe exacerbation, most likely due to viral bronchitis.
INITIAL MANAGEMENT
The patient was given oxygen 28% by Venturi mask, and his oxygen saturation went up to 90%. He was started on nebulized albuterol 2.5 mg with ipratropium bromide 500 µg every 4 hours, prednisone 40 mg orally daily for 5 days, and ceftriaxone 1 g intravenously every 24 hours. The first dose of each medication was given in the emergency department.
The patient was then admitted to a progressive care unit, where he was placed on noninvasive positive pressure ventilation, continuous cardiac monitoring, and pulse oximetry. He was started on enoxaparin 40 mg subcutaneously daily to prevent venous thromboembolism, and the oral medications he had been taking at home were continued. Because he was receiving a glucocorticoid, his blood glucose was monitored in the fasting state, 2 hours after each meal, and as needed.
Two hours after he started noninvasive positive pressure ventilation, his arterial blood gases were remeasured and showed the following results:
- pH 7.35
- Partial pressure of carbon dioxide (Paco2) 52 mm Hg
- Bicarbonate 28 mmol/L
- Partial pressure of oxygen (Pao2) 60 mm Hg
- Oxygen saturation 90%.
HOSPITAL COURSE
On hospital day 3, his dyspnea had slightly improved. His respiratory rate was 26 to 28 breaths per minute. His oxygen saturation remained between 90% and 92%.
At 10:21 pm, his cardiac monitor showed an episode of focal atrial tachycardia at a rate of 129 beats per minute that lasted for 3 minutes and 21 seconds, terminating spontaneously. He denied any change in his clinical condition during the episode, with no chest pain, palpitation, or change in dyspnea. There was no change in his vital signs. He had another similar asymptomatic episode lasting 4 minutes and 9 seconds at 6:30 am of hospital day 4.
Because of these episodes, the attending physician ordered thyroid function tests.
THYROID FUNCTION TESTING
1. Which thyroid function test is most likely to be helpful in the assessment of this patient’s thyroid status?
- Serum thyroid-stimulating hormone (TSH) alone
- Serum TSH and total thyroxine (T4)
- Serum TSH and total triiodothyronine (T3)
- Serum TSH and free T4
- Serum TSH and free T3
There are several tests to assess thyroid function: the serum TSH, total T4, free T4, total T3, and free T3 concentrations.1
In normal physiology, TSH from the pituitary stimulates the thyroid gland to produce and secrete T4 and T3, which in turn inhibit TSH secretion through negative feedback. A negative log-linear relation exists between serum free T4 and TSH levels.2 Thus, the serum free T4 level can remain within the normal reference range even if the TSH level is high or low.
TSH assays can have different detection limits. A third-generation TSH assay with a detection limit of 0.01 mU/L is recommended for use in clinical practice.3
TSH testing alone. Given its superior sensitivity and specificity, serum TSH measurement is considered the best single test for assessing thyroid function in most cases.4 Nevertheless, measurement of the serum TSH level alone could be misleading in several situations, eg, hypothalamic or pituitary disorders, recent treatment of thyrotoxicosis, impaired sensitivity to thyroid hormone, and acute nonthyroidal illness.4
Free vs total T4 and T3 levels
Serum total T4 includes a fraction that is bound, mainly to thyroxin-binding globulin, and a very small unbound (free) fraction. The same applies to T3. Only free thyroid hormones represent the “active” fraction available for interaction with their protein receptors in the nucleus.8 Patients with conditions that can affect the thyroid-binding protein concentrations usually have altered serum total T4 and T3 levels, whereas their free hormone concentrations remain normal. Accordingly, measurement of free hormone levels, especially free T4, is usually recommended.
Although equilibrium dialysis is the method most likely to provide an accurate serum free T4 measurement, it is not commonly used because of its limited availability and high cost. Thus, most commercial laboratories use “direct” free T4 measurement or, to a lesser degree, the free T4 index.9 However, none of the currently available free T4 tests actually measure free T4 directly; rather, they estimate it.10
Commercial laboratories can provide a direct free T3 estimate, but it may be less reliable than total T3. If serum T3 measurement is indicated, serum total T3 is usually measured. However, total T3 measurement is rarely indicated for patients with hypothyroidism because it usually remains within the normal reference range.11 Nevertheless, serum total T3 measurement could be useful in patients with T3 toxicosis and in those who are acutely ill.
Accordingly, in acutely ill hospitalized patients like ours, measuring serum TSH using a third-generation assay and free T4 is essential to assess thyroid function. Many clinicians also measure serum total T3.
CASE CONTINUED: LOW TSH, LOW-NORMAL FREE T4, LOW TOTAL T3
The attending physician ordered serum TSH, free T4, and total T3 measurements, which yielded the following:
- TSH 0.1 mU/L (0.5–5.0)
- Total T3 55 ng/dL (80–180)
- Free T4 0.9 ng/dL (0.9–2.4).
2. Which best explains this patient’s abnormal thyroid test results?
- His acute illness
- Central hypothyroidism due to pituitary infarction
- His albuterol therapy
- Subclinical thyrotoxicosis
- Hashimoto thyroiditis
Since euthyroid patients with an acute illness may have abnormal thyroid test results (Table 2),5–7 thyroid function testing is not recommended unless there is a strong indication for it, such as new-onset atrial fibrillation, atrial flutter, or focal atrial tachycardia.1 In such patients, it is important to know whether the test abnormalities represent true thyroid disorder or are the result of a nonthyroidal illness.
In healthy people, T4 is converted to T3 (the principal active hormone) by type 1 deiodinase (D1) mainly in the liver and kidneys, whereas this reaction is catalyzed by type 2 deiodinase (D2) in the hypothalamus and pituitary. Type 3 deiodinase (D3) converts T4 to reverse T3, a biologically inactive molecule.12 D1 also mediates conversion of reverse T3 to diiodothyronine (T2) (Figure 1).
Thyroid function testing in patients with nonthyroidal illness usually shows low serum total T3, normal or low serum TSH, and normal, low, or high serum free T4. However, transient mild serum TSH elevation can be seen in some patients during the recovery period.16 These abnormalities with their mechanisms are shown in Table 2.5–7 In several commercial kits, serum direct free T4 can be falsely decreased or increased.8
THE DIFFERENTIAL DIAGNOSIS
Our patient had low serum TSH, low-normal serum direct free T4, and low serum total T3. This profile could be caused by a nonthyroidal illness, “true” central hypothyroidism, or his glucocorticoid treatment. The reason we use the term “true” in this setting is that some experts suggest that the thyroid function test abnormalities in patients with acute nonthyroidal illness represent a transient central hypothyroidism.17 The clinical presentation is key in differentiating true central hypothyroidism from nonthyroidal illness.
In addition, measuring serum cortisol may help to differentiate between the 2 states, as it would be elevated in patients with nonthyroidal illness as part of a stress response but low in patients with true central hypothyroidism, since it is usually part of combined pituitary hormone deficiency.18 Of note, some critically ill patients have low serum cortisol because of transient central adrenal insufficiency.19,20
The serum concentration of reverse T3 has been suggested as a way to differentiate between hypothyroidism (low) and nonthyroidal illness (high); however, further studies showed that it does not reliably differentiate between the conditions.21
GLUCOCORTICOIDS AND THYROID FUNCTION TESTS
By inhibiting D1, glucocorticoids can decrease peripheral conversion of T4 to T3 and thus decrease serum total T3. This effect depends on the type and dose of the glucocorticoid and the duration of therapy.
In one study,22 there was a significant reduction in serum total T3 concentration 24 hours after a single oral dose of dexamethasone 12 mg in normal participants. This effect lasted 48 hours, after which serum total T3 returned to its pretreatment level.
In another study,23 a daily oral dose of betamethasone 1.5 mg for 5 days did not significantly reduce the serum total T3 in healthy volunteers, but a daily dose of 3 mg did. This effect was more pronounced at a daily dose of 4.5 mg, whereas a dose of 6.0 mg had no further effect.
Long-term glucocorticoid therapy also decreases serum total T4 and total T3 by lowering serum thyroid-binding globulin.24
Finally, glucocorticoids can decrease TSH secretion by directly inhibiting thyrotropin-releasing hormone.25,26 However, chronic hypercortisolism, whether endogenous or exogenous, does not cause clinically central hypothyroidism, possibly because of the negative feedback mechanism of low thyroid hormones on the pituitary and the hypothalamus.27
Other drugs including dopamine, dopamine agonists, dobutamine, and somatostatin analogues can suppress serum TSH. As with glucocorticoids, these drugs do not cause clinically evident central hypothyroidism.28 Bexarotene, a retinoid X receptor ligand used in the treatment of cutaneous T-cell lymphoma, has been reported to cause clinically evident central hypothyroidism by suppressing TSH and increasing T4 clearance.29
BETA-BLOCKERS, BETA-AGONISTS AND THYROID FUNCTION
While there is general agreement that beta-adrenergic antagonists (beta-blockers) do not affect the serum TSH concentration, conflicting data have been reported concerning their effect on other thyroid function tests. This may be due to several factors, including dose, duration of therapy, the patient’s thyroid status, and differences in laboratory methodology.30
In studies of propranolol, serum total T4 concentrations did not change or were increased with daily doses of 160 mg or more in both euthyroid participants and hyperthyroid patients31–33; serum total T3 concentrations did not change or were decreased with 40 mg or more daily34; and serum reverse T3 concentrations were increased with daily doses of 80 mg or more.31 It is most likely that propranolol exerts these changes by inhibiting D1 activity in peripheral tissues.
Furthermore, a significant decrease in serum total T3 concentrations was observed in hyperthyroid patients treated with atenolol 100 mg daily, metoprolol 100 mg daily, and alprenolol 100 mg daily, but not with sotalol 80 mg daily or nadolol (up to 240 mg daily).35,36
On the other hand, beta-adrenergic agonists have not been reported to cause significant changes in thyroid function tests.37
SUBCLINICAL THYROTOXICOSIS OR HASHIMOTO THYROIDITIS?
Our patient’s thyroid function test results are more likely due to his nonthyroidal illness and glucocorticoid therapy, as there is no clinical evidence to point to a hypothalamic-pituitary disorder accounting for true central hypothyroidism.
The other options mentioned in question 2 are unlikely to explain our patient’s thyroid function test results.
Subclinical thyrotoxicosis is characterized by suppressed serum TSH, but both serum free T4 and total T3 remain within the normal reference ranges. In addition, the serum TSH level may help to differentiate between thyrotoxicosis and nonthyroidal illness. In the former, serum TSH is usually suppressed (< 0.01 mU/L), whereas in the latter it is usually low but detectable (0.05– 0.3 mU/L).38,39
Hashimoto thyroiditis is a chronic autoimmune thyroid disease characterized by diffuse lymphocytic infiltration of the thyroid gland. Almost all patients with Hashimoto thyroiditis have elevated levels of antibodies to thyroid peroxidase or thyroglobulin.40 Clinically, patients with Hashimoto thyroiditis can either be hypothyroid or have normal thyroid function, which is not the case in our patient.
CASE CONTINUED
An endocrinologist, consulted for a second opinion, agreed that the patient’s thyroid function test results were most likely due to his nonthyroidal illness and glucocorticoid therapy.
3. In view of the endocrinologist’s opinion, which should be the next step in the management of the patient’s thyroid condition?
- Start levothyroxine (T4) therapy
- Start liothyronine (T3) therapy
- Start N-acetylcysteine therapy
- Start thyrotropin-releasing hormone therapy
- Remeasure thyroid hormones after full recovery from his acute illness
It is not clear whether the changes in thyroid hormone levels during an acute illness are a pathologic alteration for which thyroid hormone therapy may be beneficial, or a physiologic adaptation for which such therapy would not be indicated.41
However, current data argue against thyroid hormone therapy using T4 or T3 for patients with nonthyroidal illness syndrome (also called euthyroid sick syndrome).42 Indeed, several randomized controlled trials showed that thyroid hormone therapy is not beneficial in such patients and may be detrimental.41,43
Therapies other than thyroid hormone have been investigated to ameliorate thyroid hormone abnormalities in patients with nonthyroidal illness. These include N-acetylcysteine, thyrotropin-releasing hormone therapy, and nutritional support.
Some studies showed that giving N-acetylcysteine, an antioxidant, increased serum T3 and decreased serum reverse T3 concentrations in patients with acute myocardial infarction.44 Nevertheless, the mortality rate and length of hospitalization were not affected. Further studies are needed to know whether N-acetylcysteine therapy is beneficial for such patients.
Similarly, a study using a thyrotropin-releasing hormone analogue along with growth hormone-releasing peptide 2 showed an increase in serum TSH, T4, and T3 levels in critically ill patients.45 The benefit of this therapy has yet to be determined. On the other hand, early nutritional support was reported to prevent thyroid hormonal changes in patients postoperatively.46
Measuring thyroid hormone levels after full recovery is the most appropriate next step in our patient, as the changes in thyroid hormone concentrations subside as the acute illness resolves.47
CASE CONTINUED
The patient continued to improve. On hospital day 6, he was feeling better but still had mild respiratory distress. There had been no further episodes of arrhythmia since day 4. His blood pressure was 136/86 mm Hg, heart rate 88 beats per minute and regular, respiratory rate 18 breaths per minute, and oral temperature 37.1°C. His oxygen saturation was 92% on room air.
Before discharge, he was encouraged to quit smoking. He was offered behavioral counseling and medication therapy, but he only said that he would think about it. He was discharged on oral cefixime for 4 more days and was instructed to switch to a long-acting bronchodilator along with his other home medications and to return in 1 week to have his thyroid hormones checked.
One week later, his laboratory results were:
- TSH 11.2 mU/L (reference range 0.5–5.0)
- Free T4 1.2 ng/dL (0.9–2.4)
- Total T3 92 ng/dL (80–180).
Clinically, the patient was euthyroid, and examination of his thyroid was unremarkable.
4. Based on these last test results, which statement is correct?
- Levothyroxine therapy should be started
- His serum TSH elevation is most likely transient
- Thyroid ultrasonography is strongly indicated
- A radioactive iodine uptake study should be performed
- Measurement of thyroid-stimulating immunoglobulins is indicated
During recovery from nonthyroidal illness, some patients may have elevated serum TSH levels that are usually transient and modest (< 20 mU/L).48 Normalization of the thyroid function tests including serum TSH may take several weeks49 or months.50 However, a systematic review found that the likelihood of permanent primary hypothyroidism is high in patients with serum TSH levels higher than 20 mU/L during the recovery phase of their nonthyroidal illness.51
Ultrasonography is useful for evaluating patients with thyroid nodules or goiter but is of little benefit for patients like ours, in whom the thyroid is normal on examination.
Similarly, a radioactive iodine uptake study is not indicated, as it is principally used to help differentiate between types of thyrotoxicosis. (Radioactive iodine is also used to treat differentiated thyroid cancer.)
Thyroid-stimulating immunoglobins are TSH receptor-stimulating antibodies that cause Graves disease. Nevertheless, measuring them is not routinely indicated for its diagnosis. However, their measurement is of significant help in the diagnosis of Graves disease if a radioactive iodine uptake study cannot be performed (as in pregnancy) and in atypical presentations such as euthyroid Graves ophthalmopathy.52 Other indications for thyroid-stimulating immunoglobin measurement are beyond the scope of the article. Our patient’s test results are not consistent with hyperthyroidism, so measuring thyroid-stimulating immunoglobins is not indicated.
CASE CONCLUSION: BETTER, BUT STILL SMOKING
The patient missed his 1-month clinic follow-up, but he visited the clinic for follow-up 3 months later. He was feeling well with no complaints. Test results including serum TSH, free T4, and total T3 were within normal ranges. His COPD was under control, with an FEV1 88% of predicted.
He was again encouraged to quit smoking and was offered drug therapy and behavioral counseling, but he declined. In addition, he was instructed to adhere to his annual influenza vaccination.
KEY POINTS
- In patients with acute illness, it is recommended that thyroid function not be assessed unless there is a strong indication.
- If thyroid function assessment is indicated for critically ill patients, serum TSH and free T4 concentrations should be measured. Some clinicians also measure serum total T3 level.
- Thyroid function testing in critically ill patients usually shows low serum total T3, normal or low serum TSH, and normal or low serum free T4.
- Many drugs can alter thyroid hormone levels.
- Thyroid hormone therapy is not recommended for critically ill patients with low T3, low T4, or both.
- During recovery from nonthyroidal illness, some patients may have mild elevation in serum TSH levels (< 20 mU/L).
- Thyroid hormone levels may take several weeks or months to return to normal after the acute illness.
- Patients with serum TSH levels higher than 20 mU/L during the recovery phase of their nonthyroidal illness are more likely to have permanent primary hypothyroidism.
- Lamb EJ, Martin J. Thyroid function tests: often justified in the acutely ill. Ann Clin Biochem 2000; 37(pt 2):158–164. doi:10.1258/0004563001899159
- Spencer CA, LoPresti JS, Patel A, et al. Applications of a new chemiluminometric thyrotropin assay to subnormal measurement. J Clin Endocrinol Metab 1990; 70(2):453–460. doi:10.1210/jcem-70-2-453
- Ross DS, Ardisson LJ, Meskell MJ. Measurement of thyrotropin in clinical and subclinical hyperthyroidism using a new chemiluminescent assay. J Clin Endocrinol Metab 1989; 69(3):684–688. doi:10.1210/jcem-69-3-684
- Koulouri O, Moran C, Halsall D, Chatterjee K, Gurnell M. Pitfalls in the measurement and interpretation of thyroid function tests. Best Pract Res Clin Endocrinol Metab 2013; 27(6):745–762. doi:10.1016/j.beem.2013.10.003
- Lechan RM, Fekete C. Role of thyroid hormone deiodination in the hypothalamus. Thyroid 2005; 15(8):883–897. doi:10.1089/thy.2005.15.883
- Chopra IJ, Hershman JM, Pardridge WM, Nicoloff JT. Thyroid function in nonthyroidal ilnesses. Ann Intern Med 1983; 98(6):946–957. doi:10.7326/0003-4819-98-6-946
- Chopra IJ, Solomon DH, Hepner HW, Mortenstein AA. Misleadingly low free thyroxine index and usefulness of reverse triiodothyronine measurement in nonthyroidal illnesses. Ann Intern Med 1979; 90(6):905–912. doi:10.7326/0003-4819-90-6-905
- Pontecorvi A, Robbins J. The plasma membrane and thyroid hormone entry into cells. Trends Endocrinol Metab 1989; 1(2):90–94. pmid:18411097
- Hennemann G, Krenning EP. Pitfalls in the interpretation of thyroid function tests in old age and non-thyroidal illness. Horm Res 1987; 26(1–4):100–104. doi:10.1159/000180688
- Baloch Z, Carayon P, Conte-Devolx B, et al; Guidelines Committee, National Academy of Clinical Biochemistry. Laboratory medicine practice guidelines. Laboratory support for the diagnosis and monitoring of thyroid disease. Thyroid 2003; 13(1):3–126. doi:10.1089/105072503321086962
- Lum S, Nicoloff JT, Spencer CA, Kaptein EM. Peripheral tissue mechanism for maintenance of serum triiodothyronine values in a thyroxine-deficient state in man. J Clin Invest 1984; 73(2):570–575. doi:10.1172/JCI111245
- Ortiga-Carvalho TM, Chiamolera MI, Pazos-Moura CC, Wondisford FE. Hypothalamus-pituitary-thyroid axis. Compr Physiol 2016; 6(3):1387–1428. doi:10.1002/cphy.c150027
- de Vries EM, Fliers E, Boelen A. The molecular basis of the non-thyroidal illness syndrome. J Endocrinol 2015; 225(3):R67–R81. doi:10.1530/JOE-15-0133
- Chopra IJ, Huang TS, Beredo A, Solomon DH, Teco GN, Mean JF. Evidence for an inhibitor of extrathyroidal conversion of thyroxine to 3, 5, 3'-triiodothyronine in sera of patients with nonthyroidal illnesses. J Clin Endocrinol Metab 1985; 60(4):666–672. doi:10.1210/jcem-60-4-666
- Peeters RP, Debaveye Y, Fliers E, Visser TJ. Changes within the thyroid axis during critical illness. Crit Care Clin 2006; 22(1):41–55. doi:10.1016/j.ccc.2005.08.006
- Spencer C, Eigen A, Shen D, et al. Specificity of sensitive assays of thyrotropin (TSH) used to screen for thyroid disease in hospitalized patients. Clin Chem 1987; 33(8):1391–1396. pmid:3301067
- Adler SM, Wartofsky L. The nonthyroidal illness syndrome. Endocrinol Metab Clin North Am 2007; 36(3):657–672. doi:10.1016/j.ecl.2007.04.007
- Persani L. Central hypothyroidism: pathogenic, diagnostic, and therapeutic challenges. J Clin Endocrinol Metab 2012; 97(9):3068–3078. doi:10.1210/jc.2012-1616
- Kidess AI, Caplan RH, Reynertson RH, Wickus GG, Goodnough DE. Transient corticotropin deficiency in critical illness. Mayo Clin Proc 1993; 68(5):435–441. doi:10.1016/s0025-6196(12)60188-8
- Lamberts SW, Bruining HA, De Jong FH. Corticosteroid therapy in severe illness. N Engl J Med 1997; 337(18):1285–1292. doi:10.1056/NEJM199710303371807
- Burmeister LA. Reverse T3 does not reliably differentiate hypothyroid sick syndrome from euthyroid sick syndrome. Thyroid 1995; 5(6):435–441. doi:10.1089/thy.1995.5.435
- Duick DS, Warren DW, Nicoloff JT, Otis CL, Croxson MS. Effect of single dose dexamethasone on the concentration of serum triiodothyronine in man. J Clin Endocrinol Metab 1974; 39(6):1151–1154. doi:10.1210/jcem-39-6-1151
- Gamstedt A, Järnerot G, Kågedal B. Dose related effects of betamethasone on iodothyronines and thyroid hormone-binding proteins in serum. Acta Endocrinol (Copenh) 1981; 96(4):484–490. doi:10.1530/acta.0.0960484
- Wartofsky L, Burman KD. Alterations in thyroid function in patients with systemic illness: the “euthyroid sick syndrome.” Endocr Rev 1982; 3(2):164–217. doi:10.1210/edrv-3-2-164
- Wilber JF, Utiger RD. The effect of glucocorticoids on thyrotropin secretion. J Clin Invest 1969; 48(11):2096–2103. doi:10.1172/JCI106176
- Nicoloff JT, Fisher DA, Appleman MD Jr. The role of glucocorticoids in the regulation of thyroid function in man. J Clin Invest 1970; 49(10):1922–1929. doi:10.1172/JCI106411
- Surks MI, Sievert R. Drugs and thyroid function. N Engl J Med 1995; 333(25):1688–1694. doi:10.1056/NEJM199512213332507
- Haugen BR. Drugs that suppress TSH or cause central hypothyroidism. Best Pract Res Clin Endocrinol Metab 2009; 23(6):793–800. doi:10.1016/j.beem.2009.08.003
- Sherman SI, Gopal J, Haugen BR, et al. Central hypothyroidism associated with retinoid X receptor–selective ligands. N Engl J Med 1999; 340(14):1075–1079. doi:10.1056/NEJM199904083401404
- Murchison LE, How J, Bewsher PD. Comparison of propranolol and metoprolol in the management of hyperthyroidism. Br J Clin Pharmacol 1979; 8(6):581–587. doi:10.1111/j.1365-2125.1979.tb01048.x
- Faber J, Friis T, Kirkegaard C, et al. Serum T4, T3 and reverse T3 during treatment with propranolol in hyperthyroidism, L-T4 treated myxedema and in normal man. Horm Metab Res 1979; 11(1):34–36. doi:10.1055/s-0028-1092678
- Kristensen BO, Weeke J. Propranolol-induced increments in total and free serum thyroxine in patients with essential hypertension. Clin Pharmacol Ther 1977; 22(6):864–867. doi:10.1002/cpt1977226864
- Murchison LE, Bewsher PD, Chesters MI, Ferrier WR. Comparison of propranolol and practolol in the management of hyperthyroidism. Br J Clin Pharmacol 1976; 3(2):273–277. doi:10.1111/j.1365-2125.1976.tb00603.x
- Lotti G, Delitala G, Devilla L, Alagna S, Masala A. Reduction of plasma triiodothyronine (T3) induced by propranolol. Clin Endocrinol 1977; 6(6):405–410. doi:10.1111/j.1365-2265.1977.tb03322.x
- Perrild H, Hansen JM, Skovsted L, Christensen LK. Different effects of propranolol, alprenolol, sotalol, atenolol and metoprolol on serum T3 and serum rT3 in hyperthyroidism. Clin Endocrinol (Oxf) 1983; 18(2):139–142. pmid:6133659
- Reeves RA, From GL, Paul W, Leenen FH. Nadolol, propranolol, and thyroid hormones: evidence for a membrane-stabilizing action of propranolol. Clin Pharmacol Ther 1985; 37(2):157–161. doi:10.1038/clpt.1985.28
- Walker N, Jung RT, Jennings G, James WP. The effect of a beta-receptor agonist (salbutamol) on peripheral thyroid metabolism in euthyroid subjects. Horm Metab Res 1981; 13(10):590–591. doi:10.1055/s-2007-1019346
- Melmed S, Geola FL, Reed AW, Pekary AE, Park J, Hershman JM. A comparison of methods for assessing thyroid function in nonthyroidal illness. J Clin Endocrinol Metab 1982; 54(2):300–306. doi:10.1210/jcem-54-2-300
- Docter R, Krenning E, De Jong M, Hennemann G. The sick euthyroid syndrome: changes in thyroid hormone serum parameters and hormone metabolism. Clin Endocrinol (Oxf) 1993; 39(5):499–518. pmid:8252737
- Mariotti S, Caturegli P, Piccolo P, Barbesino G, Pinchera A. Antithyroid peroxidase autoantibodies in thyroid diseases. J Clin Endocrinol Metab 1990; 71(3):661–669. doi:10.1210/jcem-71-3-661
- De Groot LJ. Non-thyroidal illness syndrome is a manifestation of hypothalamic-pituitary dysfunction, and in view of current evidence, should be treated with appropriate replacement therapies. Crit Care Clin 2006; 22(1):57–86. doi:10.1016/j.ccc.2005.10.001
- Jonklaas J, Bianco AC, Bauer AJ, et al; American Thyroid Association Task Force on Thyroid Hormone Replacement. Guidelines for the treatment of hypothyroidism: prepared by the American Thyroid Association Task Force on Thyroid Hormone Replacement. Thyroid 2014; 24(12):1670–1751. doi:10.1089/thy.2014.0028
- Kaptein EM, Beale E, Chan LS. Thyroid hormone therapy for obesity and nonthyroidal illnesses: a systematic review. J Clin Endocrinol Metab 2009; 94(10):3663–3675. doi:10.1210/jc.2009-0899
- Vidart J, Wajner SM, Leite RS, et al. N-acetylcysteine administration prevents nonthyroidal illness syndrome in patients with acute myocardial infarction: a randomized clinical trial. J Clin Endocrinol Metab 2014; 99(12):4537–4545. doi:10.1210/jc.2014-2192
- Van den Berghe G, Wouters P, Weekers F, et al. Reactivation of pituitary hormone release and metabolic improvement by infusion of growth hormone-releasing peptide and thyrotropin-releasing hormone in patients with protracted critical illness. J Clin Endocrinol Metab 1999; 84(4):1311–1323. doi:10.1210/jcem.84.4.5636
- Langouche L, Vander Perre S, Marques M, et al. Impact of early nutrient restriction during critical illness on the nonthyroidal illness syndrome and its relation with outcome: a randomized, controlled clinical study. J Clin Endocrinol Metab 2013; 98(3):1006–1013. doi:10.1210/jc.2012-2809
- Economidou F, Douka E, Tzanela M, Nanas S, Kotanidou A. Thyroid function during critical illness. Hormones (Athens) 2011; 10(2):117–124. doi:10.14310/horm.2002.1301
- Hamblin PS, Dyer SA, Mohr VS, et al. Relationship between thyrotropin and thyroxine changes during recovery from severe hypothyroxinemia of critical illness. J Clin Endocrinol Metab 1986; 62(4):717–722. doi:10.1210/jcem-62-4-717
- Iglesias P, Diez JJ. Thyroid dysfunction and kidney disease. Eur J Endocrinol 2009; 160(4):503–515. doi:10.1530/EJE-08-0837
- Spencer CA. Clinical utility and cost-effectiveness of sensitive thyrotropin assays in ambulatory and hospitalized patients. Mayo Clin Proc 1988; 63(12):1214–1222. doi:10.1016/s0025-6196(12)65408-1
- Attia J, Margetts P, Guyatt G. Diagnosis of thyroid disease in hospitalized patients: a systematic review. Arch Intern Med 1999; 159(7):658–665. pmid:10218744
- Barbesino G, Tomer Y. Clinical review: clinical utility of TSH receptor antibodies. J Clin Endocrinol Metab 2013; 98(6):2247–2255. doi:10.1210/jc.2012-4309
- Lamb EJ, Martin J. Thyroid function tests: often justified in the acutely ill. Ann Clin Biochem 2000; 37(pt 2):158–164. doi:10.1258/0004563001899159
- Spencer CA, LoPresti JS, Patel A, et al. Applications of a new chemiluminometric thyrotropin assay to subnormal measurement. J Clin Endocrinol Metab 1990; 70(2):453–460. doi:10.1210/jcem-70-2-453
- Ross DS, Ardisson LJ, Meskell MJ. Measurement of thyrotropin in clinical and subclinical hyperthyroidism using a new chemiluminescent assay. J Clin Endocrinol Metab 1989; 69(3):684–688. doi:10.1210/jcem-69-3-684
- Koulouri O, Moran C, Halsall D, Chatterjee K, Gurnell M. Pitfalls in the measurement and interpretation of thyroid function tests. Best Pract Res Clin Endocrinol Metab 2013; 27(6):745–762. doi:10.1016/j.beem.2013.10.003
- Lechan RM, Fekete C. Role of thyroid hormone deiodination in the hypothalamus. Thyroid 2005; 15(8):883–897. doi:10.1089/thy.2005.15.883
- Chopra IJ, Hershman JM, Pardridge WM, Nicoloff JT. Thyroid function in nonthyroidal ilnesses. Ann Intern Med 1983; 98(6):946–957. doi:10.7326/0003-4819-98-6-946
- Chopra IJ, Solomon DH, Hepner HW, Mortenstein AA. Misleadingly low free thyroxine index and usefulness of reverse triiodothyronine measurement in nonthyroidal illnesses. Ann Intern Med 1979; 90(6):905–912. doi:10.7326/0003-4819-90-6-905
- Pontecorvi A, Robbins J. The plasma membrane and thyroid hormone entry into cells. Trends Endocrinol Metab 1989; 1(2):90–94. pmid:18411097
- Hennemann G, Krenning EP. Pitfalls in the interpretation of thyroid function tests in old age and non-thyroidal illness. Horm Res 1987; 26(1–4):100–104. doi:10.1159/000180688
- Baloch Z, Carayon P, Conte-Devolx B, et al; Guidelines Committee, National Academy of Clinical Biochemistry. Laboratory medicine practice guidelines. Laboratory support for the diagnosis and monitoring of thyroid disease. Thyroid 2003; 13(1):3–126. doi:10.1089/105072503321086962
- Lum S, Nicoloff JT, Spencer CA, Kaptein EM. Peripheral tissue mechanism for maintenance of serum triiodothyronine values in a thyroxine-deficient state in man. J Clin Invest 1984; 73(2):570–575. doi:10.1172/JCI111245
- Ortiga-Carvalho TM, Chiamolera MI, Pazos-Moura CC, Wondisford FE. Hypothalamus-pituitary-thyroid axis. Compr Physiol 2016; 6(3):1387–1428. doi:10.1002/cphy.c150027
- de Vries EM, Fliers E, Boelen A. The molecular basis of the non-thyroidal illness syndrome. J Endocrinol 2015; 225(3):R67–R81. doi:10.1530/JOE-15-0133
- Chopra IJ, Huang TS, Beredo A, Solomon DH, Teco GN, Mean JF. Evidence for an inhibitor of extrathyroidal conversion of thyroxine to 3, 5, 3'-triiodothyronine in sera of patients with nonthyroidal illnesses. J Clin Endocrinol Metab 1985; 60(4):666–672. doi:10.1210/jcem-60-4-666
- Peeters RP, Debaveye Y, Fliers E, Visser TJ. Changes within the thyroid axis during critical illness. Crit Care Clin 2006; 22(1):41–55. doi:10.1016/j.ccc.2005.08.006
- Spencer C, Eigen A, Shen D, et al. Specificity of sensitive assays of thyrotropin (TSH) used to screen for thyroid disease in hospitalized patients. Clin Chem 1987; 33(8):1391–1396. pmid:3301067
- Adler SM, Wartofsky L. The nonthyroidal illness syndrome. Endocrinol Metab Clin North Am 2007; 36(3):657–672. doi:10.1016/j.ecl.2007.04.007
- Persani L. Central hypothyroidism: pathogenic, diagnostic, and therapeutic challenges. J Clin Endocrinol Metab 2012; 97(9):3068–3078. doi:10.1210/jc.2012-1616
- Kidess AI, Caplan RH, Reynertson RH, Wickus GG, Goodnough DE. Transient corticotropin deficiency in critical illness. Mayo Clin Proc 1993; 68(5):435–441. doi:10.1016/s0025-6196(12)60188-8
- Lamberts SW, Bruining HA, De Jong FH. Corticosteroid therapy in severe illness. N Engl J Med 1997; 337(18):1285–1292. doi:10.1056/NEJM199710303371807
- Burmeister LA. Reverse T3 does not reliably differentiate hypothyroid sick syndrome from euthyroid sick syndrome. Thyroid 1995; 5(6):435–441. doi:10.1089/thy.1995.5.435
- Duick DS, Warren DW, Nicoloff JT, Otis CL, Croxson MS. Effect of single dose dexamethasone on the concentration of serum triiodothyronine in man. J Clin Endocrinol Metab 1974; 39(6):1151–1154. doi:10.1210/jcem-39-6-1151
- Gamstedt A, Järnerot G, Kågedal B. Dose related effects of betamethasone on iodothyronines and thyroid hormone-binding proteins in serum. Acta Endocrinol (Copenh) 1981; 96(4):484–490. doi:10.1530/acta.0.0960484
- Wartofsky L, Burman KD. Alterations in thyroid function in patients with systemic illness: the “euthyroid sick syndrome.” Endocr Rev 1982; 3(2):164–217. doi:10.1210/edrv-3-2-164
- Wilber JF, Utiger RD. The effect of glucocorticoids on thyrotropin secretion. J Clin Invest 1969; 48(11):2096–2103. doi:10.1172/JCI106176
- Nicoloff JT, Fisher DA, Appleman MD Jr. The role of glucocorticoids in the regulation of thyroid function in man. J Clin Invest 1970; 49(10):1922–1929. doi:10.1172/JCI106411
- Surks MI, Sievert R. Drugs and thyroid function. N Engl J Med 1995; 333(25):1688–1694. doi:10.1056/NEJM199512213332507
- Haugen BR. Drugs that suppress TSH or cause central hypothyroidism. Best Pract Res Clin Endocrinol Metab 2009; 23(6):793–800. doi:10.1016/j.beem.2009.08.003
- Sherman SI, Gopal J, Haugen BR, et al. Central hypothyroidism associated with retinoid X receptor–selective ligands. N Engl J Med 1999; 340(14):1075–1079. doi:10.1056/NEJM199904083401404
- Murchison LE, How J, Bewsher PD. Comparison of propranolol and metoprolol in the management of hyperthyroidism. Br J Clin Pharmacol 1979; 8(6):581–587. doi:10.1111/j.1365-2125.1979.tb01048.x
- Faber J, Friis T, Kirkegaard C, et al. Serum T4, T3 and reverse T3 during treatment with propranolol in hyperthyroidism, L-T4 treated myxedema and in normal man. Horm Metab Res 1979; 11(1):34–36. doi:10.1055/s-0028-1092678
- Kristensen BO, Weeke J. Propranolol-induced increments in total and free serum thyroxine in patients with essential hypertension. Clin Pharmacol Ther 1977; 22(6):864–867. doi:10.1002/cpt1977226864
- Murchison LE, Bewsher PD, Chesters MI, Ferrier WR. Comparison of propranolol and practolol in the management of hyperthyroidism. Br J Clin Pharmacol 1976; 3(2):273–277. doi:10.1111/j.1365-2125.1976.tb00603.x
- Lotti G, Delitala G, Devilla L, Alagna S, Masala A. Reduction of plasma triiodothyronine (T3) induced by propranolol. Clin Endocrinol 1977; 6(6):405–410. doi:10.1111/j.1365-2265.1977.tb03322.x
- Perrild H, Hansen JM, Skovsted L, Christensen LK. Different effects of propranolol, alprenolol, sotalol, atenolol and metoprolol on serum T3 and serum rT3 in hyperthyroidism. Clin Endocrinol (Oxf) 1983; 18(2):139–142. pmid:6133659
- Reeves RA, From GL, Paul W, Leenen FH. Nadolol, propranolol, and thyroid hormones: evidence for a membrane-stabilizing action of propranolol. Clin Pharmacol Ther 1985; 37(2):157–161. doi:10.1038/clpt.1985.28
- Walker N, Jung RT, Jennings G, James WP. The effect of a beta-receptor agonist (salbutamol) on peripheral thyroid metabolism in euthyroid subjects. Horm Metab Res 1981; 13(10):590–591. doi:10.1055/s-2007-1019346
- Melmed S, Geola FL, Reed AW, Pekary AE, Park J, Hershman JM. A comparison of methods for assessing thyroid function in nonthyroidal illness. J Clin Endocrinol Metab 1982; 54(2):300–306. doi:10.1210/jcem-54-2-300
- Docter R, Krenning E, De Jong M, Hennemann G. The sick euthyroid syndrome: changes in thyroid hormone serum parameters and hormone metabolism. Clin Endocrinol (Oxf) 1993; 39(5):499–518. pmid:8252737
- Mariotti S, Caturegli P, Piccolo P, Barbesino G, Pinchera A. Antithyroid peroxidase autoantibodies in thyroid diseases. J Clin Endocrinol Metab 1990; 71(3):661–669. doi:10.1210/jcem-71-3-661
- De Groot LJ. Non-thyroidal illness syndrome is a manifestation of hypothalamic-pituitary dysfunction, and in view of current evidence, should be treated with appropriate replacement therapies. Crit Care Clin 2006; 22(1):57–86. doi:10.1016/j.ccc.2005.10.001
- Jonklaas J, Bianco AC, Bauer AJ, et al; American Thyroid Association Task Force on Thyroid Hormone Replacement. Guidelines for the treatment of hypothyroidism: prepared by the American Thyroid Association Task Force on Thyroid Hormone Replacement. Thyroid 2014; 24(12):1670–1751. doi:10.1089/thy.2014.0028
- Kaptein EM, Beale E, Chan LS. Thyroid hormone therapy for obesity and nonthyroidal illnesses: a systematic review. J Clin Endocrinol Metab 2009; 94(10):3663–3675. doi:10.1210/jc.2009-0899
- Vidart J, Wajner SM, Leite RS, et al. N-acetylcysteine administration prevents nonthyroidal illness syndrome in patients with acute myocardial infarction: a randomized clinical trial. J Clin Endocrinol Metab 2014; 99(12):4537–4545. doi:10.1210/jc.2014-2192
- Van den Berghe G, Wouters P, Weekers F, et al. Reactivation of pituitary hormone release and metabolic improvement by infusion of growth hormone-releasing peptide and thyrotropin-releasing hormone in patients with protracted critical illness. J Clin Endocrinol Metab 1999; 84(4):1311–1323. doi:10.1210/jcem.84.4.5636
- Langouche L, Vander Perre S, Marques M, et al. Impact of early nutrient restriction during critical illness on the nonthyroidal illness syndrome and its relation with outcome: a randomized, controlled clinical study. J Clin Endocrinol Metab 2013; 98(3):1006–1013. doi:10.1210/jc.2012-2809
- Economidou F, Douka E, Tzanela M, Nanas S, Kotanidou A. Thyroid function during critical illness. Hormones (Athens) 2011; 10(2):117–124. doi:10.14310/horm.2002.1301
- Hamblin PS, Dyer SA, Mohr VS, et al. Relationship between thyrotropin and thyroxine changes during recovery from severe hypothyroxinemia of critical illness. J Clin Endocrinol Metab 1986; 62(4):717–722. doi:10.1210/jcem-62-4-717
- Iglesias P, Diez JJ. Thyroid dysfunction and kidney disease. Eur J Endocrinol 2009; 160(4):503–515. doi:10.1530/EJE-08-0837
- Spencer CA. Clinical utility and cost-effectiveness of sensitive thyrotropin assays in ambulatory and hospitalized patients. Mayo Clin Proc 1988; 63(12):1214–1222. doi:10.1016/s0025-6196(12)65408-1
- Attia J, Margetts P, Guyatt G. Diagnosis of thyroid disease in hospitalized patients: a systematic review. Arch Intern Med 1999; 159(7):658–665. pmid:10218744
- Barbesino G, Tomer Y. Clinical review: clinical utility of TSH receptor antibodies. J Clin Endocrinol Metab 2013; 98(6):2247–2255. doi:10.1210/jc.2012-4309
An obese 48-year-old man with progressive fatigue and decreased libido
A 48-year-old man presents to his primary care physician because of progressively decreasing energy and gradual decline in both libido and erectile function for the past 18 months. He has noticed decreased morning erections as well. He rates his libido at 3 to 4 on a scale of 10 for the past 6 months. He also reports poor motivation, depressed mood, impaired concentration, and sleep disturbances. He reports no hair loss, headache, or dizziness, and no decrease in shaving frequency. Review of his systems is otherwise unremarkable.
He has had dyslipidemia for 3 years and is not known to have hypertension or diabetes. His medications include atorvastatin, vitamin E, and multivitamins.
He is married with 3 children and does not wish to have more. He works as a software engineer and leads a sedentary lifestyle. He is a nonsmoker and occasionally drinks alcohol on the weekends.
On physical examination, he is alert and oriented and appears well. His height is 5 feet 10 inches (178 cm), weight 230 lb (104 kg), and body mass index (BMI) 32.8 kg/m2. His blood pressure is 115/83 mm Hg and pulse rate is 82 beats per minute and regular. Findings on cardiovascular and pulmonary examination are normal. He has large fatty breasts but without palpable glandular tissue.
Genitourinary examination reveals normal hair distribution, a normal-sized penis, and slightly soft testes with testicular volume of 18–20 mL bilaterally.
His primary care physician suspects that he has low testosterone and orders some basic laboratory tests; the results are normal except for a low total testosterone level (Table 1).
FURTHER TESTING
1. Which of the following tests should his physician order next?
- Repeat total testosterone measurement
- Free testosterone measurement by commercial assay
- Calculated free testosterone
- Bioavailable testosterone measurement
- Serum inhibin B measurement
This patient presents with several nonspecific symptoms. But collectively they suggest testosterone deficiency (hypogonadism).
Together, erectile dysfunction, low libido, and decreased morning erections strongly suggest hypogonadism.2 Loss of body hair and decreased shaving frequency are specific symptoms of hypogonadism; however, they require years to develop.3 Gynecomastia can also occur due to loss of the inhibitory action of testosterone on breast growth and a relative increase in estradiol. This occurs more in primary hypogonadism, due to the increase in luteinizing hormone (LH), which stimulates the remaining Leydig cells to secrete estradiol rather than testosterone.4
To diagnose hypogonadism in men and to start treatment for it, current guidelines recommend that the patient should have clinical features as well as laboratory evidence of low testosterone.5,6
Measuring testosterone: Total, free, bound, and bioavailable
Testosterone, a steroid hormone, circulates in the serum either as free testosterone or bound to several plasma proteins, mainly sex-hormone binding globulin (SHBG) and albumin.
Total testosterone includes both the free and bound fractions, whereas bioavailable testosterone includes both free and the portion bound to albumin, which has low affinity and can dissociate and be used at the tissue level.11
Low levels of total testosterone do not necessarily reflect a hypogonadal state, as a man with altered SHBG levels or binding capabilities can have low total but normal free testosterone levels and no manifestations.12 Several conditions can alter the levels of SHBG, including obesity, diabetes, aging, thyroid dysfunction, and others.5,13
Because our patient is obese, his total testosterone level is not a reliable indicator of hypogonadism, and repeating its measurement will not add diagnostic value.
Therefore, an alternative measurement should be used to accurately reflect the testosterone levels. From a physiologic point of view, bioavailable testosterone is the active form of testosterone and is the most accurate to be measured in a patient with hypogonadism. Nevertheless, because of technical difficulties in its measurement and lack of evidence correlating bioavailable testosterone with the clinical picture of hypogonadism, it is recommended that the level of free testosterone be used.5
The gold standard for direct measurement of serum free testosterone is equilibrium dialysis, but this is expensive and time-consuming.14 Commercial assays for free testosterone exist but have been deemed unreliable.14,15 It is recommended that free testosterone be measured by equilibrium dialysis or calculated using equations based on total testosterone, SHBG, and albumin levels.5 These equations are reliable and give results very close to the values obtained by equilibrium dialysis.15 Therefore, in our patient, it would be suitable to calculate the free testosterone level next.
Serum levels of free testosterone vary according to several factors. Diurnal variation of testosterone has been established: levels are highest in the morning and decline throughout the day.16 Food decreases testosterone levels.17 In addition, there is considerable day-to-day variation.18 Therefore, at least 2 readings of fasting morning testosterone on 2 separate days are recommended for the diagnosis of hypogonadism.5
Inhibin B is a hormone produced by Sertoli cells in the testes in response to follicle-stimulating hormone (FSH) stimulation. In turn, it acts as negative feedback, together with testosterone, to inhibit FSH release from the pituitary. Inhibin B has been shown to reflect spermatogenesis in the testes and therefore fertility.19 Inhibin B levels were found to be low in patients with central hypogonadism, due to less FSH release; however, they did not correlate with testosterone levels.20
CASE RESUMED: CHARACTERIZING HIS HYPOGONADISM
The patient’s physician orders morning fasting total testosterone, SHBG, and albumin testing and calculates the free testosterone level, which yields a value of 3 ng/dL (reference range 4.5–17). This is confirmed by a repeat measurement, which yields a value of 2.9 ng/dL. Laboratory test results combined with his clinical presentation are consistent with hypogonadism.
2. What is the most appropriate next step?
- Measurement of serum LH and FSH
- Measurement of serum prolactin
- Scrotal ultrasonography
- Gonadotropin-releasing hormone (GnRH) stimulation test
- Semen analysis
After hypogonadism is diagnosed, it is important to distinguish if it is primary or central. This is achieved by measuring serum LH and FSH.5 All biotin supplements should be stopped at least 72 hours before measuring LH and FSH, as biotin can interfere with the assays, yielding false values.21
Secretion of FSH and LH from the anterior pituitary is under the influence of pulsatile release of GnRH from the hypothalamus. LH acts on Leydig cells in the testes to produce testosterone, whereas FSH acts on Sertoli cells, together with testosterone, to bring about spermatogenesis in the seminiferous tubules. Testosterone acts centrally as negative feedback to decrease the release of LH and FSH.
Primary hypogonadism occurs due to testicular failure, ie, the testes themselves fail to produce testosterone, leading to hypogonadism. The decrease in testosterone levels, together with inhibin B if Sertoli cells are damaged, lead to loss of negative feedback on the hypothalamus and pituitary, and therefore increased levels of LH and FSH. This is termed hypergonadotropic hypogonadism. Testicular failure may also result in impaired spermatogenesis and infertility due to destruction of testicular structures, in which case fertility cannot be restored.
Central hypogonadism occurs when the pituitary fails to produce LH and FSH (secondary hypogonadism) or when the hypothalamus fails to produce GnRH and subsequently the lack of secretion of LH and FSH from the pituitary (tertiary hypogonadism). The lack of LH will result in no stimulation of Leydig cells to produce testosterone, and therefore its deficiency. Serum hormone levels in central hypogonadism will reveal low testosterone, with either low or inappropriately normal gonadotropins (LH and FSH). This is termed hypogonadotropic hypogonadism. The lack of FSH, together with testosterone deficiency will also result in decreased spermatogenesis and therefore infertility. Testicular structures are preserved, however, and fertility can be restored with appropriate therapy, as discussed below.
Prolactin should be measured only if the patient has central hypogonadism. Its measurement is not warranted at this point in the patient’s workup. The implications of prolactin and its relationship to hypogonadism will be discussed later.
Although, this stepwise approach is not convenient for many patients, some physicians follow it because it is cost-effective, especially in those who are not insured. However, other physicians order FSH, LH, and sometimes prolactin with the confirmatory low testosterone measurement. Laboratories can also be instructed to wait to measure the pituitary hormones and to do so only if low testosterone is confirmed.
Varicocele, a possible cause of male infertility, can also impair Leydig cell function and cause low testosterone. In fact, surgical repair of varicocele has been demonstrated to increase serum testosterone.22 Scrotal ultrasonography is used to diagnose varicocele, but this also should be ordered at a later stage in the workup if primary hypogonadism is diagnosed.
The GnRH stimulation test is important for the diagnosis and evaluation of precocious or delayed puberty in children. In boys with delayed puberty, a poorer response to GnRH stimulation indicates central hypogonadism rather than constitutional delay.23 It has no role in the evaluation of postpubertal or adult-onset hypogonadism.
Semen analysis is important to evaluate fertility if the patient is interested in further procreation.5 Low testosterone levels may result in impaired spermatogenesis and therefore infertility. On the other hand, treatment with exogenous testosterone will also result in infertility, by feedback inhibition of LH and FSH and therefore inhibition of spermatogenesis. If the patient wishes to preserve fertility, treatment options other than testosterone should be considered; examples include clomiphene citrate, human menopausal gonadotropin, and human chorionic gonadotropin.23,24
Our patient has no desire to expand his family; therefore, a semen analysis and attempts to preserve spermatogenesis are not indicated.
CASE RESUMED: SEARCHING FOR CAUSES
His physician orders testing of serum LH and FSH, yielding the following values:
- LH 1.6 mIU/mL (reference range 1.8–12)
- FSH 1.9 mIU/mL (reference range 1.5–12.5).
The diagnosis of central hypogonadism is established.
3. Which investigation is the least appropriate in the further evaluation of this patient?
- Serum prolactin measurement
- Serum ferritin measurement
- Pituitary magnetic resonance imaging (MRI)
- Chromosomal karyotyping
The diagnosis of central hypogonadism warrants evaluation for possible causes. These are summarized in Table 4.
Serum free thyroxine and morning cortisol
Since this patient’s LH and FSH values are abnormal, it is important to evaluate the status of other anterior pituitary hormones. In patients with pituitary abnormalities, serum free T4 is a more reliable test for assessing thyroid function than thyroid-stimulating hormone (TSH), because of loss of the negative feedback of thyroid hormones on the diseased pituitary. In contrast, serum TSH is considered the best single thyroid test to assess primary thyroid dysfunction.
Other measurements include prolactin and morning cortisol (reflecting adrenocorticotropic hormone status).
Prolactin measurement
Prolactin measurement is important to evaluate for hyperprolactinemia, as this will lead to hypogonadism by inhibition of GnRH secretion.25 Different pathologic, pharmacologic, and physiologic conditions can result in hyperprolactinemia, including prolactinomas, other pituitary and hypothalamic lesions, primary hypothyroidism, and medications such as antipsychotics.25 Dopamine agonists are the mainstay treatment for hyperprolactinemia.
Ferritin measurement
Ferritin measurement is indicated to diagnose iron overload conditions such as hemochromatosis, which can result in primary hypogonadism via testicular damage or in secondary hypogonadism via pituitary damage.26
Pituitary MRI with contrast
Pituitary MRI with contrast is used to diagnose structural lesions of the pituitary or hypothalamus. This diagnostic modality is indicated for patients with pituitary dysfunction, including central hypogonadism, manifestations of a mass effect (headache, visual field defects), persistent hyperprolactinemia, and panhypopituitarism, among others. To improve the diagnostic yield of pituitary MRI, the Endocrine Society guidelines recommend it for men with serum total testosterone levels below 150 ng/dL.5 However, some clinicians have a lower threshold for ordering pituitary MRI for patients with central hypogonadism. Physician judgment and expertise should be exercised and the decision made on an individual basis.
Chromosomal karyotyping
Chromosomal karyotyping is not indicated in our patient. It is reserved for those with primary hypogonadism to diagnose Klinefelter syndrome, which has a karyotype of 47,XXY.
CASE RESUMED: MOSH SYNDROME
Our patient’s prolactin, free T4, morning cortisol, and ferritin levels are measured, yielding normal values. No abnormalities are seen on pituitary MRI. A clinical reevaluation is conducted, revealing no history of head trauma or head and neck radiation. The lack of an obvious cause in our patient’s clinical presentation and workup, together with his obesity (BMI 32.8 kg/m2) supports the diagnosis of obesity as the cause of his hypogonadism.
Obesity can be a cause of secondary hypogonadism, which has led to the term “MOSH” (male obesity-associated secondary hypogonadism) syndrome. In fact, a cross-sectional study has demonstrated that 40% of nondiabetic obese (BMI ≥ 30 kg/m2) men over age 45 have low serum free testosterone levels, compared with 26% for lean (BMI < 25 kg/m2) men.27 Moreover, obesity has been found to be a strong predictor of testosterone replacement therapy.28 Other studies have also found an inverse relationship between BMI and testosterone levels.29
Several mechanisms interact in the pathogenesis of MOSH syndrome. Adipose tissue possesses aromatase activity, which converts androgens into estrogens.30 Peripheral estrogen production can in turn exert feedback inhibition on pituitary gonadotropin secretion.31 In obese men, increased adipose tissue leads to increased aromatase activity and more estrogen, so more feedback inhibition on the pituitary and subsequently secondary hypogonadism.
Leptin, a hormone produced by adipocytes, is also increased in obesity, and was found to be inversely correlated with serum testosterone.32 Studies have demonstrated that leptin has an inhibitory effect on the enzymatic pathway that synthesizes testosterone in Leydig cells.33
Proinflammatory cytokines have also been implicated, as central obesity is associated with an increase in these cytokines, which in turn act negatively on the hypothalamus and impair GnRH release leading to lower testosterone.34,35
Treating obesity-related hypogonadism
In a pilot study,36 lifestyle attempts to reduce obesity were shown to improve hormonal levels. Bariatric surgery has also been demonstrated to be successful.37
Clomiphene citrate, a selective estrogen receptor modulator, increases endogenous testosterone secretion by inhibiting the negative feedback of estrogen on the hypothalamus and pituitary and thus increasing LH and FSH. It also preserves endogenous testosterone production, since it does not suppress the hypothalamic-pituitary-testicular axis.38 This made clomiphene citrate a potential treatment for men with central hypogonadism including those with MOSH.39
Nevertheless, there are no randomized trials to prove its safety and efficacy in the management of central hypogonadism.5 Regarding its use in men wishing to preserve fertility, most studies did not show improvement. However, a meta-analysis demonstrated statistically significant increased pregnancy rates in partners of men with idiopathic infertility if the men used 50 mg of clomiphene citrate daily.40
Testosterone deficiency can be a marker of metabolic syndrome, which needs to be managed more urgently than hypogonadism. A cross-sectional study found not only an association between metabolic syndrome and low serum testosterone, but also with each individual component of metabolic syndrome on its own, all of which need to be addressed.10
CASE CONTINUED: BEGINNING TREATMENT
The physician counsels the patient regarding the implications, potential adverse outcomes, and available treatments for his obesity, including lifestyle modification and bariatric surgery. The patient declines surgery and wishes to adopt a weight-reducing diet and exercise program, for which he is referred to a dietitian.
In addition, in view of the patient’s clinically and biochemically proven hypogonadism, his physician offers testosterone replacement therapy. He orders a serum prostate-specific antigen (PSA) level, which is 1.3 ng/dL (reference range < 4 ng/dL). The patient is prescribed 5 g of 1% testosterone gel daily.
TESTOSTERONE REPLACEMENT THERAPY
4. Which is the most common adverse effect of testosterone replacement therapy?
- Cardiovascular events
- Erythrocytosis
- Prostate cancer
- Infertility
- Obstructive sleep apnea
Clinicians should be very cautious in initiating testosterone replacement therapy in any patient with an unstable medical condition.
There are several formulations of testosterone replacement therapy, including intramuscular injections, transdermal gels or patches, buccal tablets, an intranasal gel, and oral tablets. Of note, there are 2 different forms of oral testosterone preparations: testosterone undecanoate and 17-alpha alkylated testosterone. The former is unavailable in the United States and the latter is not recommended for use due to its proven hepatic toxicity.41
Testosterone and erythrocytosis
Meta-analyses have concluded that the most frequent adverse event of testosterone replacement therapy is a significant rise in hematocrit.42 This rise was found to be dose-dependent and was more marked in older men.43 Although all preparations can cause erythrocytosis, parenteral forms have been observed to raise it the most, particularly short-term injectables.44,45
The mechanism behind this increase is attributed to increased erythropoietin levels and improved usage of iron for red blood cell synthesis.46 In fact, testosterone replacement therapy has been shown to improve hemoglobin levels in patients with anemia.47 On the other hand, increasing hematocrit levels may lead to thrombotic and vasoocclusive events.44
It is strongly recommended that baseline hematocrit levels be measured before initiating testosterone replacement therapy.5,6 The hematocrit level should also be monitored 3 to 6 months into treatment and yearly thereafter while on testosterone.5Figure 1 summarizes the appropriate steps to undertake regarding hematocrit levels, according to the American Urological Association.6
Testosterone and prostate cancer
The relationship between testosterone treatment and prostate cancer has long been studied. Historically, testosterone replacement therapy was believed to increase the risk of prostate cancer; however, recent studies and meta-analyses have shown that this is not the case.42,48 Nevertheless, clinical guidelines still recommend prostate monitoring for men on testosterone replacement therapy.5,6
Testosterone and cardiovascular risk
The evidence regarding this issue has been contradictory and inconsistent. Meta-analyses have demonstrated that low testosterone is associated with higher risk of major adverse cardiovascular events.50 These studies argue for the use of testosterone replacement therapy in hypogonadal men to decrease the risk. However, other studies and meta-analyses have found that testosterone replacement therapy is associated with increased cardiovascular risk and have concluded that major adverse cardiac events are in fact a risk of testosterone replacement therapy.51
Current recommendations advocate against the use of testosterone replacement therapy in men with uncontrolled heart failure or with cardiovascular events in the past 3 to 6 months.5,6 Cardiovascular risk factors should be addressed and corrected, and patients should be educated on cardiovascular symptoms and the need to report them if they occur.
Testosterone and infertility
As described earlier, testosterone replacement therapy increases negative feedback on the pituitary and decreases LH and FSH production, leading to less spermatogenesis. Other treatment options should be sought for hypogonadal men wishing to preserve fertility.
Other adverse effects
Other adverse effects of testosterone replacement therapy include acne, oily skin, obstructive sleep apnea, gynecomastia, and balding.
Given all the adverse events that can be associated with testosterone replacement therapy, the risks and benefits of treating hypogonadism in each patient should be taken into consideration, and an individualized approach is required.
CASE RESUMED: FOLLOW-UP
The patient presents 3 months later for follow-up. He reports significant improvement in his presenting symptoms including energy, libido, and erectile function. He also reports some improvement in his mood and concentration. He has lost 12 lb (5.4 kg) and is still trying to improve his diet and exercise program. He is compliant with his testosterone gel therapy.
His serum calculated free testosterone level is 7.8 ng/dL (4.5–17), and his hematocrit is 46%. The patient is instructed to continue his treatment and to return after 9 months for further follow-up.
TAKE-HOME POINTS
- Men with hypogonadism usually present with nonspecific manifestations, so clinicians should keep a high index of suspicion.
- Both clinical and biochemical evidence of hypogonadism should be present to diagnose and start treatment for it.
- Low levels of serum total testosterone do not necessarily reflect hypogonadism.
- The hormonal profile of central hypogonadism reveals low serum testosterone with low or inappropriately normal serum LH and FSH levels.
Obesity can cause central hypogonadism and should be suspected after pituitary and other systemic causes are excluded.
- Araujo AB, Esche GR, Kupelian V, et al. Prevalence of symptomatic androgen deficiency in men. J Clin Endocrinol Metab 2007; 92(11):4241–4247. doi:10.1210/jc.2007-1245
- Wu FCW, Tajar A, Beynon JM, et al; EMAS Group. Identification of late-onset hypogonadism in middle-aged and elderly men. N Engl J Med 2010; 363(2):123–135. doi:10.1056/NEJMoa0911101
- Arver S, Lehtihet M. Current guidelines for the diagnosis of testosterone deficiency. Front Horm Res 2009; 37:5–20. doi:10.1159/000175839
- Narula HS, Carlson HE. Gynaecomastia—pathophysiology, diagnosis and treatment. Nat Rev Endocrinol 2014; 10(11):684–698. doi:10.1038/nrendo.2014.139
- Bhasin S, Brito JP, Cunningham GR, et al. Testosterone therapy in men with hypogonadism: an Endocrine Society clinical practice guideline. J Clin Endocrinol Metab 2018; 103(5):1715–1744. doi:10.1210/jc.2018-00229
- Mulhall JP, Trost LW, Brannigan RE, et al. Evaluation and management of testosterone deficiency: AUA guideline. J Urol 2018; 200(2):423–432. doi:10.1016/j.juro.2018.03.115
- Balasubramanian V, Naing S. Hypogonadism in chronic obstructive pulmonary disease: incidence and effects. Curr Opin Pulm Med 2012; 18(2):112–117. doi:10.1097/MCP.0b013e32834feb37
- Atlantis E, Fahey P, Cochrane B, Wittert G, Smith S. Endogenous testosterone level and testosterone supplementation therapy in chronic obstructive pulmonary disease (COPD): a systematic review and meta-analysis. BMJ Open 2013; 3(8)pii:e003127. doi:10.1136/bmjopen-2013-003127
- Bawor M, Bami H, Dennis BB, et al. Testosterone suppression in opioid users: a systematic review and meta-analysis. Drug Alcohol Depend 2015; 149:1–9. doi:10.1016/j.drugalcdep.2015.01.038
- Tan WS, Ng CJ, Khoo EM, Low WY, Tan HM. The triad of erectile dysfunction, testosterone deficiency syndrome and metabolic syndrome: findings from a multi-ethnic Asian men study (The Subang Men's Health Study). Aging Male 2011; 14(4):231–236. doi:10.3109/13685538.2011.597463
- Goldman AL, Bhasin S, Wu FCW, Krishna M, Matsumoto AM, Jasuja R. A reappraisal of testosterone’s binding in circulation: physiological and clinical implications. Endocr Rev 2017; 38(4):302–324. doi:10.1210/er.2017-00025
- Antonio L, Wu FC, O’Neill TW, et al; European Male Ageing Study Study Group. Low free testosterone is associated with hypogonadal signs and symptoms in men with normal total testosterone. J Clin Endocrinol Metab 2016; 101(7):2647–2657. doi:10.1210/jc.2015-4106
- Liu F, Shen X, Wang R, et al. Association of central obesity with sex hormone binding globulin: a cross-sectional study of 1166 Chinese men. Open Med (Wars) 2018; 13:196–202. doi:10.1515/med-2018-0030
- Vermeulen A, Verdonck L, Kaufman JM. A critical evaluation of simple methods for the estimation of free testosterone in serum. J Clin Endocrinol Metab 1999; 84(10):3666–3672. doi:10.1210/jcem.84.10.6079
- Halmenschlager G, Rhoden EL, Riedner CE. Calculated free testosterone and radioimmunoassay free testosterone as a predictor of subnormal levels of total testosterone. Int Urol Nephrol 2012; 44(3):673–681. doi:10.1007/s11255-011-0066-z
- Brambilla DJ, Matsumoto AM, Araujo AB, McKinlay JB. The effect of diurnal variation on clinical measurement of serum testosterone and other sex hormone levels in men. J Clin Endocrinol Metab 2009; 94(3):907–913. doi:10.1210/jc.2008-1902
- Lehtihet M, Arver S, Bartuseviciene I, Pousette Å. S-testosterone decrease after a mixed meal in healthy men independent of SHBG and gonadotrophin levels. Andrologia 2012; 44(6):405–410. doi:10.1111/j.1439-0272.2012.01296.x
- Brambilla DJ, O’Donnell AB, Matsumoto AM, McKinlay JB. Intraindividual variation in levels of serum testosterone and other reproductive and adrenal hormones in men. Clin Endocrinol (Oxf) 2007; 67(6):853–862. doi:10.1111/j.1365-2265.2007.02976.x
- Manzoor SM, Sattar A, Hashim R, et al. Serum inhibin B as a diagnostic marker of male infertility. J Ayub Med Coll Abbottabad 2012; 24(3–4):113–116. pmid:24669628
- Kolb BA, Stanczyk FZ, Sokol RZ. Serum inhibin B levels in males with gonadal dysfunction. Fertil Steril 2000; 74(2):234–238. pmid:10927037
- Trambas CM, Sikaris KA, Lu ZX. More on biotin treatment mimicking Graves’ disease. N Engl J Med 2016; 375(17):1698. doi:10.1056/NEJMc1611875
- Li F, Yue H, Yamaguchi K, et al. Effect of surgical repair on testosterone production in infertile men with varicocele: a meta-analysis. Int J Urol 2012; 19(2):149–154. doi:10.1111/j.1442-2042.2011.02890.x
- Crosnoe-Shipley LE, Elkelany OO, Rahnema CD, Kim ED. Treatment of hypogonadotropic male hypogonadism: case-based scenarios. World J Nephrol 2015; 4(2):245–253. doi:10.5527/wjn.v4.i2.245
- Majzoub A, Sabanegh E Jr. Testosterone replacement in the infertile man. Transl Androl Urol 2016; 5(6):859–865. doi:10.21037/tau.2016.08.03
- Majumdar A, Mangal NS. Hyperprolactinemia. J Hum Reprod Sci 2013; 6(3):168–175. doi:10.4103/0974-1208.121400
- El Osta R, Grandpre N, Monnin N, Hubert J, Koscinski I. Hypogonadotropic hypogonadism in men with hereditary hemochromatosis. Basic Clin Androl 2017; 27:13. doi:10.1186/s12610-017-0057-8
- Dhindsa S, Miller MG, McWhirter CL, et al. Testosterone concentrations in diabetic and nondiabetic obese men. Diabetes Care 2010; 33(6):1186–1192. doi:10.2337/dc09-1649
- Jasuja GK, Bhasin S, Reisman JI, et al. Who gets testosterone? Patient characteristics associated with testosterone prescribing in the Veteran Affairs system: a cross-sectional study. J Gen Intern Med 2017; 32(3):304–311. doi:10.1007/s11606-016-3940-7
- Kaplan SA, Lee JY, O’Neill EA, Meehan AG, Kusek JW. Prevalence of low testosterone and its relationship to body mass index in older men with lower urinary tract symptoms associated with benign prostatic hyperplasia. Aging Male 2013; 16(4):169–172. doi:10.3109/13685538.2013.844786
- Lee HK, Lee JK, Cho B. The role of androgen in the adipose tissue of males. World J Mens Health 2013; 31(2):136–140. doi:10.5534/wjmh.2013.31.2.136
- Raven G, De Jong FH, Kaufman JM, De Ronde W. In men, peripheral estradiol levels directly reflect the action of estrogens at the hypothalamo-pituitary level to inhibit gonadotropin secretion. J Clin Endocrinol Metab 2006; 91(9):3324–3328. doi:10.1210/jc.2006-0462
- Hofny ER, Ali ME, Abdel-Hafez HZ, et al. Semen parameters and hormonal profile in obese fertile and infertile males. Fertil Steril 2010; 94(2):581–584. doi:10.1016/j.fertnstert.2009.03.085
- Isidori AM, Caprio M, Strollo F, et al. Leptin and androgens in male obesity: evidence for leptin contribution to reduced androgen levels. J Clin Endocrinol Metab 1999; 84(10):3673–3680. doi:10.1210/jcem.84.10.6082
- El-Wakkad A, Hassan NM, Sibaii H, El-Zayat SR. Proinflammatory, anti-inflammatory cytokines and adiponkines in students with central obesity. Cytokine 2013; 61(2):682–687. doi:10.1016/j.cyto.2012.11.010
- Maggio M, Basaria S, Ceda GP, et al. The relationship between testosterone and molecular markers of inflammation in older men. J Endocrinol Invest 2005; 28(suppl proceedings 11):116–119. pmid:16760639
- de Lorenzo A, Noce A, Moriconi E, et al. MOSH syndrome (male obesity secondary hypogonadism): clinical assessment and possible therapeutic approaches. Nutrients 2018; 10(4)pii:E474. doi:10.3390/nu10040474
- Escobar-Morreale HF, Santacruz E, Luque-Ramírez M, Botella Carretero JI. Prevalence of ‘obesity-associated gonadal dysfunction’ in severely obese men and women and its resolution after bariatric surgery: a systematic review and meta-analysis. Hum Reprod Update 2017; 23(4):390–408. doi:10.1093/humupd/dmx012
- Lo EM, Rodriguez KM, Pastuszak AW, Khera M. Alternatives to testosterone therapy: a review. Sex Med Rev 2018; 6(1):106–113. doi:10.1016/j.sxmr.2017.09.004
- Soares AH, Horie NC, Chiang LAP, et al. Effects of clomiphene citrate on male obesity-associated hypogonadism: a randomized, double-blind, placebo-controlled study. Int J Obes (Lond) 2018; 42(5):953–963. doi:10.1038/s41366-018-0105-2
- Chua ME, Escusa KG, Luna S, Tapia LC, Dofitas B, Morales M. Revisiting oestrogen antagonists (clomiphene or tamoxifen) as medical empiric therapy for idiopathic male infertility: a meta-analysis. Andrology 2013; 1(5):749–757. doi:10.1111/j.2047-2927.2013.00107.x
- Westaby D, Ogle SJ, Paradinas FJ, Randell JB, Murray-Lyon IM. Liver damage from long-term methyltestosterone. Lancet 1977; 2(8032):262–263. pmid:69876
- Fernández-Balsells MM, Murad MH, Lane M, et al. Clinical review 1: Adverse effects of testosterone therapy in adult men: a systematic review and meta-analysis. J Clin Endocrinol Metab 2010; 95(6):2560–2575. doi:10.1210/jc.2009-2575
- Coviello AD, Kaplan B, Lakshman KM, Chen T, Singh AB, Bhasin S. Effects of graded doses of testosterone on erythropoiesis in healthy young and older men. J Clin Endocrinol Metab 2008; 93(3):914–919. doi:10.1210/jc.2007-1692
- Ohlander SJ, Varghese B, Pastuszak AW. Erythrocytosis following testosterone therapy. Sex Med Rev 2018; 6(1):77–85. doi:10.1016/j.sxmr.2017.04.001
- Jones SD Jr, Dukovac T, Sangkum P, Yafi FA, Hellstrom WJ. Erythrocytosis and polycythemia secondary to testosterone replacement therapy in the aging male. Sex Med Rev 2015; 3(2):101–112. doi:10.1002/smrj.43
- Bachman E, Travison TG, Basaria S, et al. Testosterone induces erythrocytosis via increased erythropoietin and suppressed hepcidin: evidence for a new erythropoietin/hemoglobin set point. J Gerontol A Biol Sci Med Sci 2014; 69(6):725–735. doi:10.1093/gerona/glt154
- Roy CN, Snyder PJ, Stephens-Shields AJ, et al. Association of testosterone levels with anemia in older men: a controlled clinical trial. JAMA Intern Med 2017; 177(4):480–490. doi:10.1001/jamainternmed.2016.9540
- Klap J, Schmid M, Loughlin KR. The relationship between total testosterone levels and prostate cancer: a review of the continuing controversy. J Urol 2015; 193(2):403–413. doi:10.1016/j.juro.2014.07.123
- Gilbert SM, Cavallo CB, Kahane H, Lowe FC. Evidence suggesting PSA cutpoint of 2.5 ng/mL for prompting prostate biopsy: review of 36,316 biopsies. Urology 2005; 65(3):549–553. doi:10.1016/j.urology.2004.10.064
- Araujo AB, Dixon JM, Suarez EA, Murad MH, Guey LT, Wittert GA. Clinical review: Endogenous testosterone and mortality in men: a systematic review and meta-analysis. J Clin Endocrinol Metab 2011; 96(10):3007–3019. doi:10.1210/jc.2011-1137
- Xu L, Freeman G, Cowling BJ, Schooling CM. Testosterone therapy and cardiovascular events among men: a systematic review and meta-analysis of placebo-controlled randomized trials. BMC Med 2013; 11:108. doi:10.1186/1741-7015-11-108
A 48-year-old man presents to his primary care physician because of progressively decreasing energy and gradual decline in both libido and erectile function for the past 18 months. He has noticed decreased morning erections as well. He rates his libido at 3 to 4 on a scale of 10 for the past 6 months. He also reports poor motivation, depressed mood, impaired concentration, and sleep disturbances. He reports no hair loss, headache, or dizziness, and no decrease in shaving frequency. Review of his systems is otherwise unremarkable.
He has had dyslipidemia for 3 years and is not known to have hypertension or diabetes. His medications include atorvastatin, vitamin E, and multivitamins.
He is married with 3 children and does not wish to have more. He works as a software engineer and leads a sedentary lifestyle. He is a nonsmoker and occasionally drinks alcohol on the weekends.
On physical examination, he is alert and oriented and appears well. His height is 5 feet 10 inches (178 cm), weight 230 lb (104 kg), and body mass index (BMI) 32.8 kg/m2. His blood pressure is 115/83 mm Hg and pulse rate is 82 beats per minute and regular. Findings on cardiovascular and pulmonary examination are normal. He has large fatty breasts but without palpable glandular tissue.
Genitourinary examination reveals normal hair distribution, a normal-sized penis, and slightly soft testes with testicular volume of 18–20 mL bilaterally.
His primary care physician suspects that he has low testosterone and orders some basic laboratory tests; the results are normal except for a low total testosterone level (Table 1).
FURTHER TESTING
1. Which of the following tests should his physician order next?
- Repeat total testosterone measurement
- Free testosterone measurement by commercial assay
- Calculated free testosterone
- Bioavailable testosterone measurement
- Serum inhibin B measurement
This patient presents with several nonspecific symptoms. But collectively they suggest testosterone deficiency (hypogonadism).
Together, erectile dysfunction, low libido, and decreased morning erections strongly suggest hypogonadism.2 Loss of body hair and decreased shaving frequency are specific symptoms of hypogonadism; however, they require years to develop.3 Gynecomastia can also occur due to loss of the inhibitory action of testosterone on breast growth and a relative increase in estradiol. This occurs more in primary hypogonadism, due to the increase in luteinizing hormone (LH), which stimulates the remaining Leydig cells to secrete estradiol rather than testosterone.4
To diagnose hypogonadism in men and to start treatment for it, current guidelines recommend that the patient should have clinical features as well as laboratory evidence of low testosterone.5,6
Measuring testosterone: Total, free, bound, and bioavailable
Testosterone, a steroid hormone, circulates in the serum either as free testosterone or bound to several plasma proteins, mainly sex-hormone binding globulin (SHBG) and albumin.
Total testosterone includes both the free and bound fractions, whereas bioavailable testosterone includes both free and the portion bound to albumin, which has low affinity and can dissociate and be used at the tissue level.11
Low levels of total testosterone do not necessarily reflect a hypogonadal state, as a man with altered SHBG levels or binding capabilities can have low total but normal free testosterone levels and no manifestations.12 Several conditions can alter the levels of SHBG, including obesity, diabetes, aging, thyroid dysfunction, and others.5,13
Because our patient is obese, his total testosterone level is not a reliable indicator of hypogonadism, and repeating its measurement will not add diagnostic value.
Therefore, an alternative measurement should be used to accurately reflect the testosterone levels. From a physiologic point of view, bioavailable testosterone is the active form of testosterone and is the most accurate to be measured in a patient with hypogonadism. Nevertheless, because of technical difficulties in its measurement and lack of evidence correlating bioavailable testosterone with the clinical picture of hypogonadism, it is recommended that the level of free testosterone be used.5
The gold standard for direct measurement of serum free testosterone is equilibrium dialysis, but this is expensive and time-consuming.14 Commercial assays for free testosterone exist but have been deemed unreliable.14,15 It is recommended that free testosterone be measured by equilibrium dialysis or calculated using equations based on total testosterone, SHBG, and albumin levels.5 These equations are reliable and give results very close to the values obtained by equilibrium dialysis.15 Therefore, in our patient, it would be suitable to calculate the free testosterone level next.
Serum levels of free testosterone vary according to several factors. Diurnal variation of testosterone has been established: levels are highest in the morning and decline throughout the day.16 Food decreases testosterone levels.17 In addition, there is considerable day-to-day variation.18 Therefore, at least 2 readings of fasting morning testosterone on 2 separate days are recommended for the diagnosis of hypogonadism.5
Inhibin B is a hormone produced by Sertoli cells in the testes in response to follicle-stimulating hormone (FSH) stimulation. In turn, it acts as negative feedback, together with testosterone, to inhibit FSH release from the pituitary. Inhibin B has been shown to reflect spermatogenesis in the testes and therefore fertility.19 Inhibin B levels were found to be low in patients with central hypogonadism, due to less FSH release; however, they did not correlate with testosterone levels.20
CASE RESUMED: CHARACTERIZING HIS HYPOGONADISM
The patient’s physician orders morning fasting total testosterone, SHBG, and albumin testing and calculates the free testosterone level, which yields a value of 3 ng/dL (reference range 4.5–17). This is confirmed by a repeat measurement, which yields a value of 2.9 ng/dL. Laboratory test results combined with his clinical presentation are consistent with hypogonadism.
2. What is the most appropriate next step?
- Measurement of serum LH and FSH
- Measurement of serum prolactin
- Scrotal ultrasonography
- Gonadotropin-releasing hormone (GnRH) stimulation test
- Semen analysis
After hypogonadism is diagnosed, it is important to distinguish if it is primary or central. This is achieved by measuring serum LH and FSH.5 All biotin supplements should be stopped at least 72 hours before measuring LH and FSH, as biotin can interfere with the assays, yielding false values.21
Secretion of FSH and LH from the anterior pituitary is under the influence of pulsatile release of GnRH from the hypothalamus. LH acts on Leydig cells in the testes to produce testosterone, whereas FSH acts on Sertoli cells, together with testosterone, to bring about spermatogenesis in the seminiferous tubules. Testosterone acts centrally as negative feedback to decrease the release of LH and FSH.
Primary hypogonadism occurs due to testicular failure, ie, the testes themselves fail to produce testosterone, leading to hypogonadism. The decrease in testosterone levels, together with inhibin B if Sertoli cells are damaged, lead to loss of negative feedback on the hypothalamus and pituitary, and therefore increased levels of LH and FSH. This is termed hypergonadotropic hypogonadism. Testicular failure may also result in impaired spermatogenesis and infertility due to destruction of testicular structures, in which case fertility cannot be restored.
Central hypogonadism occurs when the pituitary fails to produce LH and FSH (secondary hypogonadism) or when the hypothalamus fails to produce GnRH and subsequently the lack of secretion of LH and FSH from the pituitary (tertiary hypogonadism). The lack of LH will result in no stimulation of Leydig cells to produce testosterone, and therefore its deficiency. Serum hormone levels in central hypogonadism will reveal low testosterone, with either low or inappropriately normal gonadotropins (LH and FSH). This is termed hypogonadotropic hypogonadism. The lack of FSH, together with testosterone deficiency will also result in decreased spermatogenesis and therefore infertility. Testicular structures are preserved, however, and fertility can be restored with appropriate therapy, as discussed below.
Prolactin should be measured only if the patient has central hypogonadism. Its measurement is not warranted at this point in the patient’s workup. The implications of prolactin and its relationship to hypogonadism will be discussed later.
Although, this stepwise approach is not convenient for many patients, some physicians follow it because it is cost-effective, especially in those who are not insured. However, other physicians order FSH, LH, and sometimes prolactin with the confirmatory low testosterone measurement. Laboratories can also be instructed to wait to measure the pituitary hormones and to do so only if low testosterone is confirmed.
Varicocele, a possible cause of male infertility, can also impair Leydig cell function and cause low testosterone. In fact, surgical repair of varicocele has been demonstrated to increase serum testosterone.22 Scrotal ultrasonography is used to diagnose varicocele, but this also should be ordered at a later stage in the workup if primary hypogonadism is diagnosed.
The GnRH stimulation test is important for the diagnosis and evaluation of precocious or delayed puberty in children. In boys with delayed puberty, a poorer response to GnRH stimulation indicates central hypogonadism rather than constitutional delay.23 It has no role in the evaluation of postpubertal or adult-onset hypogonadism.
Semen analysis is important to evaluate fertility if the patient is interested in further procreation.5 Low testosterone levels may result in impaired spermatogenesis and therefore infertility. On the other hand, treatment with exogenous testosterone will also result in infertility, by feedback inhibition of LH and FSH and therefore inhibition of spermatogenesis. If the patient wishes to preserve fertility, treatment options other than testosterone should be considered; examples include clomiphene citrate, human menopausal gonadotropin, and human chorionic gonadotropin.23,24
Our patient has no desire to expand his family; therefore, a semen analysis and attempts to preserve spermatogenesis are not indicated.
CASE RESUMED: SEARCHING FOR CAUSES
His physician orders testing of serum LH and FSH, yielding the following values:
- LH 1.6 mIU/mL (reference range 1.8–12)
- FSH 1.9 mIU/mL (reference range 1.5–12.5).
The diagnosis of central hypogonadism is established.
3. Which investigation is the least appropriate in the further evaluation of this patient?
- Serum prolactin measurement
- Serum ferritin measurement
- Pituitary magnetic resonance imaging (MRI)
- Chromosomal karyotyping
The diagnosis of central hypogonadism warrants evaluation for possible causes. These are summarized in Table 4.
Serum free thyroxine and morning cortisol
Since this patient’s LH and FSH values are abnormal, it is important to evaluate the status of other anterior pituitary hormones. In patients with pituitary abnormalities, serum free T4 is a more reliable test for assessing thyroid function than thyroid-stimulating hormone (TSH), because of loss of the negative feedback of thyroid hormones on the diseased pituitary. In contrast, serum TSH is considered the best single thyroid test to assess primary thyroid dysfunction.
Other measurements include prolactin and morning cortisol (reflecting adrenocorticotropic hormone status).
Prolactin measurement
Prolactin measurement is important to evaluate for hyperprolactinemia, as this will lead to hypogonadism by inhibition of GnRH secretion.25 Different pathologic, pharmacologic, and physiologic conditions can result in hyperprolactinemia, including prolactinomas, other pituitary and hypothalamic lesions, primary hypothyroidism, and medications such as antipsychotics.25 Dopamine agonists are the mainstay treatment for hyperprolactinemia.
Ferritin measurement
Ferritin measurement is indicated to diagnose iron overload conditions such as hemochromatosis, which can result in primary hypogonadism via testicular damage or in secondary hypogonadism via pituitary damage.26
Pituitary MRI with contrast
Pituitary MRI with contrast is used to diagnose structural lesions of the pituitary or hypothalamus. This diagnostic modality is indicated for patients with pituitary dysfunction, including central hypogonadism, manifestations of a mass effect (headache, visual field defects), persistent hyperprolactinemia, and panhypopituitarism, among others. To improve the diagnostic yield of pituitary MRI, the Endocrine Society guidelines recommend it for men with serum total testosterone levels below 150 ng/dL.5 However, some clinicians have a lower threshold for ordering pituitary MRI for patients with central hypogonadism. Physician judgment and expertise should be exercised and the decision made on an individual basis.
Chromosomal karyotyping
Chromosomal karyotyping is not indicated in our patient. It is reserved for those with primary hypogonadism to diagnose Klinefelter syndrome, which has a karyotype of 47,XXY.
CASE RESUMED: MOSH SYNDROME
Our patient’s prolactin, free T4, morning cortisol, and ferritin levels are measured, yielding normal values. No abnormalities are seen on pituitary MRI. A clinical reevaluation is conducted, revealing no history of head trauma or head and neck radiation. The lack of an obvious cause in our patient’s clinical presentation and workup, together with his obesity (BMI 32.8 kg/m2) supports the diagnosis of obesity as the cause of his hypogonadism.
Obesity can be a cause of secondary hypogonadism, which has led to the term “MOSH” (male obesity-associated secondary hypogonadism) syndrome. In fact, a cross-sectional study has demonstrated that 40% of nondiabetic obese (BMI ≥ 30 kg/m2) men over age 45 have low serum free testosterone levels, compared with 26% for lean (BMI < 25 kg/m2) men.27 Moreover, obesity has been found to be a strong predictor of testosterone replacement therapy.28 Other studies have also found an inverse relationship between BMI and testosterone levels.29
Several mechanisms interact in the pathogenesis of MOSH syndrome. Adipose tissue possesses aromatase activity, which converts androgens into estrogens.30 Peripheral estrogen production can in turn exert feedback inhibition on pituitary gonadotropin secretion.31 In obese men, increased adipose tissue leads to increased aromatase activity and more estrogen, so more feedback inhibition on the pituitary and subsequently secondary hypogonadism.
Leptin, a hormone produced by adipocytes, is also increased in obesity, and was found to be inversely correlated with serum testosterone.32 Studies have demonstrated that leptin has an inhibitory effect on the enzymatic pathway that synthesizes testosterone in Leydig cells.33
Proinflammatory cytokines have also been implicated, as central obesity is associated with an increase in these cytokines, which in turn act negatively on the hypothalamus and impair GnRH release leading to lower testosterone.34,35
Treating obesity-related hypogonadism
In a pilot study,36 lifestyle attempts to reduce obesity were shown to improve hormonal levels. Bariatric surgery has also been demonstrated to be successful.37
Clomiphene citrate, a selective estrogen receptor modulator, increases endogenous testosterone secretion by inhibiting the negative feedback of estrogen on the hypothalamus and pituitary and thus increasing LH and FSH. It also preserves endogenous testosterone production, since it does not suppress the hypothalamic-pituitary-testicular axis.38 This made clomiphene citrate a potential treatment for men with central hypogonadism including those with MOSH.39
Nevertheless, there are no randomized trials to prove its safety and efficacy in the management of central hypogonadism.5 Regarding its use in men wishing to preserve fertility, most studies did not show improvement. However, a meta-analysis demonstrated statistically significant increased pregnancy rates in partners of men with idiopathic infertility if the men used 50 mg of clomiphene citrate daily.40
Testosterone deficiency can be a marker of metabolic syndrome, which needs to be managed more urgently than hypogonadism. A cross-sectional study found not only an association between metabolic syndrome and low serum testosterone, but also with each individual component of metabolic syndrome on its own, all of which need to be addressed.10
CASE CONTINUED: BEGINNING TREATMENT
The physician counsels the patient regarding the implications, potential adverse outcomes, and available treatments for his obesity, including lifestyle modification and bariatric surgery. The patient declines surgery and wishes to adopt a weight-reducing diet and exercise program, for which he is referred to a dietitian.
In addition, in view of the patient’s clinically and biochemically proven hypogonadism, his physician offers testosterone replacement therapy. He orders a serum prostate-specific antigen (PSA) level, which is 1.3 ng/dL (reference range < 4 ng/dL). The patient is prescribed 5 g of 1% testosterone gel daily.
TESTOSTERONE REPLACEMENT THERAPY
4. Which is the most common adverse effect of testosterone replacement therapy?
- Cardiovascular events
- Erythrocytosis
- Prostate cancer
- Infertility
- Obstructive sleep apnea
Clinicians should be very cautious in initiating testosterone replacement therapy in any patient with an unstable medical condition.
There are several formulations of testosterone replacement therapy, including intramuscular injections, transdermal gels or patches, buccal tablets, an intranasal gel, and oral tablets. Of note, there are 2 different forms of oral testosterone preparations: testosterone undecanoate and 17-alpha alkylated testosterone. The former is unavailable in the United States and the latter is not recommended for use due to its proven hepatic toxicity.41
Testosterone and erythrocytosis
Meta-analyses have concluded that the most frequent adverse event of testosterone replacement therapy is a significant rise in hematocrit.42 This rise was found to be dose-dependent and was more marked in older men.43 Although all preparations can cause erythrocytosis, parenteral forms have been observed to raise it the most, particularly short-term injectables.44,45
The mechanism behind this increase is attributed to increased erythropoietin levels and improved usage of iron for red blood cell synthesis.46 In fact, testosterone replacement therapy has been shown to improve hemoglobin levels in patients with anemia.47 On the other hand, increasing hematocrit levels may lead to thrombotic and vasoocclusive events.44
It is strongly recommended that baseline hematocrit levels be measured before initiating testosterone replacement therapy.5,6 The hematocrit level should also be monitored 3 to 6 months into treatment and yearly thereafter while on testosterone.5Figure 1 summarizes the appropriate steps to undertake regarding hematocrit levels, according to the American Urological Association.6
Testosterone and prostate cancer
The relationship between testosterone treatment and prostate cancer has long been studied. Historically, testosterone replacement therapy was believed to increase the risk of prostate cancer; however, recent studies and meta-analyses have shown that this is not the case.42,48 Nevertheless, clinical guidelines still recommend prostate monitoring for men on testosterone replacement therapy.5,6
Testosterone and cardiovascular risk
The evidence regarding this issue has been contradictory and inconsistent. Meta-analyses have demonstrated that low testosterone is associated with higher risk of major adverse cardiovascular events.50 These studies argue for the use of testosterone replacement therapy in hypogonadal men to decrease the risk. However, other studies and meta-analyses have found that testosterone replacement therapy is associated with increased cardiovascular risk and have concluded that major adverse cardiac events are in fact a risk of testosterone replacement therapy.51
Current recommendations advocate against the use of testosterone replacement therapy in men with uncontrolled heart failure or with cardiovascular events in the past 3 to 6 months.5,6 Cardiovascular risk factors should be addressed and corrected, and patients should be educated on cardiovascular symptoms and the need to report them if they occur.
Testosterone and infertility
As described earlier, testosterone replacement therapy increases negative feedback on the pituitary and decreases LH and FSH production, leading to less spermatogenesis. Other treatment options should be sought for hypogonadal men wishing to preserve fertility.
Other adverse effects
Other adverse effects of testosterone replacement therapy include acne, oily skin, obstructive sleep apnea, gynecomastia, and balding.
Given all the adverse events that can be associated with testosterone replacement therapy, the risks and benefits of treating hypogonadism in each patient should be taken into consideration, and an individualized approach is required.
CASE RESUMED: FOLLOW-UP
The patient presents 3 months later for follow-up. He reports significant improvement in his presenting symptoms including energy, libido, and erectile function. He also reports some improvement in his mood and concentration. He has lost 12 lb (5.4 kg) and is still trying to improve his diet and exercise program. He is compliant with his testosterone gel therapy.
His serum calculated free testosterone level is 7.8 ng/dL (4.5–17), and his hematocrit is 46%. The patient is instructed to continue his treatment and to return after 9 months for further follow-up.
TAKE-HOME POINTS
- Men with hypogonadism usually present with nonspecific manifestations, so clinicians should keep a high index of suspicion.
- Both clinical and biochemical evidence of hypogonadism should be present to diagnose and start treatment for it.
- Low levels of serum total testosterone do not necessarily reflect hypogonadism.
- The hormonal profile of central hypogonadism reveals low serum testosterone with low or inappropriately normal serum LH and FSH levels.
Obesity can cause central hypogonadism and should be suspected after pituitary and other systemic causes are excluded.
A 48-year-old man presents to his primary care physician because of progressively decreasing energy and gradual decline in both libido and erectile function for the past 18 months. He has noticed decreased morning erections as well. He rates his libido at 3 to 4 on a scale of 10 for the past 6 months. He also reports poor motivation, depressed mood, impaired concentration, and sleep disturbances. He reports no hair loss, headache, or dizziness, and no decrease in shaving frequency. Review of his systems is otherwise unremarkable.
He has had dyslipidemia for 3 years and is not known to have hypertension or diabetes. His medications include atorvastatin, vitamin E, and multivitamins.
He is married with 3 children and does not wish to have more. He works as a software engineer and leads a sedentary lifestyle. He is a nonsmoker and occasionally drinks alcohol on the weekends.
On physical examination, he is alert and oriented and appears well. His height is 5 feet 10 inches (178 cm), weight 230 lb (104 kg), and body mass index (BMI) 32.8 kg/m2. His blood pressure is 115/83 mm Hg and pulse rate is 82 beats per minute and regular. Findings on cardiovascular and pulmonary examination are normal. He has large fatty breasts but without palpable glandular tissue.
Genitourinary examination reveals normal hair distribution, a normal-sized penis, and slightly soft testes with testicular volume of 18–20 mL bilaterally.
His primary care physician suspects that he has low testosterone and orders some basic laboratory tests; the results are normal except for a low total testosterone level (Table 1).
FURTHER TESTING
1. Which of the following tests should his physician order next?
- Repeat total testosterone measurement
- Free testosterone measurement by commercial assay
- Calculated free testosterone
- Bioavailable testosterone measurement
- Serum inhibin B measurement
This patient presents with several nonspecific symptoms. But collectively they suggest testosterone deficiency (hypogonadism).
Together, erectile dysfunction, low libido, and decreased morning erections strongly suggest hypogonadism.2 Loss of body hair and decreased shaving frequency are specific symptoms of hypogonadism; however, they require years to develop.3 Gynecomastia can also occur due to loss of the inhibitory action of testosterone on breast growth and a relative increase in estradiol. This occurs more in primary hypogonadism, due to the increase in luteinizing hormone (LH), which stimulates the remaining Leydig cells to secrete estradiol rather than testosterone.4
To diagnose hypogonadism in men and to start treatment for it, current guidelines recommend that the patient should have clinical features as well as laboratory evidence of low testosterone.5,6
Measuring testosterone: Total, free, bound, and bioavailable
Testosterone, a steroid hormone, circulates in the serum either as free testosterone or bound to several plasma proteins, mainly sex-hormone binding globulin (SHBG) and albumin.
Total testosterone includes both the free and bound fractions, whereas bioavailable testosterone includes both free and the portion bound to albumin, which has low affinity and can dissociate and be used at the tissue level.11
Low levels of total testosterone do not necessarily reflect a hypogonadal state, as a man with altered SHBG levels or binding capabilities can have low total but normal free testosterone levels and no manifestations.12 Several conditions can alter the levels of SHBG, including obesity, diabetes, aging, thyroid dysfunction, and others.5,13
Because our patient is obese, his total testosterone level is not a reliable indicator of hypogonadism, and repeating its measurement will not add diagnostic value.
Therefore, an alternative measurement should be used to accurately reflect the testosterone levels. From a physiologic point of view, bioavailable testosterone is the active form of testosterone and is the most accurate to be measured in a patient with hypogonadism. Nevertheless, because of technical difficulties in its measurement and lack of evidence correlating bioavailable testosterone with the clinical picture of hypogonadism, it is recommended that the level of free testosterone be used.5
The gold standard for direct measurement of serum free testosterone is equilibrium dialysis, but this is expensive and time-consuming.14 Commercial assays for free testosterone exist but have been deemed unreliable.14,15 It is recommended that free testosterone be measured by equilibrium dialysis or calculated using equations based on total testosterone, SHBG, and albumin levels.5 These equations are reliable and give results very close to the values obtained by equilibrium dialysis.15 Therefore, in our patient, it would be suitable to calculate the free testosterone level next.
Serum levels of free testosterone vary according to several factors. Diurnal variation of testosterone has been established: levels are highest in the morning and decline throughout the day.16 Food decreases testosterone levels.17 In addition, there is considerable day-to-day variation.18 Therefore, at least 2 readings of fasting morning testosterone on 2 separate days are recommended for the diagnosis of hypogonadism.5
Inhibin B is a hormone produced by Sertoli cells in the testes in response to follicle-stimulating hormone (FSH) stimulation. In turn, it acts as negative feedback, together with testosterone, to inhibit FSH release from the pituitary. Inhibin B has been shown to reflect spermatogenesis in the testes and therefore fertility.19 Inhibin B levels were found to be low in patients with central hypogonadism, due to less FSH release; however, they did not correlate with testosterone levels.20
CASE RESUMED: CHARACTERIZING HIS HYPOGONADISM
The patient’s physician orders morning fasting total testosterone, SHBG, and albumin testing and calculates the free testosterone level, which yields a value of 3 ng/dL (reference range 4.5–17). This is confirmed by a repeat measurement, which yields a value of 2.9 ng/dL. Laboratory test results combined with his clinical presentation are consistent with hypogonadism.
2. What is the most appropriate next step?
- Measurement of serum LH and FSH
- Measurement of serum prolactin
- Scrotal ultrasonography
- Gonadotropin-releasing hormone (GnRH) stimulation test
- Semen analysis
After hypogonadism is diagnosed, it is important to distinguish if it is primary or central. This is achieved by measuring serum LH and FSH.5 All biotin supplements should be stopped at least 72 hours before measuring LH and FSH, as biotin can interfere with the assays, yielding false values.21
Secretion of FSH and LH from the anterior pituitary is under the influence of pulsatile release of GnRH from the hypothalamus. LH acts on Leydig cells in the testes to produce testosterone, whereas FSH acts on Sertoli cells, together with testosterone, to bring about spermatogenesis in the seminiferous tubules. Testosterone acts centrally as negative feedback to decrease the release of LH and FSH.
Primary hypogonadism occurs due to testicular failure, ie, the testes themselves fail to produce testosterone, leading to hypogonadism. The decrease in testosterone levels, together with inhibin B if Sertoli cells are damaged, lead to loss of negative feedback on the hypothalamus and pituitary, and therefore increased levels of LH and FSH. This is termed hypergonadotropic hypogonadism. Testicular failure may also result in impaired spermatogenesis and infertility due to destruction of testicular structures, in which case fertility cannot be restored.
Central hypogonadism occurs when the pituitary fails to produce LH and FSH (secondary hypogonadism) or when the hypothalamus fails to produce GnRH and subsequently the lack of secretion of LH and FSH from the pituitary (tertiary hypogonadism). The lack of LH will result in no stimulation of Leydig cells to produce testosterone, and therefore its deficiency. Serum hormone levels in central hypogonadism will reveal low testosterone, with either low or inappropriately normal gonadotropins (LH and FSH). This is termed hypogonadotropic hypogonadism. The lack of FSH, together with testosterone deficiency will also result in decreased spermatogenesis and therefore infertility. Testicular structures are preserved, however, and fertility can be restored with appropriate therapy, as discussed below.
Prolactin should be measured only if the patient has central hypogonadism. Its measurement is not warranted at this point in the patient’s workup. The implications of prolactin and its relationship to hypogonadism will be discussed later.
Although, this stepwise approach is not convenient for many patients, some physicians follow it because it is cost-effective, especially in those who are not insured. However, other physicians order FSH, LH, and sometimes prolactin with the confirmatory low testosterone measurement. Laboratories can also be instructed to wait to measure the pituitary hormones and to do so only if low testosterone is confirmed.
Varicocele, a possible cause of male infertility, can also impair Leydig cell function and cause low testosterone. In fact, surgical repair of varicocele has been demonstrated to increase serum testosterone.22 Scrotal ultrasonography is used to diagnose varicocele, but this also should be ordered at a later stage in the workup if primary hypogonadism is diagnosed.
The GnRH stimulation test is important for the diagnosis and evaluation of precocious or delayed puberty in children. In boys with delayed puberty, a poorer response to GnRH stimulation indicates central hypogonadism rather than constitutional delay.23 It has no role in the evaluation of postpubertal or adult-onset hypogonadism.
Semen analysis is important to evaluate fertility if the patient is interested in further procreation.5 Low testosterone levels may result in impaired spermatogenesis and therefore infertility. On the other hand, treatment with exogenous testosterone will also result in infertility, by feedback inhibition of LH and FSH and therefore inhibition of spermatogenesis. If the patient wishes to preserve fertility, treatment options other than testosterone should be considered; examples include clomiphene citrate, human menopausal gonadotropin, and human chorionic gonadotropin.23,24
Our patient has no desire to expand his family; therefore, a semen analysis and attempts to preserve spermatogenesis are not indicated.
CASE RESUMED: SEARCHING FOR CAUSES
His physician orders testing of serum LH and FSH, yielding the following values:
- LH 1.6 mIU/mL (reference range 1.8–12)
- FSH 1.9 mIU/mL (reference range 1.5–12.5).
The diagnosis of central hypogonadism is established.
3. Which investigation is the least appropriate in the further evaluation of this patient?
- Serum prolactin measurement
- Serum ferritin measurement
- Pituitary magnetic resonance imaging (MRI)
- Chromosomal karyotyping
The diagnosis of central hypogonadism warrants evaluation for possible causes. These are summarized in Table 4.
Serum free thyroxine and morning cortisol
Since this patient’s LH and FSH values are abnormal, it is important to evaluate the status of other anterior pituitary hormones. In patients with pituitary abnormalities, serum free T4 is a more reliable test for assessing thyroid function than thyroid-stimulating hormone (TSH), because of loss of the negative feedback of thyroid hormones on the diseased pituitary. In contrast, serum TSH is considered the best single thyroid test to assess primary thyroid dysfunction.
Other measurements include prolactin and morning cortisol (reflecting adrenocorticotropic hormone status).
Prolactin measurement
Prolactin measurement is important to evaluate for hyperprolactinemia, as this will lead to hypogonadism by inhibition of GnRH secretion.25 Different pathologic, pharmacologic, and physiologic conditions can result in hyperprolactinemia, including prolactinomas, other pituitary and hypothalamic lesions, primary hypothyroidism, and medications such as antipsychotics.25 Dopamine agonists are the mainstay treatment for hyperprolactinemia.
Ferritin measurement
Ferritin measurement is indicated to diagnose iron overload conditions such as hemochromatosis, which can result in primary hypogonadism via testicular damage or in secondary hypogonadism via pituitary damage.26
Pituitary MRI with contrast
Pituitary MRI with contrast is used to diagnose structural lesions of the pituitary or hypothalamus. This diagnostic modality is indicated for patients with pituitary dysfunction, including central hypogonadism, manifestations of a mass effect (headache, visual field defects), persistent hyperprolactinemia, and panhypopituitarism, among others. To improve the diagnostic yield of pituitary MRI, the Endocrine Society guidelines recommend it for men with serum total testosterone levels below 150 ng/dL.5 However, some clinicians have a lower threshold for ordering pituitary MRI for patients with central hypogonadism. Physician judgment and expertise should be exercised and the decision made on an individual basis.
Chromosomal karyotyping
Chromosomal karyotyping is not indicated in our patient. It is reserved for those with primary hypogonadism to diagnose Klinefelter syndrome, which has a karyotype of 47,XXY.
CASE RESUMED: MOSH SYNDROME
Our patient’s prolactin, free T4, morning cortisol, and ferritin levels are measured, yielding normal values. No abnormalities are seen on pituitary MRI. A clinical reevaluation is conducted, revealing no history of head trauma or head and neck radiation. The lack of an obvious cause in our patient’s clinical presentation and workup, together with his obesity (BMI 32.8 kg/m2) supports the diagnosis of obesity as the cause of his hypogonadism.
Obesity can be a cause of secondary hypogonadism, which has led to the term “MOSH” (male obesity-associated secondary hypogonadism) syndrome. In fact, a cross-sectional study has demonstrated that 40% of nondiabetic obese (BMI ≥ 30 kg/m2) men over age 45 have low serum free testosterone levels, compared with 26% for lean (BMI < 25 kg/m2) men.27 Moreover, obesity has been found to be a strong predictor of testosterone replacement therapy.28 Other studies have also found an inverse relationship between BMI and testosterone levels.29
Several mechanisms interact in the pathogenesis of MOSH syndrome. Adipose tissue possesses aromatase activity, which converts androgens into estrogens.30 Peripheral estrogen production can in turn exert feedback inhibition on pituitary gonadotropin secretion.31 In obese men, increased adipose tissue leads to increased aromatase activity and more estrogen, so more feedback inhibition on the pituitary and subsequently secondary hypogonadism.
Leptin, a hormone produced by adipocytes, is also increased in obesity, and was found to be inversely correlated with serum testosterone.32 Studies have demonstrated that leptin has an inhibitory effect on the enzymatic pathway that synthesizes testosterone in Leydig cells.33
Proinflammatory cytokines have also been implicated, as central obesity is associated with an increase in these cytokines, which in turn act negatively on the hypothalamus and impair GnRH release leading to lower testosterone.34,35
Treating obesity-related hypogonadism
In a pilot study,36 lifestyle attempts to reduce obesity were shown to improve hormonal levels. Bariatric surgery has also been demonstrated to be successful.37
Clomiphene citrate, a selective estrogen receptor modulator, increases endogenous testosterone secretion by inhibiting the negative feedback of estrogen on the hypothalamus and pituitary and thus increasing LH and FSH. It also preserves endogenous testosterone production, since it does not suppress the hypothalamic-pituitary-testicular axis.38 This made clomiphene citrate a potential treatment for men with central hypogonadism including those with MOSH.39
Nevertheless, there are no randomized trials to prove its safety and efficacy in the management of central hypogonadism.5 Regarding its use in men wishing to preserve fertility, most studies did not show improvement. However, a meta-analysis demonstrated statistically significant increased pregnancy rates in partners of men with idiopathic infertility if the men used 50 mg of clomiphene citrate daily.40
Testosterone deficiency can be a marker of metabolic syndrome, which needs to be managed more urgently than hypogonadism. A cross-sectional study found not only an association between metabolic syndrome and low serum testosterone, but also with each individual component of metabolic syndrome on its own, all of which need to be addressed.10
CASE CONTINUED: BEGINNING TREATMENT
The physician counsels the patient regarding the implications, potential adverse outcomes, and available treatments for his obesity, including lifestyle modification and bariatric surgery. The patient declines surgery and wishes to adopt a weight-reducing diet and exercise program, for which he is referred to a dietitian.
In addition, in view of the patient’s clinically and biochemically proven hypogonadism, his physician offers testosterone replacement therapy. He orders a serum prostate-specific antigen (PSA) level, which is 1.3 ng/dL (reference range < 4 ng/dL). The patient is prescribed 5 g of 1% testosterone gel daily.
TESTOSTERONE REPLACEMENT THERAPY
4. Which is the most common adverse effect of testosterone replacement therapy?
- Cardiovascular events
- Erythrocytosis
- Prostate cancer
- Infertility
- Obstructive sleep apnea
Clinicians should be very cautious in initiating testosterone replacement therapy in any patient with an unstable medical condition.
There are several formulations of testosterone replacement therapy, including intramuscular injections, transdermal gels or patches, buccal tablets, an intranasal gel, and oral tablets. Of note, there are 2 different forms of oral testosterone preparations: testosterone undecanoate and 17-alpha alkylated testosterone. The former is unavailable in the United States and the latter is not recommended for use due to its proven hepatic toxicity.41
Testosterone and erythrocytosis
Meta-analyses have concluded that the most frequent adverse event of testosterone replacement therapy is a significant rise in hematocrit.42 This rise was found to be dose-dependent and was more marked in older men.43 Although all preparations can cause erythrocytosis, parenteral forms have been observed to raise it the most, particularly short-term injectables.44,45
The mechanism behind this increase is attributed to increased erythropoietin levels and improved usage of iron for red blood cell synthesis.46 In fact, testosterone replacement therapy has been shown to improve hemoglobin levels in patients with anemia.47 On the other hand, increasing hematocrit levels may lead to thrombotic and vasoocclusive events.44
It is strongly recommended that baseline hematocrit levels be measured before initiating testosterone replacement therapy.5,6 The hematocrit level should also be monitored 3 to 6 months into treatment and yearly thereafter while on testosterone.5Figure 1 summarizes the appropriate steps to undertake regarding hematocrit levels, according to the American Urological Association.6
Testosterone and prostate cancer
The relationship between testosterone treatment and prostate cancer has long been studied. Historically, testosterone replacement therapy was believed to increase the risk of prostate cancer; however, recent studies and meta-analyses have shown that this is not the case.42,48 Nevertheless, clinical guidelines still recommend prostate monitoring for men on testosterone replacement therapy.5,6
Testosterone and cardiovascular risk
The evidence regarding this issue has been contradictory and inconsistent. Meta-analyses have demonstrated that low testosterone is associated with higher risk of major adverse cardiovascular events.50 These studies argue for the use of testosterone replacement therapy in hypogonadal men to decrease the risk. However, other studies and meta-analyses have found that testosterone replacement therapy is associated with increased cardiovascular risk and have concluded that major adverse cardiac events are in fact a risk of testosterone replacement therapy.51
Current recommendations advocate against the use of testosterone replacement therapy in men with uncontrolled heart failure or with cardiovascular events in the past 3 to 6 months.5,6 Cardiovascular risk factors should be addressed and corrected, and patients should be educated on cardiovascular symptoms and the need to report them if they occur.
Testosterone and infertility
As described earlier, testosterone replacement therapy increases negative feedback on the pituitary and decreases LH and FSH production, leading to less spermatogenesis. Other treatment options should be sought for hypogonadal men wishing to preserve fertility.
Other adverse effects
Other adverse effects of testosterone replacement therapy include acne, oily skin, obstructive sleep apnea, gynecomastia, and balding.
Given all the adverse events that can be associated with testosterone replacement therapy, the risks and benefits of treating hypogonadism in each patient should be taken into consideration, and an individualized approach is required.
CASE RESUMED: FOLLOW-UP
The patient presents 3 months later for follow-up. He reports significant improvement in his presenting symptoms including energy, libido, and erectile function. He also reports some improvement in his mood and concentration. He has lost 12 lb (5.4 kg) and is still trying to improve his diet and exercise program. He is compliant with his testosterone gel therapy.
His serum calculated free testosterone level is 7.8 ng/dL (4.5–17), and his hematocrit is 46%. The patient is instructed to continue his treatment and to return after 9 months for further follow-up.
TAKE-HOME POINTS
- Men with hypogonadism usually present with nonspecific manifestations, so clinicians should keep a high index of suspicion.
- Both clinical and biochemical evidence of hypogonadism should be present to diagnose and start treatment for it.
- Low levels of serum total testosterone do not necessarily reflect hypogonadism.
- The hormonal profile of central hypogonadism reveals low serum testosterone with low or inappropriately normal serum LH and FSH levels.
Obesity can cause central hypogonadism and should be suspected after pituitary and other systemic causes are excluded.
- Araujo AB, Esche GR, Kupelian V, et al. Prevalence of symptomatic androgen deficiency in men. J Clin Endocrinol Metab 2007; 92(11):4241–4247. doi:10.1210/jc.2007-1245
- Wu FCW, Tajar A, Beynon JM, et al; EMAS Group. Identification of late-onset hypogonadism in middle-aged and elderly men. N Engl J Med 2010; 363(2):123–135. doi:10.1056/NEJMoa0911101
- Arver S, Lehtihet M. Current guidelines for the diagnosis of testosterone deficiency. Front Horm Res 2009; 37:5–20. doi:10.1159/000175839
- Narula HS, Carlson HE. Gynaecomastia—pathophysiology, diagnosis and treatment. Nat Rev Endocrinol 2014; 10(11):684–698. doi:10.1038/nrendo.2014.139
- Bhasin S, Brito JP, Cunningham GR, et al. Testosterone therapy in men with hypogonadism: an Endocrine Society clinical practice guideline. J Clin Endocrinol Metab 2018; 103(5):1715–1744. doi:10.1210/jc.2018-00229
- Mulhall JP, Trost LW, Brannigan RE, et al. Evaluation and management of testosterone deficiency: AUA guideline. J Urol 2018; 200(2):423–432. doi:10.1016/j.juro.2018.03.115
- Balasubramanian V, Naing S. Hypogonadism in chronic obstructive pulmonary disease: incidence and effects. Curr Opin Pulm Med 2012; 18(2):112–117. doi:10.1097/MCP.0b013e32834feb37
- Atlantis E, Fahey P, Cochrane B, Wittert G, Smith S. Endogenous testosterone level and testosterone supplementation therapy in chronic obstructive pulmonary disease (COPD): a systematic review and meta-analysis. BMJ Open 2013; 3(8)pii:e003127. doi:10.1136/bmjopen-2013-003127
- Bawor M, Bami H, Dennis BB, et al. Testosterone suppression in opioid users: a systematic review and meta-analysis. Drug Alcohol Depend 2015; 149:1–9. doi:10.1016/j.drugalcdep.2015.01.038
- Tan WS, Ng CJ, Khoo EM, Low WY, Tan HM. The triad of erectile dysfunction, testosterone deficiency syndrome and metabolic syndrome: findings from a multi-ethnic Asian men study (The Subang Men's Health Study). Aging Male 2011; 14(4):231–236. doi:10.3109/13685538.2011.597463
- Goldman AL, Bhasin S, Wu FCW, Krishna M, Matsumoto AM, Jasuja R. A reappraisal of testosterone’s binding in circulation: physiological and clinical implications. Endocr Rev 2017; 38(4):302–324. doi:10.1210/er.2017-00025
- Antonio L, Wu FC, O’Neill TW, et al; European Male Ageing Study Study Group. Low free testosterone is associated with hypogonadal signs and symptoms in men with normal total testosterone. J Clin Endocrinol Metab 2016; 101(7):2647–2657. doi:10.1210/jc.2015-4106
- Liu F, Shen X, Wang R, et al. Association of central obesity with sex hormone binding globulin: a cross-sectional study of 1166 Chinese men. Open Med (Wars) 2018; 13:196–202. doi:10.1515/med-2018-0030
- Vermeulen A, Verdonck L, Kaufman JM. A critical evaluation of simple methods for the estimation of free testosterone in serum. J Clin Endocrinol Metab 1999; 84(10):3666–3672. doi:10.1210/jcem.84.10.6079
- Halmenschlager G, Rhoden EL, Riedner CE. Calculated free testosterone and radioimmunoassay free testosterone as a predictor of subnormal levels of total testosterone. Int Urol Nephrol 2012; 44(3):673–681. doi:10.1007/s11255-011-0066-z
- Brambilla DJ, Matsumoto AM, Araujo AB, McKinlay JB. The effect of diurnal variation on clinical measurement of serum testosterone and other sex hormone levels in men. J Clin Endocrinol Metab 2009; 94(3):907–913. doi:10.1210/jc.2008-1902
- Lehtihet M, Arver S, Bartuseviciene I, Pousette Å. S-testosterone decrease after a mixed meal in healthy men independent of SHBG and gonadotrophin levels. Andrologia 2012; 44(6):405–410. doi:10.1111/j.1439-0272.2012.01296.x
- Brambilla DJ, O’Donnell AB, Matsumoto AM, McKinlay JB. Intraindividual variation in levels of serum testosterone and other reproductive and adrenal hormones in men. Clin Endocrinol (Oxf) 2007; 67(6):853–862. doi:10.1111/j.1365-2265.2007.02976.x
- Manzoor SM, Sattar A, Hashim R, et al. Serum inhibin B as a diagnostic marker of male infertility. J Ayub Med Coll Abbottabad 2012; 24(3–4):113–116. pmid:24669628
- Kolb BA, Stanczyk FZ, Sokol RZ. Serum inhibin B levels in males with gonadal dysfunction. Fertil Steril 2000; 74(2):234–238. pmid:10927037
- Trambas CM, Sikaris KA, Lu ZX. More on biotin treatment mimicking Graves’ disease. N Engl J Med 2016; 375(17):1698. doi:10.1056/NEJMc1611875
- Li F, Yue H, Yamaguchi K, et al. Effect of surgical repair on testosterone production in infertile men with varicocele: a meta-analysis. Int J Urol 2012; 19(2):149–154. doi:10.1111/j.1442-2042.2011.02890.x
- Crosnoe-Shipley LE, Elkelany OO, Rahnema CD, Kim ED. Treatment of hypogonadotropic male hypogonadism: case-based scenarios. World J Nephrol 2015; 4(2):245–253. doi:10.5527/wjn.v4.i2.245
- Majzoub A, Sabanegh E Jr. Testosterone replacement in the infertile man. Transl Androl Urol 2016; 5(6):859–865. doi:10.21037/tau.2016.08.03
- Majumdar A, Mangal NS. Hyperprolactinemia. J Hum Reprod Sci 2013; 6(3):168–175. doi:10.4103/0974-1208.121400
- El Osta R, Grandpre N, Monnin N, Hubert J, Koscinski I. Hypogonadotropic hypogonadism in men with hereditary hemochromatosis. Basic Clin Androl 2017; 27:13. doi:10.1186/s12610-017-0057-8
- Dhindsa S, Miller MG, McWhirter CL, et al. Testosterone concentrations in diabetic and nondiabetic obese men. Diabetes Care 2010; 33(6):1186–1192. doi:10.2337/dc09-1649
- Jasuja GK, Bhasin S, Reisman JI, et al. Who gets testosterone? Patient characteristics associated with testosterone prescribing in the Veteran Affairs system: a cross-sectional study. J Gen Intern Med 2017; 32(3):304–311. doi:10.1007/s11606-016-3940-7
- Kaplan SA, Lee JY, O’Neill EA, Meehan AG, Kusek JW. Prevalence of low testosterone and its relationship to body mass index in older men with lower urinary tract symptoms associated with benign prostatic hyperplasia. Aging Male 2013; 16(4):169–172. doi:10.3109/13685538.2013.844786
- Lee HK, Lee JK, Cho B. The role of androgen in the adipose tissue of males. World J Mens Health 2013; 31(2):136–140. doi:10.5534/wjmh.2013.31.2.136
- Raven G, De Jong FH, Kaufman JM, De Ronde W. In men, peripheral estradiol levels directly reflect the action of estrogens at the hypothalamo-pituitary level to inhibit gonadotropin secretion. J Clin Endocrinol Metab 2006; 91(9):3324–3328. doi:10.1210/jc.2006-0462
- Hofny ER, Ali ME, Abdel-Hafez HZ, et al. Semen parameters and hormonal profile in obese fertile and infertile males. Fertil Steril 2010; 94(2):581–584. doi:10.1016/j.fertnstert.2009.03.085
- Isidori AM, Caprio M, Strollo F, et al. Leptin and androgens in male obesity: evidence for leptin contribution to reduced androgen levels. J Clin Endocrinol Metab 1999; 84(10):3673–3680. doi:10.1210/jcem.84.10.6082
- El-Wakkad A, Hassan NM, Sibaii H, El-Zayat SR. Proinflammatory, anti-inflammatory cytokines and adiponkines in students with central obesity. Cytokine 2013; 61(2):682–687. doi:10.1016/j.cyto.2012.11.010
- Maggio M, Basaria S, Ceda GP, et al. The relationship between testosterone and molecular markers of inflammation in older men. J Endocrinol Invest 2005; 28(suppl proceedings 11):116–119. pmid:16760639
- de Lorenzo A, Noce A, Moriconi E, et al. MOSH syndrome (male obesity secondary hypogonadism): clinical assessment and possible therapeutic approaches. Nutrients 2018; 10(4)pii:E474. doi:10.3390/nu10040474
- Escobar-Morreale HF, Santacruz E, Luque-Ramírez M, Botella Carretero JI. Prevalence of ‘obesity-associated gonadal dysfunction’ in severely obese men and women and its resolution after bariatric surgery: a systematic review and meta-analysis. Hum Reprod Update 2017; 23(4):390–408. doi:10.1093/humupd/dmx012
- Lo EM, Rodriguez KM, Pastuszak AW, Khera M. Alternatives to testosterone therapy: a review. Sex Med Rev 2018; 6(1):106–113. doi:10.1016/j.sxmr.2017.09.004
- Soares AH, Horie NC, Chiang LAP, et al. Effects of clomiphene citrate on male obesity-associated hypogonadism: a randomized, double-blind, placebo-controlled study. Int J Obes (Lond) 2018; 42(5):953–963. doi:10.1038/s41366-018-0105-2
- Chua ME, Escusa KG, Luna S, Tapia LC, Dofitas B, Morales M. Revisiting oestrogen antagonists (clomiphene or tamoxifen) as medical empiric therapy for idiopathic male infertility: a meta-analysis. Andrology 2013; 1(5):749–757. doi:10.1111/j.2047-2927.2013.00107.x
- Westaby D, Ogle SJ, Paradinas FJ, Randell JB, Murray-Lyon IM. Liver damage from long-term methyltestosterone. Lancet 1977; 2(8032):262–263. pmid:69876
- Fernández-Balsells MM, Murad MH, Lane M, et al. Clinical review 1: Adverse effects of testosterone therapy in adult men: a systematic review and meta-analysis. J Clin Endocrinol Metab 2010; 95(6):2560–2575. doi:10.1210/jc.2009-2575
- Coviello AD, Kaplan B, Lakshman KM, Chen T, Singh AB, Bhasin S. Effects of graded doses of testosterone on erythropoiesis in healthy young and older men. J Clin Endocrinol Metab 2008; 93(3):914–919. doi:10.1210/jc.2007-1692
- Ohlander SJ, Varghese B, Pastuszak AW. Erythrocytosis following testosterone therapy. Sex Med Rev 2018; 6(1):77–85. doi:10.1016/j.sxmr.2017.04.001
- Jones SD Jr, Dukovac T, Sangkum P, Yafi FA, Hellstrom WJ. Erythrocytosis and polycythemia secondary to testosterone replacement therapy in the aging male. Sex Med Rev 2015; 3(2):101–112. doi:10.1002/smrj.43
- Bachman E, Travison TG, Basaria S, et al. Testosterone induces erythrocytosis via increased erythropoietin and suppressed hepcidin: evidence for a new erythropoietin/hemoglobin set point. J Gerontol A Biol Sci Med Sci 2014; 69(6):725–735. doi:10.1093/gerona/glt154
- Roy CN, Snyder PJ, Stephens-Shields AJ, et al. Association of testosterone levels with anemia in older men: a controlled clinical trial. JAMA Intern Med 2017; 177(4):480–490. doi:10.1001/jamainternmed.2016.9540
- Klap J, Schmid M, Loughlin KR. The relationship between total testosterone levels and prostate cancer: a review of the continuing controversy. J Urol 2015; 193(2):403–413. doi:10.1016/j.juro.2014.07.123
- Gilbert SM, Cavallo CB, Kahane H, Lowe FC. Evidence suggesting PSA cutpoint of 2.5 ng/mL for prompting prostate biopsy: review of 36,316 biopsies. Urology 2005; 65(3):549–553. doi:10.1016/j.urology.2004.10.064
- Araujo AB, Dixon JM, Suarez EA, Murad MH, Guey LT, Wittert GA. Clinical review: Endogenous testosterone and mortality in men: a systematic review and meta-analysis. J Clin Endocrinol Metab 2011; 96(10):3007–3019. doi:10.1210/jc.2011-1137
- Xu L, Freeman G, Cowling BJ, Schooling CM. Testosterone therapy and cardiovascular events among men: a systematic review and meta-analysis of placebo-controlled randomized trials. BMC Med 2013; 11:108. doi:10.1186/1741-7015-11-108
- Araujo AB, Esche GR, Kupelian V, et al. Prevalence of symptomatic androgen deficiency in men. J Clin Endocrinol Metab 2007; 92(11):4241–4247. doi:10.1210/jc.2007-1245
- Wu FCW, Tajar A, Beynon JM, et al; EMAS Group. Identification of late-onset hypogonadism in middle-aged and elderly men. N Engl J Med 2010; 363(2):123–135. doi:10.1056/NEJMoa0911101
- Arver S, Lehtihet M. Current guidelines for the diagnosis of testosterone deficiency. Front Horm Res 2009; 37:5–20. doi:10.1159/000175839
- Narula HS, Carlson HE. Gynaecomastia—pathophysiology, diagnosis and treatment. Nat Rev Endocrinol 2014; 10(11):684–698. doi:10.1038/nrendo.2014.139
- Bhasin S, Brito JP, Cunningham GR, et al. Testosterone therapy in men with hypogonadism: an Endocrine Society clinical practice guideline. J Clin Endocrinol Metab 2018; 103(5):1715–1744. doi:10.1210/jc.2018-00229
- Mulhall JP, Trost LW, Brannigan RE, et al. Evaluation and management of testosterone deficiency: AUA guideline. J Urol 2018; 200(2):423–432. doi:10.1016/j.juro.2018.03.115
- Balasubramanian V, Naing S. Hypogonadism in chronic obstructive pulmonary disease: incidence and effects. Curr Opin Pulm Med 2012; 18(2):112–117. doi:10.1097/MCP.0b013e32834feb37
- Atlantis E, Fahey P, Cochrane B, Wittert G, Smith S. Endogenous testosterone level and testosterone supplementation therapy in chronic obstructive pulmonary disease (COPD): a systematic review and meta-analysis. BMJ Open 2013; 3(8)pii:e003127. doi:10.1136/bmjopen-2013-003127
- Bawor M, Bami H, Dennis BB, et al. Testosterone suppression in opioid users: a systematic review and meta-analysis. Drug Alcohol Depend 2015; 149:1–9. doi:10.1016/j.drugalcdep.2015.01.038
- Tan WS, Ng CJ, Khoo EM, Low WY, Tan HM. The triad of erectile dysfunction, testosterone deficiency syndrome and metabolic syndrome: findings from a multi-ethnic Asian men study (The Subang Men's Health Study). Aging Male 2011; 14(4):231–236. doi:10.3109/13685538.2011.597463
- Goldman AL, Bhasin S, Wu FCW, Krishna M, Matsumoto AM, Jasuja R. A reappraisal of testosterone’s binding in circulation: physiological and clinical implications. Endocr Rev 2017; 38(4):302–324. doi:10.1210/er.2017-00025
- Antonio L, Wu FC, O’Neill TW, et al; European Male Ageing Study Study Group. Low free testosterone is associated with hypogonadal signs and symptoms in men with normal total testosterone. J Clin Endocrinol Metab 2016; 101(7):2647–2657. doi:10.1210/jc.2015-4106
- Liu F, Shen X, Wang R, et al. Association of central obesity with sex hormone binding globulin: a cross-sectional study of 1166 Chinese men. Open Med (Wars) 2018; 13:196–202. doi:10.1515/med-2018-0030
- Vermeulen A, Verdonck L, Kaufman JM. A critical evaluation of simple methods for the estimation of free testosterone in serum. J Clin Endocrinol Metab 1999; 84(10):3666–3672. doi:10.1210/jcem.84.10.6079
- Halmenschlager G, Rhoden EL, Riedner CE. Calculated free testosterone and radioimmunoassay free testosterone as a predictor of subnormal levels of total testosterone. Int Urol Nephrol 2012; 44(3):673–681. doi:10.1007/s11255-011-0066-z
- Brambilla DJ, Matsumoto AM, Araujo AB, McKinlay JB. The effect of diurnal variation on clinical measurement of serum testosterone and other sex hormone levels in men. J Clin Endocrinol Metab 2009; 94(3):907–913. doi:10.1210/jc.2008-1902
- Lehtihet M, Arver S, Bartuseviciene I, Pousette Å. S-testosterone decrease after a mixed meal in healthy men independent of SHBG and gonadotrophin levels. Andrologia 2012; 44(6):405–410. doi:10.1111/j.1439-0272.2012.01296.x
- Brambilla DJ, O’Donnell AB, Matsumoto AM, McKinlay JB. Intraindividual variation in levels of serum testosterone and other reproductive and adrenal hormones in men. Clin Endocrinol (Oxf) 2007; 67(6):853–862. doi:10.1111/j.1365-2265.2007.02976.x
- Manzoor SM, Sattar A, Hashim R, et al. Serum inhibin B as a diagnostic marker of male infertility. J Ayub Med Coll Abbottabad 2012; 24(3–4):113–116. pmid:24669628
- Kolb BA, Stanczyk FZ, Sokol RZ. Serum inhibin B levels in males with gonadal dysfunction. Fertil Steril 2000; 74(2):234–238. pmid:10927037
- Trambas CM, Sikaris KA, Lu ZX. More on biotin treatment mimicking Graves’ disease. N Engl J Med 2016; 375(17):1698. doi:10.1056/NEJMc1611875
- Li F, Yue H, Yamaguchi K, et al. Effect of surgical repair on testosterone production in infertile men with varicocele: a meta-analysis. Int J Urol 2012; 19(2):149–154. doi:10.1111/j.1442-2042.2011.02890.x
- Crosnoe-Shipley LE, Elkelany OO, Rahnema CD, Kim ED. Treatment of hypogonadotropic male hypogonadism: case-based scenarios. World J Nephrol 2015; 4(2):245–253. doi:10.5527/wjn.v4.i2.245
- Majzoub A, Sabanegh E Jr. Testosterone replacement in the infertile man. Transl Androl Urol 2016; 5(6):859–865. doi:10.21037/tau.2016.08.03
- Majumdar A, Mangal NS. Hyperprolactinemia. J Hum Reprod Sci 2013; 6(3):168–175. doi:10.4103/0974-1208.121400
- El Osta R, Grandpre N, Monnin N, Hubert J, Koscinski I. Hypogonadotropic hypogonadism in men with hereditary hemochromatosis. Basic Clin Androl 2017; 27:13. doi:10.1186/s12610-017-0057-8
- Dhindsa S, Miller MG, McWhirter CL, et al. Testosterone concentrations in diabetic and nondiabetic obese men. Diabetes Care 2010; 33(6):1186–1192. doi:10.2337/dc09-1649
- Jasuja GK, Bhasin S, Reisman JI, et al. Who gets testosterone? Patient characteristics associated with testosterone prescribing in the Veteran Affairs system: a cross-sectional study. J Gen Intern Med 2017; 32(3):304–311. doi:10.1007/s11606-016-3940-7
- Kaplan SA, Lee JY, O’Neill EA, Meehan AG, Kusek JW. Prevalence of low testosterone and its relationship to body mass index in older men with lower urinary tract symptoms associated with benign prostatic hyperplasia. Aging Male 2013; 16(4):169–172. doi:10.3109/13685538.2013.844786
- Lee HK, Lee JK, Cho B. The role of androgen in the adipose tissue of males. World J Mens Health 2013; 31(2):136–140. doi:10.5534/wjmh.2013.31.2.136
- Raven G, De Jong FH, Kaufman JM, De Ronde W. In men, peripheral estradiol levels directly reflect the action of estrogens at the hypothalamo-pituitary level to inhibit gonadotropin secretion. J Clin Endocrinol Metab 2006; 91(9):3324–3328. doi:10.1210/jc.2006-0462
- Hofny ER, Ali ME, Abdel-Hafez HZ, et al. Semen parameters and hormonal profile in obese fertile and infertile males. Fertil Steril 2010; 94(2):581–584. doi:10.1016/j.fertnstert.2009.03.085
- Isidori AM, Caprio M, Strollo F, et al. Leptin and androgens in male obesity: evidence for leptin contribution to reduced androgen levels. J Clin Endocrinol Metab 1999; 84(10):3673–3680. doi:10.1210/jcem.84.10.6082
- El-Wakkad A, Hassan NM, Sibaii H, El-Zayat SR. Proinflammatory, anti-inflammatory cytokines and adiponkines in students with central obesity. Cytokine 2013; 61(2):682–687. doi:10.1016/j.cyto.2012.11.010
- Maggio M, Basaria S, Ceda GP, et al. The relationship between testosterone and molecular markers of inflammation in older men. J Endocrinol Invest 2005; 28(suppl proceedings 11):116–119. pmid:16760639
- de Lorenzo A, Noce A, Moriconi E, et al. MOSH syndrome (male obesity secondary hypogonadism): clinical assessment and possible therapeutic approaches. Nutrients 2018; 10(4)pii:E474. doi:10.3390/nu10040474
- Escobar-Morreale HF, Santacruz E, Luque-Ramírez M, Botella Carretero JI. Prevalence of ‘obesity-associated gonadal dysfunction’ in severely obese men and women and its resolution after bariatric surgery: a systematic review and meta-analysis. Hum Reprod Update 2017; 23(4):390–408. doi:10.1093/humupd/dmx012
- Lo EM, Rodriguez KM, Pastuszak AW, Khera M. Alternatives to testosterone therapy: a review. Sex Med Rev 2018; 6(1):106–113. doi:10.1016/j.sxmr.2017.09.004
- Soares AH, Horie NC, Chiang LAP, et al. Effects of clomiphene citrate on male obesity-associated hypogonadism: a randomized, double-blind, placebo-controlled study. Int J Obes (Lond) 2018; 42(5):953–963. doi:10.1038/s41366-018-0105-2
- Chua ME, Escusa KG, Luna S, Tapia LC, Dofitas B, Morales M. Revisiting oestrogen antagonists (clomiphene or tamoxifen) as medical empiric therapy for idiopathic male infertility: a meta-analysis. Andrology 2013; 1(5):749–757. doi:10.1111/j.2047-2927.2013.00107.x
- Westaby D, Ogle SJ, Paradinas FJ, Randell JB, Murray-Lyon IM. Liver damage from long-term methyltestosterone. Lancet 1977; 2(8032):262–263. pmid:69876
- Fernández-Balsells MM, Murad MH, Lane M, et al. Clinical review 1: Adverse effects of testosterone therapy in adult men: a systematic review and meta-analysis. J Clin Endocrinol Metab 2010; 95(6):2560–2575. doi:10.1210/jc.2009-2575
- Coviello AD, Kaplan B, Lakshman KM, Chen T, Singh AB, Bhasin S. Effects of graded doses of testosterone on erythropoiesis in healthy young and older men. J Clin Endocrinol Metab 2008; 93(3):914–919. doi:10.1210/jc.2007-1692
- Ohlander SJ, Varghese B, Pastuszak AW. Erythrocytosis following testosterone therapy. Sex Med Rev 2018; 6(1):77–85. doi:10.1016/j.sxmr.2017.04.001
- Jones SD Jr, Dukovac T, Sangkum P, Yafi FA, Hellstrom WJ. Erythrocytosis and polycythemia secondary to testosterone replacement therapy in the aging male. Sex Med Rev 2015; 3(2):101–112. doi:10.1002/smrj.43
- Bachman E, Travison TG, Basaria S, et al. Testosterone induces erythrocytosis via increased erythropoietin and suppressed hepcidin: evidence for a new erythropoietin/hemoglobin set point. J Gerontol A Biol Sci Med Sci 2014; 69(6):725–735. doi:10.1093/gerona/glt154
- Roy CN, Snyder PJ, Stephens-Shields AJ, et al. Association of testosterone levels with anemia in older men: a controlled clinical trial. JAMA Intern Med 2017; 177(4):480–490. doi:10.1001/jamainternmed.2016.9540
- Klap J, Schmid M, Loughlin KR. The relationship between total testosterone levels and prostate cancer: a review of the continuing controversy. J Urol 2015; 193(2):403–413. doi:10.1016/j.juro.2014.07.123
- Gilbert SM, Cavallo CB, Kahane H, Lowe FC. Evidence suggesting PSA cutpoint of 2.5 ng/mL for prompting prostate biopsy: review of 36,316 biopsies. Urology 2005; 65(3):549–553. doi:10.1016/j.urology.2004.10.064
- Araujo AB, Dixon JM, Suarez EA, Murad MH, Guey LT, Wittert GA. Clinical review: Endogenous testosterone and mortality in men: a systematic review and meta-analysis. J Clin Endocrinol Metab 2011; 96(10):3007–3019. doi:10.1210/jc.2011-1137
- Xu L, Freeman G, Cowling BJ, Schooling CM. Testosterone therapy and cardiovascular events among men: a systematic review and meta-analysis of placebo-controlled randomized trials. BMC Med 2013; 11:108. doi:10.1186/1741-7015-11-108
In reply: Postsurgical hypoparathyroidism is not primary hypoparathyroidism
In Reply: We thank Dr. Parmar and appreciate his important comments.
Regarding the difference between primary and secondary hypoparathyroidism, the definition varies among investigators. Some define primary hypoparathyroidism as a condition characterized by primary absence or deficiency of parathyroid hormone (PTH), which results in hypocalcemia and which can be congenital or acquired, including postsurgical hypoparathyroidism.1–4 In principle, this is similar to the classification of disorders affecting other endocrine glands as primary and secondary. For example, primary hypothyroidism refers to a state of low thyroid hormones resulting from impairment or loss of function of the thyroid gland itself, such as in Hashimoto thyroiditis, radioactive iodine therapy, or thyroidectomy, among others.5 We adopted this definition in our article. In contrast, secondary hypoparathyroidism is characterized by low PTH secretion in response to certain conditions that cause hypercalcemia. Non-PTH-mediated hypercalcemia is a more common term used to describe this state of secondary hypoparathyroidism.
Other investigators restrict the term “primary hypoparathyroidism” to nonacquired (congenital or hereditary) etiologies, while applying the term “secondary hypoparathyroidism” to acquired etiologies.6
Concerning the association between diabetes mellitus and hypomagnesemia, we agree that diabetes does not need to be uncontrolled to cause hypomagnesemia. However, the patient described in our article presented with severe hypomagnesemia (serum level 0.6 mg/dL), which is not commonly associated with diabetes. Most cases of hypomagnesemia in patients with type 2 diabetes mellitus are mild and asymptomatic, whereas severe manifestations including seizures, cardiac arrhythmias, and acute tetany are rarely encountered in clinical practice.7 Furthermore, numerous studies have shown a negative correlation between serum magnesium level and glycemic control.7–11 A recent study reported that plasma triglyceride and glucose levels are the main determinants of the plasma magnesium concentration in patients with type 2 diabetes.12
Our patient’s diabetes was uncontrolled, as evidenced by her hemoglobin A1c level of 9.7% and her random serum glucose level of 224 mg/dL. Therefore, it is more likely that “uncontrolled diabetes mellitus” (in addition to diuretic use) was the cause of her symptomatic severe hypomagnesemia rather than controlled diabetes mellitus.
- Mendes EM, Meireles-Brandão L, Meira C, Morais N, Ribeiro C, Guerra D. Primary hypoparathyroidism presenting as basal ganglia calcification secondary to extreme hypocalcemia. Clin Pract 2018; 8(1):1007. doi:10.4081/cp.2018.1007
- Vadiveloo T, Donnan PT, Leese GP. A population-based study of the epidemiology of chronic hypoparathyroidism. J Bone Miner Res 2018; 33(3):478-485. doi:10.1002/jbmr.3329
- Hendy GN, Cole DEC, Bastepe M. Hypoparathyroidism and pseudohypoparathyroidism. In: De Groot LJ, Chrousos G, Dungan K, et al, eds. Endotext [Internet], South Dartmouth (MA): MDText.com, Inc.; 2017. www.ncbi.nlm.nih.gov/books/NBK279165. Accessed August 20, 2018.
- Rosa RG, Barros AJ, de Lima AR, et al. Mood disorder as a manifestation of primary hypoparathyroidism: a case report. J Med Case Rep 2014; 8:326. doi:10.1186/1752-1947-8-326
- Almandoz JP, Gharib H. Hypothyroidism: etiology, diagnosis, and management. Med Clin North Am 2012; 96(2):203–221. doi:10.1016/j.mcna.2012.01.005
- Fouda UM, Fouda RM, Ammar HM, Salem M, Darouti ME. Impetigo herpetiformis during the puerperium triggered by secondary hypoparathyroidism: a case report. Cases J 2009; 2:9338. doi:10.1186/1757-1626-2-9338
- Tosiello L. Hypomagnesemia and diabetes mellitus. A review of clinical implications. Arch Intern Med 1996; 156(11):1143–1148. pmid: 8639008
- Pham PC, Pham PM, Pham PA, et al. Lower serum magnesium levels are associated with more rapid decline of renal function in patients with diabetes mellitus type 2. Clin Nephrol 2005; 63(6):429–436. pmid:15960144
- Tong GM, Rude RK. Magnesium deficiency in critical illness. J Intensive Care Med 2005; 20(1):3–17. doi:10.1177/0885066604271539
- Resnick LM, Altura BT, Gupta RK, Laragh JH, Alderman MH, Altura BM. Intracellular and extracellular magnesium depletion in type 2 (non-insulin-independent) diabetes mellitus. Diabetologia 1993; 36(8):767–770. pmid:8405745
- Pun KK, Ho PW. Subclinical hyponatremia, hyperkalemia and hypomagnesemia in patients with poorly controlled diabetes mellitus. Diabetes Res Clin Pract 1989; 7(3)163–167. pmid: 2605984
- Kurstjens S, de Baaij JH, Bouras H, Bindels RJ, Tack CJ, Hoenderop JG. Determinants of hypomagnesemia in patients with type 2 diabetes mellitus. Eur J Endocrinol 2017; 176(1):11–19. doi:10.1530/EJE-16-0517
In Reply: We thank Dr. Parmar and appreciate his important comments.
Regarding the difference between primary and secondary hypoparathyroidism, the definition varies among investigators. Some define primary hypoparathyroidism as a condition characterized by primary absence or deficiency of parathyroid hormone (PTH), which results in hypocalcemia and which can be congenital or acquired, including postsurgical hypoparathyroidism.1–4 In principle, this is similar to the classification of disorders affecting other endocrine glands as primary and secondary. For example, primary hypothyroidism refers to a state of low thyroid hormones resulting from impairment or loss of function of the thyroid gland itself, such as in Hashimoto thyroiditis, radioactive iodine therapy, or thyroidectomy, among others.5 We adopted this definition in our article. In contrast, secondary hypoparathyroidism is characterized by low PTH secretion in response to certain conditions that cause hypercalcemia. Non-PTH-mediated hypercalcemia is a more common term used to describe this state of secondary hypoparathyroidism.
Other investigators restrict the term “primary hypoparathyroidism” to nonacquired (congenital or hereditary) etiologies, while applying the term “secondary hypoparathyroidism” to acquired etiologies.6
Concerning the association between diabetes mellitus and hypomagnesemia, we agree that diabetes does not need to be uncontrolled to cause hypomagnesemia. However, the patient described in our article presented with severe hypomagnesemia (serum level 0.6 mg/dL), which is not commonly associated with diabetes. Most cases of hypomagnesemia in patients with type 2 diabetes mellitus are mild and asymptomatic, whereas severe manifestations including seizures, cardiac arrhythmias, and acute tetany are rarely encountered in clinical practice.7 Furthermore, numerous studies have shown a negative correlation between serum magnesium level and glycemic control.7–11 A recent study reported that plasma triglyceride and glucose levels are the main determinants of the plasma magnesium concentration in patients with type 2 diabetes.12
Our patient’s diabetes was uncontrolled, as evidenced by her hemoglobin A1c level of 9.7% and her random serum glucose level of 224 mg/dL. Therefore, it is more likely that “uncontrolled diabetes mellitus” (in addition to diuretic use) was the cause of her symptomatic severe hypomagnesemia rather than controlled diabetes mellitus.
In Reply: We thank Dr. Parmar and appreciate his important comments.
Regarding the difference between primary and secondary hypoparathyroidism, the definition varies among investigators. Some define primary hypoparathyroidism as a condition characterized by primary absence or deficiency of parathyroid hormone (PTH), which results in hypocalcemia and which can be congenital or acquired, including postsurgical hypoparathyroidism.1–4 In principle, this is similar to the classification of disorders affecting other endocrine glands as primary and secondary. For example, primary hypothyroidism refers to a state of low thyroid hormones resulting from impairment or loss of function of the thyroid gland itself, such as in Hashimoto thyroiditis, radioactive iodine therapy, or thyroidectomy, among others.5 We adopted this definition in our article. In contrast, secondary hypoparathyroidism is characterized by low PTH secretion in response to certain conditions that cause hypercalcemia. Non-PTH-mediated hypercalcemia is a more common term used to describe this state of secondary hypoparathyroidism.
Other investigators restrict the term “primary hypoparathyroidism” to nonacquired (congenital or hereditary) etiologies, while applying the term “secondary hypoparathyroidism” to acquired etiologies.6
Concerning the association between diabetes mellitus and hypomagnesemia, we agree that diabetes does not need to be uncontrolled to cause hypomagnesemia. However, the patient described in our article presented with severe hypomagnesemia (serum level 0.6 mg/dL), which is not commonly associated with diabetes. Most cases of hypomagnesemia in patients with type 2 diabetes mellitus are mild and asymptomatic, whereas severe manifestations including seizures, cardiac arrhythmias, and acute tetany are rarely encountered in clinical practice.7 Furthermore, numerous studies have shown a negative correlation between serum magnesium level and glycemic control.7–11 A recent study reported that plasma triglyceride and glucose levels are the main determinants of the plasma magnesium concentration in patients with type 2 diabetes.12
Our patient’s diabetes was uncontrolled, as evidenced by her hemoglobin A1c level of 9.7% and her random serum glucose level of 224 mg/dL. Therefore, it is more likely that “uncontrolled diabetes mellitus” (in addition to diuretic use) was the cause of her symptomatic severe hypomagnesemia rather than controlled diabetes mellitus.
- Mendes EM, Meireles-Brandão L, Meira C, Morais N, Ribeiro C, Guerra D. Primary hypoparathyroidism presenting as basal ganglia calcification secondary to extreme hypocalcemia. Clin Pract 2018; 8(1):1007. doi:10.4081/cp.2018.1007
- Vadiveloo T, Donnan PT, Leese GP. A population-based study of the epidemiology of chronic hypoparathyroidism. J Bone Miner Res 2018; 33(3):478-485. doi:10.1002/jbmr.3329
- Hendy GN, Cole DEC, Bastepe M. Hypoparathyroidism and pseudohypoparathyroidism. In: De Groot LJ, Chrousos G, Dungan K, et al, eds. Endotext [Internet], South Dartmouth (MA): MDText.com, Inc.; 2017. www.ncbi.nlm.nih.gov/books/NBK279165. Accessed August 20, 2018.
- Rosa RG, Barros AJ, de Lima AR, et al. Mood disorder as a manifestation of primary hypoparathyroidism: a case report. J Med Case Rep 2014; 8:326. doi:10.1186/1752-1947-8-326
- Almandoz JP, Gharib H. Hypothyroidism: etiology, diagnosis, and management. Med Clin North Am 2012; 96(2):203–221. doi:10.1016/j.mcna.2012.01.005
- Fouda UM, Fouda RM, Ammar HM, Salem M, Darouti ME. Impetigo herpetiformis during the puerperium triggered by secondary hypoparathyroidism: a case report. Cases J 2009; 2:9338. doi:10.1186/1757-1626-2-9338
- Tosiello L. Hypomagnesemia and diabetes mellitus. A review of clinical implications. Arch Intern Med 1996; 156(11):1143–1148. pmid: 8639008
- Pham PC, Pham PM, Pham PA, et al. Lower serum magnesium levels are associated with more rapid decline of renal function in patients with diabetes mellitus type 2. Clin Nephrol 2005; 63(6):429–436. pmid:15960144
- Tong GM, Rude RK. Magnesium deficiency in critical illness. J Intensive Care Med 2005; 20(1):3–17. doi:10.1177/0885066604271539
- Resnick LM, Altura BT, Gupta RK, Laragh JH, Alderman MH, Altura BM. Intracellular and extracellular magnesium depletion in type 2 (non-insulin-independent) diabetes mellitus. Diabetologia 1993; 36(8):767–770. pmid:8405745
- Pun KK, Ho PW. Subclinical hyponatremia, hyperkalemia and hypomagnesemia in patients with poorly controlled diabetes mellitus. Diabetes Res Clin Pract 1989; 7(3)163–167. pmid: 2605984
- Kurstjens S, de Baaij JH, Bouras H, Bindels RJ, Tack CJ, Hoenderop JG. Determinants of hypomagnesemia in patients with type 2 diabetes mellitus. Eur J Endocrinol 2017; 176(1):11–19. doi:10.1530/EJE-16-0517
- Mendes EM, Meireles-Brandão L, Meira C, Morais N, Ribeiro C, Guerra D. Primary hypoparathyroidism presenting as basal ganglia calcification secondary to extreme hypocalcemia. Clin Pract 2018; 8(1):1007. doi:10.4081/cp.2018.1007
- Vadiveloo T, Donnan PT, Leese GP. A population-based study of the epidemiology of chronic hypoparathyroidism. J Bone Miner Res 2018; 33(3):478-485. doi:10.1002/jbmr.3329
- Hendy GN, Cole DEC, Bastepe M. Hypoparathyroidism and pseudohypoparathyroidism. In: De Groot LJ, Chrousos G, Dungan K, et al, eds. Endotext [Internet], South Dartmouth (MA): MDText.com, Inc.; 2017. www.ncbi.nlm.nih.gov/books/NBK279165. Accessed August 20, 2018.
- Rosa RG, Barros AJ, de Lima AR, et al. Mood disorder as a manifestation of primary hypoparathyroidism: a case report. J Med Case Rep 2014; 8:326. doi:10.1186/1752-1947-8-326
- Almandoz JP, Gharib H. Hypothyroidism: etiology, diagnosis, and management. Med Clin North Am 2012; 96(2):203–221. doi:10.1016/j.mcna.2012.01.005
- Fouda UM, Fouda RM, Ammar HM, Salem M, Darouti ME. Impetigo herpetiformis during the puerperium triggered by secondary hypoparathyroidism: a case report. Cases J 2009; 2:9338. doi:10.1186/1757-1626-2-9338
- Tosiello L. Hypomagnesemia and diabetes mellitus. A review of clinical implications. Arch Intern Med 1996; 156(11):1143–1148. pmid: 8639008
- Pham PC, Pham PM, Pham PA, et al. Lower serum magnesium levels are associated with more rapid decline of renal function in patients with diabetes mellitus type 2. Clin Nephrol 2005; 63(6):429–436. pmid:15960144
- Tong GM, Rude RK. Magnesium deficiency in critical illness. J Intensive Care Med 2005; 20(1):3–17. doi:10.1177/0885066604271539
- Resnick LM, Altura BT, Gupta RK, Laragh JH, Alderman MH, Altura BM. Intracellular and extracellular magnesium depletion in type 2 (non-insulin-independent) diabetes mellitus. Diabetologia 1993; 36(8):767–770. pmid:8405745
- Pun KK, Ho PW. Subclinical hyponatremia, hyperkalemia and hypomagnesemia in patients with poorly controlled diabetes mellitus. Diabetes Res Clin Pract 1989; 7(3)163–167. pmid: 2605984
- Kurstjens S, de Baaij JH, Bouras H, Bindels RJ, Tack CJ, Hoenderop JG. Determinants of hypomagnesemia in patients with type 2 diabetes mellitus. Eur J Endocrinol 2017; 176(1):11–19. doi:10.1530/EJE-16-0517
A 71-year-old woman with shock and a high INR
A 71-year-old woman is brought to the emergency department by her neighbor after complaining of fatigue and light-headedness for the last 8 hours. The patient lives alone and was feeling well when she woke up this morning, but then began to feel nauseated and vomited twice.
The patient appears drowsy and confused and cannot provide any further history. Her medical records show that she was seen in the cardiology clinic 6 months ago but has not kept her appointments since then.
Her medical history includes atrial fibrillation, hypertension, type 2 diabetes mellitus, and osteoarthritis. Her medications are daily warfarin, atenolol, aspirin, candesartan, and metformin, and she takes acetaminophen as needed. She is neither a smoker nor a drug user, but she drinks alcohol occasionally. Her family history is significant for her mother’s death from breast cancer at age 55.
The neighbor confirms that the patient appeared well this morning and has not had any recent illnesses except for a minor cold last week that improved over 5 days with acetaminophen only.
INITIAL EVALUATION AND MANAGEMENT
Physical examination
On physical examination, her blood pressure is 80/40 mm Hg, respiratory rate 25 breaths per minute, oral temperature 38.3°C (100.9°F), and heart rate 130 beats per minute and irregular.
Her neck veins are flat, and her chest is clear to auscultation with normal heart sounds. Abdominal palpation elicits discomfort in the middle segments, voluntary withdrawal, and abdominal wall rigidity. Her skin feels dry and cool, with decreased turgor.
Initial treatment
The patient is given 1 L of 0.9% saline intravenously over the first hour and then is transferred to the intensive care unit, where a norepinephrine drip is started to treat her ongoing hypotension. Normal saline is continued at a rate of 500 mL per hour for the next 4 hours.
Cardiac monitoring and 12-lead electrocardiography show atrial fibrillation with a rapid ventricular response of 138 beats per minute, but electrical cardioversion is not done.
Initial laboratory tests
Of note, her international normalized ratio (INR) is 6.13, while the therapeutic range for a patient taking warfarin because of atrial fibrillation is 2.0 to 3.0.
Her blood pH is 7.34 (reference range 7.35–7.45), and her bicarbonate level is 18 mmol/L (22–26); a low pH and low bicarbonate together indicate metabolic acidosis. Her sodium level is 128 mmol/L (135–145), her chloride level is 100 mmol/L (97–107), and, as mentioned, her bicarbonate level is 18 mmol/L; therefore, her anion gap is 128 – (100 + 18) = 10 mmol/L, which is normal (≤ 10).1
Her serum creatinine level is 1.3 mg/dL (0.5–1.1), and her blood urea nitrogen level is 35 mg/dL (7–20).
Her potassium level is 5.8 mmol/L, which is consistent with hyperkalemia (reference range 3.5–5.2).
DIFFERENTIAL DIAGNOSIS
1. Which of the following is the most likely cause of this patient’s symptoms?
- Adrenal crisis
- Cardiogenic shock due to decreased cardiac contractility
- Intracranial hemorrhage
- Acute abdomen due to small bowel obstruction
- Septic shock due to bacterial toxin-induced loss of vascular tone
Our patient is presenting with shock. Given our inability to obtain a meaningful history, the differential diagnosis is broad and includes all of the above.
Adrenal crisis
The sudden onset and laboratory results that include hyperkalemia, hyponatremia, and normal anion gap metabolic acidosis raise suspicion of adrenal crisis resulting in acute mineralocorticoid and glucocorticoid insufficiency.1
The patient’s elevated serum creatinine and high blood urea nitrogen-to-creatinine ratio of 26.9 (reference range 10–20) also suggest intravascular volume contraction. Her low hemoglobin level and supratherapeutic INR, possibly due to an interaction between warfarin and acetaminophen combined with poor medical follow-up, raise suspicion of acute bilateral adrenal necrosis due to hemorrhage.
Bilateral adrenal hemorrhage is one cause of adrenal crisis resulting in bilateral adrenal necrosis. Risk factors for adrenal hemorrhage include anticoagulation therapy, underlying coagulopathy, postoperative states, and certain infections such as meningococcemia and Haemophilus influenzae infection.2–5 Nevertheless, in most cases the INR is in the therapeutic range and the patient has no bleeding elsewhere.4 Other causes of adrenal necrosis include emboli, sepsis, and blunt trauma.6,7
Other causes of adrenal crisis are listed in Table 3.
Cardiogenic shock
Intracranial hemorrhage
Intracranial hemorrhage can present with a decreased level of consciousness, but it is less likely to cause hypotension, as the cranial space is limited. If massive intracranial hemorrhage would occur, the increase in intracranial pressure would more likely cause hypertension by the Cushing reflex than hypotension.
Acute abdomen
Abdominal pain and rigidity along with fever can be presenting symptoms of both adrenal insufficiency and an acute abdomen due to intestinal obstruction.4 However, intestinal obstruction typically causes a high anion gap metabolic acidosis due to lactic acidosis, instead of the normal anion gap metabolic acidosis present in this patient.8 Moreover, her deranged electrolytes, supratherapeutic INR, and absence of previous gastroenterologic conditions make adrenal crisis a more likely diagnosis.
Septic shock
Septic shock would also cause fever and hypotension as bacterial toxins induce a pyrexic response and vasodilation. However, at such an early stage of sepsis, the patient would be expected to be warm and hyperemic, whereas this patient’s skin is cool and dry due to volume depletion secondary to adrenal insufficiency.9 Sepsis would also cause a high anion gap metabolic acidosis due to lactic acidosis, as opposed to this patient’s normal anion gap metabolic acidosis. These findings, along with the metabolic derangements and the absence of a focus of infection, make sepsis a less likely possibility.
CASE CONTINUED: CARDIOMEGALY, PERSISTENT HYPOTENSION
Blood is drawn for cultures and measurement of troponins and lactic acid, and urine samples are taken for culture and biochemical analysis. Chest radiography shows mild cardiomegaly. The patient is started empirically on vancomycin and cefepime, and her warfarin is discontinued.
Five hours after presenting to the emergency department, her blood pressure remains at 80/40 mm Hg even after receiving 3 L of normal saline intravenously.
PROMPT MANAGEMENT OF ADRENAL CRISIS
2. Which of the following is the most appropriate next step in managing this patient?
- Draw samples for serum cortisol and plasma adrenocorticotropic hormone (ACTH) levels, then give hydrocortisone 100 mg intravenously
- Perform abdominal computed tomography (CT) without contrast
- Perform transthoracic echocardiography
- Increase the norepinephrine infusion
- Immediately give fludrocortisone
First give fluids
The first step in managing a patient with suspected adrenal crisis is liberal intravenous fluid administration to replenish the depleted intravascular space. The amount and choice of fluid is empiric, but a recommendation is 1 L of normal saline or dextrose 5% in normal saline, infused quickly over the first hour and then titrated according to the patient’s fluid status.10
Measure cortisol and ACTH; start corticosteroids immediately
Immediate therapy with an appropriate stress dose of intravenous corticosteroids (eg, hydrocortisone 100 mg) is essential. However, this should be done after drawing blood for cortisol and ACTH measurements.10
Do not delay corticosteroid therapy while awaiting the results of the diagnostic tests.
Cortisol and ACTH levels are useful in assessing adrenal function in critically ill patients.11 While inappropriately low serum cortisol usually indicates adrenal insufficiency, measuring plasma ACTH is important to distinguish whether the adrenal insufficiency is primary (ie, due to dysfunction of the adrenal gland itself) or central, ie, either secondary (due to dysfunction of the pituitary gland) or tertiary (due to dysfunction of the hypothalamus). Primary adrenal insufficiency is associated with elevated plasma ACTH, whereas central (secondary or tertiary) adrenal insufficiency is associated with low or inappropriately normal plasma ACTH levels (Figure 1).
In addition, in the early phase of evolving primary adrenal insufficiency, measurement of plasma renin and aldosterone levels may be beneficial, as mineralocorticoid deficiency may predominate.10,12,13
One of the most important aims of early corticosteroid supplementation is to prevent further hyponatremia by reducing a reactive increase in antidiuretic hormone secretion caused by cortisol deficiency. Corticosteroids also help to restore normal blood pressure by increasing vascular tone, as glucocorticoid receptor activation potentiates the vasoconstrictor actions of norepinephrine, angiotensin II, and other vasoconstrictors.14,15
Which corticosteroid to use?
Which corticosteroid to use in previously undiagnosed adrenal insufficiency is controversial. The Endocrine Society10 and Japan Endocrine Society16 clinical practice guidelines recommend hydrocortisone in a 100-mg intravenous bolus followed by 200 mg over 24 hours.
The choice of hydrocortisone is justified by its superior mineralocorticoid activity.10,16 Further, hydrocortisone is preferred over dexamethasone if the patient is known to have primary adrenal insufficiency, or if the serum potassium level is higher than 6.0 mmol/L.
Some clinicians, on the other hand, recommend dexamethasone, given as a 4-mg intravenous bolus followed by 4-mg boluses every 12 hours. Their rationale is that dexamethasone, unlike hydrocortisone, does not interfere with subsequent serum cortisol assays if the patient later undergoes ACTH stimulation testing.17 Dexamethasone may also be preferred to minimize unwanted mineralocorticoid effects, such as in neurosurgical patients at risk of brain edema.
If hydrocortisone is used, ACTH stimulation testing can be done after withholding hydrocortisone for 24 hours once the patient is stable. (It should be restarted after the test if the results are abnormal.)
Other possible steps
Abdominal CT should be done in our patient to address the possibility of bilateral adrenal hemorrhage. However, it is preferable to wait until the patient is stabilized.
Echocardiography. Our patient is likely to have an element of cardiac failure, given her hypertension and cardiomegaly. However, decompensated heart failure is probably not the cause of her presentation. Thus, the first priority is to treat her adrenal crisis, and echocardiography should be deferred.
Increasing the norepinephrine infusion is unlikely to improve her blood pressure very much, as she is significantly volume-depleted. Further, low cortisol decreases the vascular response to norepinephrine.15
Mineralocorticoids such as fludrocortisone are used to treat primary adrenal insufficiency. However, they are not required during acute management of adrenal crisis, as 40 mg of hydrocortisone offers mineralocorticoid activity equivalent to 100 µg of fludrocortisone. Thus, the high doses of hydrocortisone used to treat adrenal crisis provide adequate mineralocorticoid therapy.10,18
If dexamethasone is used, its effect along with normal saline supplementation would be sufficient to replete the intravenous space and bring the sodium level back up to normal in the acute setting.
CASE RESUMED: IMPROVEMENT WITH HYDROCORTISONE
The patient’s blood is drawn for serum cortisol and plasma ACTH measurements. A 100-mg intravenous bolus of hydrocortisone is given, followed by a 50-mg bolus every 6 hours until the patient stabilizes.
Twenty-four hours later, the patient states that she has more energy, and her appetite has improved. The norepinephrine infusion is stopped 48 hours after presentation, at which time her blood pressure is 120/70 mm Hg, heart rate 85 beats per minute and irregular, and temperature 36.7°C (98.1°F). Her current laboratory values include the following:
- Serum sodium 137 mmolL
- Serum potassium 4.3 mmol/L
- Hemoglobin 9.3 g/dL
- Serum cortisol (random) 7.2 μg/dL
- Plasma ACTH 752 pg/mL (10–60 pg/mL).
ESTABLISHING THE DIAGNOSIS OF ADRENAL INSUFFICIENCY
3. Which of the following is the most appropriate test to establish the diagnosis of adrenal insufficiency?
- 7 am total serum cortisol measurement
- Random serum cortisol measurement
- 7 am salivary cortisol measurement
- 24-hour urinary free cortisol measurement
- ACTH stimulation test for cortisol
- Insulin tolerance test for cortisol
These tests also help determine the type of adrenal insufficiency (primary, secondary, or tertiary) and guide further management. Secondary adrenal insufficiency is caused by inadequate pituitary ACTH secretion and subsequent inadequate cortisol production, whereas tertiary adrenal insufficiency is caused by inadequate hypothalamic corticotropin-releasing hormone secretion and subsequent inadequate ACTH and cortisol production. The diagnosis of adrenal insufficiency relies first on demonstrating inappropriately low total serum cortisol production. Subsequently, serum ACTH helps to differentiate between primary (high ACTH) and secondary or tertiary (low or inappropriately normal ACTH) adrenal insufficiency.
Each test listed above may demonstrate a low cortisol level. However, in a nonacute setting, safety concerns (especially regarding insulin tolerance testing), poor diagnostic value, feasibility (ie, the difficulty of 24-hour tests), and poor sensitivity of 7 am cortisol make the ACTH stimulation test the most appropriate test in clinical practice to establish the diagnosis of adrenal insufficiency.
7 am serum cortisol measurement
Measuring the serum cortisol level early in the morning in the nonacute setting could be of diagnostic value, as an extremely low value (< 3–5 μg/dL) is almost 100% specific for adrenal insufficiency in the absence of concurrent exogenous steroid intake. However, the very low cutoff for this test causes poor sensitivity (about 33%), as many patients have partial adrenal insufficiency and hence have higher serum cortisol levels that may even be in the normal physiologic range.19–22
Random serum cortisol measurements
Random serum cortisol measurements are not very useful in a nonacute setting, since cortisol levels are affected by factors such as stress and hydration status. Moreover, they fluctuate during the day in a circadian rhythm.
On the other hand, random serum cortisol is a very good test to evaluate for adrenal insufficiency in the acute setting. A random value higher than 15 to 18 μg/dL is almost always associated with adequate adrenal function and generally rules out adrenal insufficiency.11,23,24
7 am salivary cortisol measurement
The same principle applies to early morning salivary cortisol. Only extremely low values (< 2.65 ng/mL) may distinguish patients with adrenal insufficiency from healthy individuals, with 97.1% sensitivity and 93.3% specificity.25
Of note, early morning salivary cortisol is not routinely measured in most clinical practices for evaluation of adrenal function. Hence, morning serum and morning salivary cortisol are useful screening tools and have meaningful results when their values are in the extremes of the spectrum, but they are not reliable as a single test, as they may overlook patients with partial adrenal insufficiency.
Urinary cortisol measurement
Urinary cortisol measurement is not used to diagnose adrenal insufficiency, as values can be normal in patients with partial adrenal insufficiency.
The ACTH stimulation test
The ACTH stimulation test involves an intramuscular or intravenous injection of cosyntropin (a synthetic analogue of ACTH fragment 1–24 that has the full activity of native ACTH) and measuring total serum cortisol at baseline, 30 minutes, and 60 minutes to assess the response of the adrenal glands.
The test can be done using a high or low dose of cosyntropin. The Endocrine Society’s 2016 guidelines recommend the high dose (250 μg) for most patients.10 The standard high-dose stimulation test can be done at any time during the day.26 If the cosyntropin is injected intravenously, any value higher than 18 to 20 μg/dL indicates normal adrenal function and excludes adrenal insufficiency.27,28 If intramuscular injection is used, any value higher than 16 to 18 μg/dL at 30 minutes post-consyntropin excludes adrenal insufficiency.29
The ACTH stimulation test may not exclude acute secondary or tertiary adrenal insufficiency.
Insulin tolerance testing
Insulin tolerance testing remains the gold standard for diagnosing adrenal insufficiency and assessing the integrity of the pituitary-adrenal axis. However, given its difficulty to perform, safety concerns, and the availability of other reliable tests, its use in clinical practice is limited. It is nonetheless useful in assessing patients with recent onset of ACTH deficiency.30,31
CASE RESUMED: PATIENT DISCHARGED, LOST TO FOLLOW-UP
Abdominal CT without contrast is done and demonstrates bilateral adrenal hemorrhage. Thus, the patient is diagnosed with primary acute adrenal insufficiency due to adrenal necrosis.
She is started on oral hydrocortisone and fludrocortisone after intravenous hydrocortisone is discontinued. She is counseled about adhering to medications, wearing a medical alert bracelet, giving herself emergency cortisol injections, taking higher doses of hydrocortisone if she is ill, and monitoring her INR. She is discharged home after her symptoms resolve.
The patient does not keep her scheduled appointment and is lost to follow-up. She returns 2 years later complaining of fatigue and feeling unwell. She admits that she stopped taking hydrocortisone 1 year ago after reading an online article about corticosteroid side effects. She has continued to take fludrocortisone.
MINERALOCORTICOID VS CORTICOSTEROID DEFICIENCY
4. Which of the following is least likely to be present in this patient at this time?
- Intravascular volume depletion
- Hyponatremia
- Skin hyperpigmentation
- Normokalemia
- Elevated serum ACTH level
Intravascular volume depletion
Intravascular volume depletion is the least likely to be present. This is because intravascular volume depletion is mainly secondary to mineralocorticoid deficiency rather than corticosteroid deficiency, which is not present in this patient, as she is compliant with her mineralocorticoid replacement therapy.32,33 However, even with sufficient mineralocorticoid replacement, mild hypotension may be present in this patient due to corticosteroid deficiency-induced loss of vascular tone.
Hyponatremia
Hyponatremia in adrenal insufficiency is not due only to mineralocorticoid deficiency. Patients with secondary or tertiary adrenal insufficiency may also exhibit hyponatremia.34 ACTH deficiency in such patients is not expected to cause mineralocorticoid deficiency, as ACTH has only a minor role in aldosterone production.
It has been proposed that hyponatremia in secondary adrenal insufficiency is due to cortisol deficiency resulting in an increase of antidiuretic hormone secretion.35,36 The mechanisms for increased antidiuretic hormone include cortisol deficiency resulting in an increased corticotropin-releasing hormone level, which acts as an antidiuretic hormone secretagogue,37,38 and cortisol directly suppressing antidiuretic hormone secretion.39
In our patient, volume expansion and hyponatremia are expected due to increased antidiuretic hormone secretion as a result of corticosteroid insufficiency.
Hyperpigmentation
Hyperpigmentation of the skin is present only in long-standing primary adrenal insufficiency. This is due to chronic cortisol deficiency causing an increased secretion of pro-opiomelanocortin, a prohormone that is cleaved into ACTH, melanocyte-stimulating hormone, and other hormones. Melanocyte-stimulating hormone causes skin hyperpigmentation due to increased melanin synthesis.40 The hyperpigmentation is seen in sun-exposed areas, pressure areas, palmar creases, nipples, and mucous membranes.
This patient has long-standing corticosteroid deficiency due to noncompliance and primary adrenal insufficiency, and as a result she is expected to have elevated serum ACTH and hyperpigmentation.
Normokalemia
Mineralocorticoid deficiency results in hyperkalemia and metabolic acidosis by impairing renal excretion of potassium and acid.41 This patient is compliant with her mineralocorticoid replacement regimen; thus, potassium levels and pH are expected to be normal.
TAKE-HOME POINTS
- Suspect adrenal crisis in any patient who presents with shock.
- Acute abdomen or unexplained fever could be among the manifestations.
- Initial management requires liberal normal saline intravenous fluid administration to replete the intravascular space.
- Draw blood samples for serum chemistry, cortisol, and ACTH, followed immediately by intravenous hydrocortisone supplementation.
- In critically ill patients, evaluate adrenal function with random serum cortisol; in a nonacute setting use the ACTH stimulation test.
- Chronic management of primary adrenal insufficiency requires corticosteroid and mineralocorticoid therapy.
- Mani S, Rutecki GW. A patient with altered mental status and an acid-base disturbance. Cleve Clin J Med 2017; 84(1):27–34. doi:10.3949/ccjm.84a.16042
- Almiani M, Gorthi J, Subbiah S, Firoz M. Quiz page November 2012: an unusual case of acute hyponatremia and normal anion gap metabolic acidosis. Am J Kidney Dis 2012; 60(5):xxxiii–xxxvi. doi:10.1053/j.ajkd.2012.05.026
- Migeon CJ, Kenny FM, Hung W, Voorhess ML. Study of adrenal function in children with meningitis. Pediatrics 1967; 40(2):163–183.
- Rao RH, Vagnucci AH, Amico JA. Bilateral massive adrenal hemorrhage: early recognition and treatment. Ann Intern Med 1989; 110(3):227–235.
- Shimizu S, Tahara Y, Atsumi T, et al. Waterhouse-Friderichsen syndrome caused by invasive Haemophilus influenzae type B infection in a previously healthy young man. Anaesth Intensive Care 2010; 38(1):214–215.
- Castaldo ET, Guillamondegui OD, Greco JA 3rd, Feurer ID, Miller RS, Morris JA Jr. Are adrenal injuries predictive of adrenal insufficiency in patients sustaining blunt trauma? Am Surg 2008; 74(3):262–266.
- Xarli VP, Steele AA, Davis PJ, Buescher ES, Rios CN, Garcia-Bunuel R. Adrenal hemorrhage in the adult. Medicine (Baltimore) 1978; 57(3):211–221.
- Takeuchi K, Tsuzuki Y, Ando T, et al. Clinical studies of strangulating small bowel obstruction. Am Surg 2004; 70(1):40–44.
- MacKenzie IM. The haemodynamics of human septic shock. Anaesthesia 2001; 56(2):130–144.
- Bornstein SR, Allolio B, Arlt W, et al. Diagnosis and treatment of primary adrenal insufficiency: an Endocrine Society clinical practice guideline. J Clin Endocrinol Metab 2016; 101(2):364–389. doi:10.1210/jc.2015-1710
- Hamrahian AH, Fleseriu M; AACE Adrenal Scientific Committee. Evaluation and management of adrenal insufficiency in critically ill patients: disease state review. Endocr Pract 2017; 23(6):716–725. doi:10.4158/EP161720.RA
- Saenger P, Levine LS, Irvine WJ, et al. Progressive adrenal failure in polyglandular autoimmune disease. J Clin Endocrinol Metab 1982; 54(4):863–867.
- Coco G, Dal Pra C, Presotto F, et al. Estimated risk for developing autoimmune Addison's disease in patients with adrenal cortex autoantibodies. J Clin Endocrinol Metab 2006; 91(5):1637–1645. doi:10.1210/jc.2005-0860
- Ullian ME. The role of corticosteroids in the regulation of vascular tone. Cardiovasc Res 1999; 41(1):55–64.
- Yang S, Zhang L. Glucocorticoids and vascular reactivity. Curr Vasc Pharmacol 2004; 2(1):1–12.
- Yanase T, Tajima T, Katabami T, et al. Diagnosis and treatment of adrenal insufficiency including adrenal crisis: a Japan Endocrine Society clinical practice guideline [Opinion]. Endocr J 2016; 63(9):765–784. doi:10.1507/endocrj.EJ16-0242
- Taylor RL, Grebe SK, Singh RJ. Quantitative, highly sensitive liquid chromatography-tandem mass spectrometry method for detection of synthetic corticosteroids. Clin Chem 2004; 50(10):2345–2352. doi:10.1373/clinchem.2004.033605
- Goldfien A, Laidlaw JC, Haydar NA, Renold AE, Thorn GW. Fluorohydrocortisone and chlorohydrocortisone, highly potent derivatives of compound F. N Engl J Med 1955; 252(11):415–421. doi:10.1056/NEJM195503172521101
- Jenkins D, Forsham PH, Laidlaw JC, Reddy WJ, Thorn GW. Use of ACTH in the diagnosis of adrenal cortical insufficiency. Am J Med 1955; 18(1):3–14.
- Hägg E, Asplund K, Lithner F. Value of basal plasma cortisol assays in the assessment of pituitary-adrenal insufficiency. Clin Endocrinol (Oxf) 1987; 26(2):221–226.
- Deutschbein T, Unger N, Mann K, Petersenn S. Diagnosis of secondary adrenal insufficiency: unstimulated early morning cortisol in saliva and serum in comparison with the insulin tolerance test. Horm Metab Res 2009; 41(4):834–839. doi:10.1055/s-0029-1225630
- Erturk E, Jaffe CA, Barkan AL. Evaluation of the integrity of the hypothalamic-pituitary-adrenal axis by insulin hypoglycemia test. J Clin Endocrinol Metab 1998; 83(7):2350–2354.
- Cooper MS, Stewart PM. Corticosteroid insufficiency in acutely ill patients. N Engl J Med 2003; 348(8):727–734. doi:10.1056/NEJMra020529
- Dellinger RP, Levy MM, Rhodes A, et al; Surviving Sepsis Campaign Guidelines Committee including the Pediatric Subgroup. Surviving Sepsis Campaign: international guidelines for management of severe sepsis and septic shock, 2012. Intensive Care Med 2013; 39(2):165–228. doi:10.1007/s00134-012-2769-8
- Ceccato F, Barbot M, Zilio M, et al. Performance of salivary cortisol in the diagnosis of Cushing's syndrome, adrenal incidentaloma, and adrenal insufficiency. Eur J Endocrinol 2013; 169(1):31–36. doi:10.1530/EJE-13-0159
- Dickstein G, Shechner C, Nicholson WE, et al. Adrenocorticotropin stimulation test: effects of basal cortisol level, time of day, and suggested new sensitive low dose test. J Clin Endocrinol Metab 1991; 72(4):773–778. doi:10.1210/jcem-72-4-773
- May ME, Carey RM. Rapid adrenocorticotropic hormone test in practice. Retrospective review. Am J Med 1985; 79(6):679–884.
- Speckart PF, Nicoloff JT, Bethune JE. Screening for adrenocortical insufficiency with cosyntropin (synthetic ACTH). Arch Intern Med 1971; 128(5):761–763.
- Peechakara S, Bena J, Clarke NJ, et al. Total and free cortisol levels during 1 μg, 25 μg, and 250 μg cosyntropin stimulation tests compared to insulin tolerance test: results of a randomized, prospective, pilot study. Endocrine 2017; 57(3):388–393. doi:10.1007/s12020-017-1371-9
- Finucane FM, Liew A, Thornton E, Rogers B, Tormey W, Agha A. Clinical insights into the safety and utility of the insulin tolerance test (ITT) in the assessment of the hypothalamo-pituitary-adrenal axis. Clin Endocrinol (Oxf) 2008; 69(4):603–607. doi:10.1111/j.1365-2265.2008.03240.x
- Lindholm J, Kehlet H. Re-evaluation of the clinical value of the 30 min ACTH test in assessing the hypothalamic-pituitary-adrenocortical function. Clin Endocrinol (Oxf) 1987; 26(1):53–59.
- Charmandari E, Nicolaides NC, Chrousos GP. Adrenal insufficiency. Lancet 2014; 383(9935):2152–2167. doi:10.1016/S0140-6736(13)61684-0
- Burke CW. Adrenocortical insufficiency. Clin Endocrinol Metab 1985; 14(4):947–976.
- Jessani N, Jehangir W, Behman D, Yousif A, Spiler IJ. Secondary adrenal insufficiency: an overlooked cause of hyponatremia. J Clin Med Res 2015; 7(4):286–288. doi:10.14740/jocmr2041w
- Oelkers W. Hyponatremia and inappropriate secretion of vasopressin (antidiuretic hormone) in patients with hypopituitarism. N Engl J Med 1989; 321(8):492–496. doi:10.1056/NEJM198908243210802
- Ishikawa S, Schrier RW. Effect of arginine vasopressin antagonist on renal water excretion in glucocorticoid and mineralocorticoid deficient rats. Kidney Int 1982; 22(6):587–593.
- Wolfson B, Manning RW, Davis LG, Arentzen R, Baldino F Jr. Co-localization of corticotropin releasing factor and vasopressin mRNA in neurones after adrenalectomy. Nature 1985; 315(6014):59–61.
- Kalogeras KT, Nieman LK, Friedman TC, et al. Inferior petrosal sinus sampling in healthy subjects reveals a unilateral corticotropin-releasing hormone-induced arginine vasopressin release associated with ipsilateral adrenocorticotropin secretion. J Clin Invest 1996; 97:2045–2050.
- Kovacs KJ, Foldes A, Sawchenko PE. Glucocorticoid negative feedback selectively targets vasopressin transcription in parvocellular neurosecretory neurons. J Neurosci 2000; 20:3843–3852.
- Sarkar SB, Sarkar S, Ghosh S, Bandyopadhyay S. Addison's disease. Contemp Clin Dent 2012; 3(4):484–486. doi:10.4103/0976-237X.107450
- Szylman P, Better OS, Chaimowitz C, Rosler A. Role of hyperkalemia in the metabolic acidosis of isolated hypoaldosteronism. N Engl J Med 1976; 294(7):361–365. doi:10.1056/NEJM197602122940703
A 71-year-old woman is brought to the emergency department by her neighbor after complaining of fatigue and light-headedness for the last 8 hours. The patient lives alone and was feeling well when she woke up this morning, but then began to feel nauseated and vomited twice.
The patient appears drowsy and confused and cannot provide any further history. Her medical records show that she was seen in the cardiology clinic 6 months ago but has not kept her appointments since then.
Her medical history includes atrial fibrillation, hypertension, type 2 diabetes mellitus, and osteoarthritis. Her medications are daily warfarin, atenolol, aspirin, candesartan, and metformin, and she takes acetaminophen as needed. She is neither a smoker nor a drug user, but she drinks alcohol occasionally. Her family history is significant for her mother’s death from breast cancer at age 55.
The neighbor confirms that the patient appeared well this morning and has not had any recent illnesses except for a minor cold last week that improved over 5 days with acetaminophen only.
INITIAL EVALUATION AND MANAGEMENT
Physical examination
On physical examination, her blood pressure is 80/40 mm Hg, respiratory rate 25 breaths per minute, oral temperature 38.3°C (100.9°F), and heart rate 130 beats per minute and irregular.
Her neck veins are flat, and her chest is clear to auscultation with normal heart sounds. Abdominal palpation elicits discomfort in the middle segments, voluntary withdrawal, and abdominal wall rigidity. Her skin feels dry and cool, with decreased turgor.
Initial treatment
The patient is given 1 L of 0.9% saline intravenously over the first hour and then is transferred to the intensive care unit, where a norepinephrine drip is started to treat her ongoing hypotension. Normal saline is continued at a rate of 500 mL per hour for the next 4 hours.
Cardiac monitoring and 12-lead electrocardiography show atrial fibrillation with a rapid ventricular response of 138 beats per minute, but electrical cardioversion is not done.
Initial laboratory tests
Of note, her international normalized ratio (INR) is 6.13, while the therapeutic range for a patient taking warfarin because of atrial fibrillation is 2.0 to 3.0.
Her blood pH is 7.34 (reference range 7.35–7.45), and her bicarbonate level is 18 mmol/L (22–26); a low pH and low bicarbonate together indicate metabolic acidosis. Her sodium level is 128 mmol/L (135–145), her chloride level is 100 mmol/L (97–107), and, as mentioned, her bicarbonate level is 18 mmol/L; therefore, her anion gap is 128 – (100 + 18) = 10 mmol/L, which is normal (≤ 10).1
Her serum creatinine level is 1.3 mg/dL (0.5–1.1), and her blood urea nitrogen level is 35 mg/dL (7–20).
Her potassium level is 5.8 mmol/L, which is consistent with hyperkalemia (reference range 3.5–5.2).
DIFFERENTIAL DIAGNOSIS
1. Which of the following is the most likely cause of this patient’s symptoms?
- Adrenal crisis
- Cardiogenic shock due to decreased cardiac contractility
- Intracranial hemorrhage
- Acute abdomen due to small bowel obstruction
- Septic shock due to bacterial toxin-induced loss of vascular tone
Our patient is presenting with shock. Given our inability to obtain a meaningful history, the differential diagnosis is broad and includes all of the above.
Adrenal crisis
The sudden onset and laboratory results that include hyperkalemia, hyponatremia, and normal anion gap metabolic acidosis raise suspicion of adrenal crisis resulting in acute mineralocorticoid and glucocorticoid insufficiency.1
The patient’s elevated serum creatinine and high blood urea nitrogen-to-creatinine ratio of 26.9 (reference range 10–20) also suggest intravascular volume contraction. Her low hemoglobin level and supratherapeutic INR, possibly due to an interaction between warfarin and acetaminophen combined with poor medical follow-up, raise suspicion of acute bilateral adrenal necrosis due to hemorrhage.
Bilateral adrenal hemorrhage is one cause of adrenal crisis resulting in bilateral adrenal necrosis. Risk factors for adrenal hemorrhage include anticoagulation therapy, underlying coagulopathy, postoperative states, and certain infections such as meningococcemia and Haemophilus influenzae infection.2–5 Nevertheless, in most cases the INR is in the therapeutic range and the patient has no bleeding elsewhere.4 Other causes of adrenal necrosis include emboli, sepsis, and blunt trauma.6,7
Other causes of adrenal crisis are listed in Table 3.
Cardiogenic shock
Intracranial hemorrhage
Intracranial hemorrhage can present with a decreased level of consciousness, but it is less likely to cause hypotension, as the cranial space is limited. If massive intracranial hemorrhage would occur, the increase in intracranial pressure would more likely cause hypertension by the Cushing reflex than hypotension.
Acute abdomen
Abdominal pain and rigidity along with fever can be presenting symptoms of both adrenal insufficiency and an acute abdomen due to intestinal obstruction.4 However, intestinal obstruction typically causes a high anion gap metabolic acidosis due to lactic acidosis, instead of the normal anion gap metabolic acidosis present in this patient.8 Moreover, her deranged electrolytes, supratherapeutic INR, and absence of previous gastroenterologic conditions make adrenal crisis a more likely diagnosis.
Septic shock
Septic shock would also cause fever and hypotension as bacterial toxins induce a pyrexic response and vasodilation. However, at such an early stage of sepsis, the patient would be expected to be warm and hyperemic, whereas this patient’s skin is cool and dry due to volume depletion secondary to adrenal insufficiency.9 Sepsis would also cause a high anion gap metabolic acidosis due to lactic acidosis, as opposed to this patient’s normal anion gap metabolic acidosis. These findings, along with the metabolic derangements and the absence of a focus of infection, make sepsis a less likely possibility.
CASE CONTINUED: CARDIOMEGALY, PERSISTENT HYPOTENSION
Blood is drawn for cultures and measurement of troponins and lactic acid, and urine samples are taken for culture and biochemical analysis. Chest radiography shows mild cardiomegaly. The patient is started empirically on vancomycin and cefepime, and her warfarin is discontinued.
Five hours after presenting to the emergency department, her blood pressure remains at 80/40 mm Hg even after receiving 3 L of normal saline intravenously.
PROMPT MANAGEMENT OF ADRENAL CRISIS
2. Which of the following is the most appropriate next step in managing this patient?
- Draw samples for serum cortisol and plasma adrenocorticotropic hormone (ACTH) levels, then give hydrocortisone 100 mg intravenously
- Perform abdominal computed tomography (CT) without contrast
- Perform transthoracic echocardiography
- Increase the norepinephrine infusion
- Immediately give fludrocortisone
First give fluids
The first step in managing a patient with suspected adrenal crisis is liberal intravenous fluid administration to replenish the depleted intravascular space. The amount and choice of fluid is empiric, but a recommendation is 1 L of normal saline or dextrose 5% in normal saline, infused quickly over the first hour and then titrated according to the patient’s fluid status.10
Measure cortisol and ACTH; start corticosteroids immediately
Immediate therapy with an appropriate stress dose of intravenous corticosteroids (eg, hydrocortisone 100 mg) is essential. However, this should be done after drawing blood for cortisol and ACTH measurements.10
Do not delay corticosteroid therapy while awaiting the results of the diagnostic tests.
Cortisol and ACTH levels are useful in assessing adrenal function in critically ill patients.11 While inappropriately low serum cortisol usually indicates adrenal insufficiency, measuring plasma ACTH is important to distinguish whether the adrenal insufficiency is primary (ie, due to dysfunction of the adrenal gland itself) or central, ie, either secondary (due to dysfunction of the pituitary gland) or tertiary (due to dysfunction of the hypothalamus). Primary adrenal insufficiency is associated with elevated plasma ACTH, whereas central (secondary or tertiary) adrenal insufficiency is associated with low or inappropriately normal plasma ACTH levels (Figure 1).
In addition, in the early phase of evolving primary adrenal insufficiency, measurement of plasma renin and aldosterone levels may be beneficial, as mineralocorticoid deficiency may predominate.10,12,13
One of the most important aims of early corticosteroid supplementation is to prevent further hyponatremia by reducing a reactive increase in antidiuretic hormone secretion caused by cortisol deficiency. Corticosteroids also help to restore normal blood pressure by increasing vascular tone, as glucocorticoid receptor activation potentiates the vasoconstrictor actions of norepinephrine, angiotensin II, and other vasoconstrictors.14,15
Which corticosteroid to use?
Which corticosteroid to use in previously undiagnosed adrenal insufficiency is controversial. The Endocrine Society10 and Japan Endocrine Society16 clinical practice guidelines recommend hydrocortisone in a 100-mg intravenous bolus followed by 200 mg over 24 hours.
The choice of hydrocortisone is justified by its superior mineralocorticoid activity.10,16 Further, hydrocortisone is preferred over dexamethasone if the patient is known to have primary adrenal insufficiency, or if the serum potassium level is higher than 6.0 mmol/L.
Some clinicians, on the other hand, recommend dexamethasone, given as a 4-mg intravenous bolus followed by 4-mg boluses every 12 hours. Their rationale is that dexamethasone, unlike hydrocortisone, does not interfere with subsequent serum cortisol assays if the patient later undergoes ACTH stimulation testing.17 Dexamethasone may also be preferred to minimize unwanted mineralocorticoid effects, such as in neurosurgical patients at risk of brain edema.
If hydrocortisone is used, ACTH stimulation testing can be done after withholding hydrocortisone for 24 hours once the patient is stable. (It should be restarted after the test if the results are abnormal.)
Other possible steps
Abdominal CT should be done in our patient to address the possibility of bilateral adrenal hemorrhage. However, it is preferable to wait until the patient is stabilized.
Echocardiography. Our patient is likely to have an element of cardiac failure, given her hypertension and cardiomegaly. However, decompensated heart failure is probably not the cause of her presentation. Thus, the first priority is to treat her adrenal crisis, and echocardiography should be deferred.
Increasing the norepinephrine infusion is unlikely to improve her blood pressure very much, as she is significantly volume-depleted. Further, low cortisol decreases the vascular response to norepinephrine.15
Mineralocorticoids such as fludrocortisone are used to treat primary adrenal insufficiency. However, they are not required during acute management of adrenal crisis, as 40 mg of hydrocortisone offers mineralocorticoid activity equivalent to 100 µg of fludrocortisone. Thus, the high doses of hydrocortisone used to treat adrenal crisis provide adequate mineralocorticoid therapy.10,18
If dexamethasone is used, its effect along with normal saline supplementation would be sufficient to replete the intravenous space and bring the sodium level back up to normal in the acute setting.
CASE RESUMED: IMPROVEMENT WITH HYDROCORTISONE
The patient’s blood is drawn for serum cortisol and plasma ACTH measurements. A 100-mg intravenous bolus of hydrocortisone is given, followed by a 50-mg bolus every 6 hours until the patient stabilizes.
Twenty-four hours later, the patient states that she has more energy, and her appetite has improved. The norepinephrine infusion is stopped 48 hours after presentation, at which time her blood pressure is 120/70 mm Hg, heart rate 85 beats per minute and irregular, and temperature 36.7°C (98.1°F). Her current laboratory values include the following:
- Serum sodium 137 mmolL
- Serum potassium 4.3 mmol/L
- Hemoglobin 9.3 g/dL
- Serum cortisol (random) 7.2 μg/dL
- Plasma ACTH 752 pg/mL (10–60 pg/mL).
ESTABLISHING THE DIAGNOSIS OF ADRENAL INSUFFICIENCY
3. Which of the following is the most appropriate test to establish the diagnosis of adrenal insufficiency?
- 7 am total serum cortisol measurement
- Random serum cortisol measurement
- 7 am salivary cortisol measurement
- 24-hour urinary free cortisol measurement
- ACTH stimulation test for cortisol
- Insulin tolerance test for cortisol
These tests also help determine the type of adrenal insufficiency (primary, secondary, or tertiary) and guide further management. Secondary adrenal insufficiency is caused by inadequate pituitary ACTH secretion and subsequent inadequate cortisol production, whereas tertiary adrenal insufficiency is caused by inadequate hypothalamic corticotropin-releasing hormone secretion and subsequent inadequate ACTH and cortisol production. The diagnosis of adrenal insufficiency relies first on demonstrating inappropriately low total serum cortisol production. Subsequently, serum ACTH helps to differentiate between primary (high ACTH) and secondary or tertiary (low or inappropriately normal ACTH) adrenal insufficiency.
Each test listed above may demonstrate a low cortisol level. However, in a nonacute setting, safety concerns (especially regarding insulin tolerance testing), poor diagnostic value, feasibility (ie, the difficulty of 24-hour tests), and poor sensitivity of 7 am cortisol make the ACTH stimulation test the most appropriate test in clinical practice to establish the diagnosis of adrenal insufficiency.
7 am serum cortisol measurement
Measuring the serum cortisol level early in the morning in the nonacute setting could be of diagnostic value, as an extremely low value (< 3–5 μg/dL) is almost 100% specific for adrenal insufficiency in the absence of concurrent exogenous steroid intake. However, the very low cutoff for this test causes poor sensitivity (about 33%), as many patients have partial adrenal insufficiency and hence have higher serum cortisol levels that may even be in the normal physiologic range.19–22
Random serum cortisol measurements
Random serum cortisol measurements are not very useful in a nonacute setting, since cortisol levels are affected by factors such as stress and hydration status. Moreover, they fluctuate during the day in a circadian rhythm.
On the other hand, random serum cortisol is a very good test to evaluate for adrenal insufficiency in the acute setting. A random value higher than 15 to 18 μg/dL is almost always associated with adequate adrenal function and generally rules out adrenal insufficiency.11,23,24
7 am salivary cortisol measurement
The same principle applies to early morning salivary cortisol. Only extremely low values (< 2.65 ng/mL) may distinguish patients with adrenal insufficiency from healthy individuals, with 97.1% sensitivity and 93.3% specificity.25
Of note, early morning salivary cortisol is not routinely measured in most clinical practices for evaluation of adrenal function. Hence, morning serum and morning salivary cortisol are useful screening tools and have meaningful results when their values are in the extremes of the spectrum, but they are not reliable as a single test, as they may overlook patients with partial adrenal insufficiency.
Urinary cortisol measurement
Urinary cortisol measurement is not used to diagnose adrenal insufficiency, as values can be normal in patients with partial adrenal insufficiency.
The ACTH stimulation test
The ACTH stimulation test involves an intramuscular or intravenous injection of cosyntropin (a synthetic analogue of ACTH fragment 1–24 that has the full activity of native ACTH) and measuring total serum cortisol at baseline, 30 minutes, and 60 minutes to assess the response of the adrenal glands.
The test can be done using a high or low dose of cosyntropin. The Endocrine Society’s 2016 guidelines recommend the high dose (250 μg) for most patients.10 The standard high-dose stimulation test can be done at any time during the day.26 If the cosyntropin is injected intravenously, any value higher than 18 to 20 μg/dL indicates normal adrenal function and excludes adrenal insufficiency.27,28 If intramuscular injection is used, any value higher than 16 to 18 μg/dL at 30 minutes post-consyntropin excludes adrenal insufficiency.29
The ACTH stimulation test may not exclude acute secondary or tertiary adrenal insufficiency.
Insulin tolerance testing
Insulin tolerance testing remains the gold standard for diagnosing adrenal insufficiency and assessing the integrity of the pituitary-adrenal axis. However, given its difficulty to perform, safety concerns, and the availability of other reliable tests, its use in clinical practice is limited. It is nonetheless useful in assessing patients with recent onset of ACTH deficiency.30,31
CASE RESUMED: PATIENT DISCHARGED, LOST TO FOLLOW-UP
Abdominal CT without contrast is done and demonstrates bilateral adrenal hemorrhage. Thus, the patient is diagnosed with primary acute adrenal insufficiency due to adrenal necrosis.
She is started on oral hydrocortisone and fludrocortisone after intravenous hydrocortisone is discontinued. She is counseled about adhering to medications, wearing a medical alert bracelet, giving herself emergency cortisol injections, taking higher doses of hydrocortisone if she is ill, and monitoring her INR. She is discharged home after her symptoms resolve.
The patient does not keep her scheduled appointment and is lost to follow-up. She returns 2 years later complaining of fatigue and feeling unwell. She admits that she stopped taking hydrocortisone 1 year ago after reading an online article about corticosteroid side effects. She has continued to take fludrocortisone.
MINERALOCORTICOID VS CORTICOSTEROID DEFICIENCY
4. Which of the following is least likely to be present in this patient at this time?
- Intravascular volume depletion
- Hyponatremia
- Skin hyperpigmentation
- Normokalemia
- Elevated serum ACTH level
Intravascular volume depletion
Intravascular volume depletion is the least likely to be present. This is because intravascular volume depletion is mainly secondary to mineralocorticoid deficiency rather than corticosteroid deficiency, which is not present in this patient, as she is compliant with her mineralocorticoid replacement therapy.32,33 However, even with sufficient mineralocorticoid replacement, mild hypotension may be present in this patient due to corticosteroid deficiency-induced loss of vascular tone.
Hyponatremia
Hyponatremia in adrenal insufficiency is not due only to mineralocorticoid deficiency. Patients with secondary or tertiary adrenal insufficiency may also exhibit hyponatremia.34 ACTH deficiency in such patients is not expected to cause mineralocorticoid deficiency, as ACTH has only a minor role in aldosterone production.
It has been proposed that hyponatremia in secondary adrenal insufficiency is due to cortisol deficiency resulting in an increase of antidiuretic hormone secretion.35,36 The mechanisms for increased antidiuretic hormone include cortisol deficiency resulting in an increased corticotropin-releasing hormone level, which acts as an antidiuretic hormone secretagogue,37,38 and cortisol directly suppressing antidiuretic hormone secretion.39
In our patient, volume expansion and hyponatremia are expected due to increased antidiuretic hormone secretion as a result of corticosteroid insufficiency.
Hyperpigmentation
Hyperpigmentation of the skin is present only in long-standing primary adrenal insufficiency. This is due to chronic cortisol deficiency causing an increased secretion of pro-opiomelanocortin, a prohormone that is cleaved into ACTH, melanocyte-stimulating hormone, and other hormones. Melanocyte-stimulating hormone causes skin hyperpigmentation due to increased melanin synthesis.40 The hyperpigmentation is seen in sun-exposed areas, pressure areas, palmar creases, nipples, and mucous membranes.
This patient has long-standing corticosteroid deficiency due to noncompliance and primary adrenal insufficiency, and as a result she is expected to have elevated serum ACTH and hyperpigmentation.
Normokalemia
Mineralocorticoid deficiency results in hyperkalemia and metabolic acidosis by impairing renal excretion of potassium and acid.41 This patient is compliant with her mineralocorticoid replacement regimen; thus, potassium levels and pH are expected to be normal.
TAKE-HOME POINTS
- Suspect adrenal crisis in any patient who presents with shock.
- Acute abdomen or unexplained fever could be among the manifestations.
- Initial management requires liberal normal saline intravenous fluid administration to replete the intravascular space.
- Draw blood samples for serum chemistry, cortisol, and ACTH, followed immediately by intravenous hydrocortisone supplementation.
- In critically ill patients, evaluate adrenal function with random serum cortisol; in a nonacute setting use the ACTH stimulation test.
- Chronic management of primary adrenal insufficiency requires corticosteroid and mineralocorticoid therapy.
A 71-year-old woman is brought to the emergency department by her neighbor after complaining of fatigue and light-headedness for the last 8 hours. The patient lives alone and was feeling well when she woke up this morning, but then began to feel nauseated and vomited twice.
The patient appears drowsy and confused and cannot provide any further history. Her medical records show that she was seen in the cardiology clinic 6 months ago but has not kept her appointments since then.
Her medical history includes atrial fibrillation, hypertension, type 2 diabetes mellitus, and osteoarthritis. Her medications are daily warfarin, atenolol, aspirin, candesartan, and metformin, and she takes acetaminophen as needed. She is neither a smoker nor a drug user, but she drinks alcohol occasionally. Her family history is significant for her mother’s death from breast cancer at age 55.
The neighbor confirms that the patient appeared well this morning and has not had any recent illnesses except for a minor cold last week that improved over 5 days with acetaminophen only.
INITIAL EVALUATION AND MANAGEMENT
Physical examination
On physical examination, her blood pressure is 80/40 mm Hg, respiratory rate 25 breaths per minute, oral temperature 38.3°C (100.9°F), and heart rate 130 beats per minute and irregular.
Her neck veins are flat, and her chest is clear to auscultation with normal heart sounds. Abdominal palpation elicits discomfort in the middle segments, voluntary withdrawal, and abdominal wall rigidity. Her skin feels dry and cool, with decreased turgor.
Initial treatment
The patient is given 1 L of 0.9% saline intravenously over the first hour and then is transferred to the intensive care unit, where a norepinephrine drip is started to treat her ongoing hypotension. Normal saline is continued at a rate of 500 mL per hour for the next 4 hours.
Cardiac monitoring and 12-lead electrocardiography show atrial fibrillation with a rapid ventricular response of 138 beats per minute, but electrical cardioversion is not done.
Initial laboratory tests
Of note, her international normalized ratio (INR) is 6.13, while the therapeutic range for a patient taking warfarin because of atrial fibrillation is 2.0 to 3.0.
Her blood pH is 7.34 (reference range 7.35–7.45), and her bicarbonate level is 18 mmol/L (22–26); a low pH and low bicarbonate together indicate metabolic acidosis. Her sodium level is 128 mmol/L (135–145), her chloride level is 100 mmol/L (97–107), and, as mentioned, her bicarbonate level is 18 mmol/L; therefore, her anion gap is 128 – (100 + 18) = 10 mmol/L, which is normal (≤ 10).1
Her serum creatinine level is 1.3 mg/dL (0.5–1.1), and her blood urea nitrogen level is 35 mg/dL (7–20).
Her potassium level is 5.8 mmol/L, which is consistent with hyperkalemia (reference range 3.5–5.2).
DIFFERENTIAL DIAGNOSIS
1. Which of the following is the most likely cause of this patient’s symptoms?
- Adrenal crisis
- Cardiogenic shock due to decreased cardiac contractility
- Intracranial hemorrhage
- Acute abdomen due to small bowel obstruction
- Septic shock due to bacterial toxin-induced loss of vascular tone
Our patient is presenting with shock. Given our inability to obtain a meaningful history, the differential diagnosis is broad and includes all of the above.
Adrenal crisis
The sudden onset and laboratory results that include hyperkalemia, hyponatremia, and normal anion gap metabolic acidosis raise suspicion of adrenal crisis resulting in acute mineralocorticoid and glucocorticoid insufficiency.1
The patient’s elevated serum creatinine and high blood urea nitrogen-to-creatinine ratio of 26.9 (reference range 10–20) also suggest intravascular volume contraction. Her low hemoglobin level and supratherapeutic INR, possibly due to an interaction between warfarin and acetaminophen combined with poor medical follow-up, raise suspicion of acute bilateral adrenal necrosis due to hemorrhage.
Bilateral adrenal hemorrhage is one cause of adrenal crisis resulting in bilateral adrenal necrosis. Risk factors for adrenal hemorrhage include anticoagulation therapy, underlying coagulopathy, postoperative states, and certain infections such as meningococcemia and Haemophilus influenzae infection.2–5 Nevertheless, in most cases the INR is in the therapeutic range and the patient has no bleeding elsewhere.4 Other causes of adrenal necrosis include emboli, sepsis, and blunt trauma.6,7
Other causes of adrenal crisis are listed in Table 3.
Cardiogenic shock
Intracranial hemorrhage
Intracranial hemorrhage can present with a decreased level of consciousness, but it is less likely to cause hypotension, as the cranial space is limited. If massive intracranial hemorrhage would occur, the increase in intracranial pressure would more likely cause hypertension by the Cushing reflex than hypotension.
Acute abdomen
Abdominal pain and rigidity along with fever can be presenting symptoms of both adrenal insufficiency and an acute abdomen due to intestinal obstruction.4 However, intestinal obstruction typically causes a high anion gap metabolic acidosis due to lactic acidosis, instead of the normal anion gap metabolic acidosis present in this patient.8 Moreover, her deranged electrolytes, supratherapeutic INR, and absence of previous gastroenterologic conditions make adrenal crisis a more likely diagnosis.
Septic shock
Septic shock would also cause fever and hypotension as bacterial toxins induce a pyrexic response and vasodilation. However, at such an early stage of sepsis, the patient would be expected to be warm and hyperemic, whereas this patient’s skin is cool and dry due to volume depletion secondary to adrenal insufficiency.9 Sepsis would also cause a high anion gap metabolic acidosis due to lactic acidosis, as opposed to this patient’s normal anion gap metabolic acidosis. These findings, along with the metabolic derangements and the absence of a focus of infection, make sepsis a less likely possibility.
CASE CONTINUED: CARDIOMEGALY, PERSISTENT HYPOTENSION
Blood is drawn for cultures and measurement of troponins and lactic acid, and urine samples are taken for culture and biochemical analysis. Chest radiography shows mild cardiomegaly. The patient is started empirically on vancomycin and cefepime, and her warfarin is discontinued.
Five hours after presenting to the emergency department, her blood pressure remains at 80/40 mm Hg even after receiving 3 L of normal saline intravenously.
PROMPT MANAGEMENT OF ADRENAL CRISIS
2. Which of the following is the most appropriate next step in managing this patient?
- Draw samples for serum cortisol and plasma adrenocorticotropic hormone (ACTH) levels, then give hydrocortisone 100 mg intravenously
- Perform abdominal computed tomography (CT) without contrast
- Perform transthoracic echocardiography
- Increase the norepinephrine infusion
- Immediately give fludrocortisone
First give fluids
The first step in managing a patient with suspected adrenal crisis is liberal intravenous fluid administration to replenish the depleted intravascular space. The amount and choice of fluid is empiric, but a recommendation is 1 L of normal saline or dextrose 5% in normal saline, infused quickly over the first hour and then titrated according to the patient’s fluid status.10
Measure cortisol and ACTH; start corticosteroids immediately
Immediate therapy with an appropriate stress dose of intravenous corticosteroids (eg, hydrocortisone 100 mg) is essential. However, this should be done after drawing blood for cortisol and ACTH measurements.10
Do not delay corticosteroid therapy while awaiting the results of the diagnostic tests.
Cortisol and ACTH levels are useful in assessing adrenal function in critically ill patients.11 While inappropriately low serum cortisol usually indicates adrenal insufficiency, measuring plasma ACTH is important to distinguish whether the adrenal insufficiency is primary (ie, due to dysfunction of the adrenal gland itself) or central, ie, either secondary (due to dysfunction of the pituitary gland) or tertiary (due to dysfunction of the hypothalamus). Primary adrenal insufficiency is associated with elevated plasma ACTH, whereas central (secondary or tertiary) adrenal insufficiency is associated with low or inappropriately normal plasma ACTH levels (Figure 1).
In addition, in the early phase of evolving primary adrenal insufficiency, measurement of plasma renin and aldosterone levels may be beneficial, as mineralocorticoid deficiency may predominate.10,12,13
One of the most important aims of early corticosteroid supplementation is to prevent further hyponatremia by reducing a reactive increase in antidiuretic hormone secretion caused by cortisol deficiency. Corticosteroids also help to restore normal blood pressure by increasing vascular tone, as glucocorticoid receptor activation potentiates the vasoconstrictor actions of norepinephrine, angiotensin II, and other vasoconstrictors.14,15
Which corticosteroid to use?
Which corticosteroid to use in previously undiagnosed adrenal insufficiency is controversial. The Endocrine Society10 and Japan Endocrine Society16 clinical practice guidelines recommend hydrocortisone in a 100-mg intravenous bolus followed by 200 mg over 24 hours.
The choice of hydrocortisone is justified by its superior mineralocorticoid activity.10,16 Further, hydrocortisone is preferred over dexamethasone if the patient is known to have primary adrenal insufficiency, or if the serum potassium level is higher than 6.0 mmol/L.
Some clinicians, on the other hand, recommend dexamethasone, given as a 4-mg intravenous bolus followed by 4-mg boluses every 12 hours. Their rationale is that dexamethasone, unlike hydrocortisone, does not interfere with subsequent serum cortisol assays if the patient later undergoes ACTH stimulation testing.17 Dexamethasone may also be preferred to minimize unwanted mineralocorticoid effects, such as in neurosurgical patients at risk of brain edema.
If hydrocortisone is used, ACTH stimulation testing can be done after withholding hydrocortisone for 24 hours once the patient is stable. (It should be restarted after the test if the results are abnormal.)
Other possible steps
Abdominal CT should be done in our patient to address the possibility of bilateral adrenal hemorrhage. However, it is preferable to wait until the patient is stabilized.
Echocardiography. Our patient is likely to have an element of cardiac failure, given her hypertension and cardiomegaly. However, decompensated heart failure is probably not the cause of her presentation. Thus, the first priority is to treat her adrenal crisis, and echocardiography should be deferred.
Increasing the norepinephrine infusion is unlikely to improve her blood pressure very much, as she is significantly volume-depleted. Further, low cortisol decreases the vascular response to norepinephrine.15
Mineralocorticoids such as fludrocortisone are used to treat primary adrenal insufficiency. However, they are not required during acute management of adrenal crisis, as 40 mg of hydrocortisone offers mineralocorticoid activity equivalent to 100 µg of fludrocortisone. Thus, the high doses of hydrocortisone used to treat adrenal crisis provide adequate mineralocorticoid therapy.10,18
If dexamethasone is used, its effect along with normal saline supplementation would be sufficient to replete the intravenous space and bring the sodium level back up to normal in the acute setting.
CASE RESUMED: IMPROVEMENT WITH HYDROCORTISONE
The patient’s blood is drawn for serum cortisol and plasma ACTH measurements. A 100-mg intravenous bolus of hydrocortisone is given, followed by a 50-mg bolus every 6 hours until the patient stabilizes.
Twenty-four hours later, the patient states that she has more energy, and her appetite has improved. The norepinephrine infusion is stopped 48 hours after presentation, at which time her blood pressure is 120/70 mm Hg, heart rate 85 beats per minute and irregular, and temperature 36.7°C (98.1°F). Her current laboratory values include the following:
- Serum sodium 137 mmolL
- Serum potassium 4.3 mmol/L
- Hemoglobin 9.3 g/dL
- Serum cortisol (random) 7.2 μg/dL
- Plasma ACTH 752 pg/mL (10–60 pg/mL).
ESTABLISHING THE DIAGNOSIS OF ADRENAL INSUFFICIENCY
3. Which of the following is the most appropriate test to establish the diagnosis of adrenal insufficiency?
- 7 am total serum cortisol measurement
- Random serum cortisol measurement
- 7 am salivary cortisol measurement
- 24-hour urinary free cortisol measurement
- ACTH stimulation test for cortisol
- Insulin tolerance test for cortisol
These tests also help determine the type of adrenal insufficiency (primary, secondary, or tertiary) and guide further management. Secondary adrenal insufficiency is caused by inadequate pituitary ACTH secretion and subsequent inadequate cortisol production, whereas tertiary adrenal insufficiency is caused by inadequate hypothalamic corticotropin-releasing hormone secretion and subsequent inadequate ACTH and cortisol production. The diagnosis of adrenal insufficiency relies first on demonstrating inappropriately low total serum cortisol production. Subsequently, serum ACTH helps to differentiate between primary (high ACTH) and secondary or tertiary (low or inappropriately normal ACTH) adrenal insufficiency.
Each test listed above may demonstrate a low cortisol level. However, in a nonacute setting, safety concerns (especially regarding insulin tolerance testing), poor diagnostic value, feasibility (ie, the difficulty of 24-hour tests), and poor sensitivity of 7 am cortisol make the ACTH stimulation test the most appropriate test in clinical practice to establish the diagnosis of adrenal insufficiency.
7 am serum cortisol measurement
Measuring the serum cortisol level early in the morning in the nonacute setting could be of diagnostic value, as an extremely low value (< 3–5 μg/dL) is almost 100% specific for adrenal insufficiency in the absence of concurrent exogenous steroid intake. However, the very low cutoff for this test causes poor sensitivity (about 33%), as many patients have partial adrenal insufficiency and hence have higher serum cortisol levels that may even be in the normal physiologic range.19–22
Random serum cortisol measurements
Random serum cortisol measurements are not very useful in a nonacute setting, since cortisol levels are affected by factors such as stress and hydration status. Moreover, they fluctuate during the day in a circadian rhythm.
On the other hand, random serum cortisol is a very good test to evaluate for adrenal insufficiency in the acute setting. A random value higher than 15 to 18 μg/dL is almost always associated with adequate adrenal function and generally rules out adrenal insufficiency.11,23,24
7 am salivary cortisol measurement
The same principle applies to early morning salivary cortisol. Only extremely low values (< 2.65 ng/mL) may distinguish patients with adrenal insufficiency from healthy individuals, with 97.1% sensitivity and 93.3% specificity.25
Of note, early morning salivary cortisol is not routinely measured in most clinical practices for evaluation of adrenal function. Hence, morning serum and morning salivary cortisol are useful screening tools and have meaningful results when their values are in the extremes of the spectrum, but they are not reliable as a single test, as they may overlook patients with partial adrenal insufficiency.
Urinary cortisol measurement
Urinary cortisol measurement is not used to diagnose adrenal insufficiency, as values can be normal in patients with partial adrenal insufficiency.
The ACTH stimulation test
The ACTH stimulation test involves an intramuscular or intravenous injection of cosyntropin (a synthetic analogue of ACTH fragment 1–24 that has the full activity of native ACTH) and measuring total serum cortisol at baseline, 30 minutes, and 60 minutes to assess the response of the adrenal glands.
The test can be done using a high or low dose of cosyntropin. The Endocrine Society’s 2016 guidelines recommend the high dose (250 μg) for most patients.10 The standard high-dose stimulation test can be done at any time during the day.26 If the cosyntropin is injected intravenously, any value higher than 18 to 20 μg/dL indicates normal adrenal function and excludes adrenal insufficiency.27,28 If intramuscular injection is used, any value higher than 16 to 18 μg/dL at 30 minutes post-consyntropin excludes adrenal insufficiency.29
The ACTH stimulation test may not exclude acute secondary or tertiary adrenal insufficiency.
Insulin tolerance testing
Insulin tolerance testing remains the gold standard for diagnosing adrenal insufficiency and assessing the integrity of the pituitary-adrenal axis. However, given its difficulty to perform, safety concerns, and the availability of other reliable tests, its use in clinical practice is limited. It is nonetheless useful in assessing patients with recent onset of ACTH deficiency.30,31
CASE RESUMED: PATIENT DISCHARGED, LOST TO FOLLOW-UP
Abdominal CT without contrast is done and demonstrates bilateral adrenal hemorrhage. Thus, the patient is diagnosed with primary acute adrenal insufficiency due to adrenal necrosis.
She is started on oral hydrocortisone and fludrocortisone after intravenous hydrocortisone is discontinued. She is counseled about adhering to medications, wearing a medical alert bracelet, giving herself emergency cortisol injections, taking higher doses of hydrocortisone if she is ill, and monitoring her INR. She is discharged home after her symptoms resolve.
The patient does not keep her scheduled appointment and is lost to follow-up. She returns 2 years later complaining of fatigue and feeling unwell. She admits that she stopped taking hydrocortisone 1 year ago after reading an online article about corticosteroid side effects. She has continued to take fludrocortisone.
MINERALOCORTICOID VS CORTICOSTEROID DEFICIENCY
4. Which of the following is least likely to be present in this patient at this time?
- Intravascular volume depletion
- Hyponatremia
- Skin hyperpigmentation
- Normokalemia
- Elevated serum ACTH level
Intravascular volume depletion
Intravascular volume depletion is the least likely to be present. This is because intravascular volume depletion is mainly secondary to mineralocorticoid deficiency rather than corticosteroid deficiency, which is not present in this patient, as she is compliant with her mineralocorticoid replacement therapy.32,33 However, even with sufficient mineralocorticoid replacement, mild hypotension may be present in this patient due to corticosteroid deficiency-induced loss of vascular tone.
Hyponatremia
Hyponatremia in adrenal insufficiency is not due only to mineralocorticoid deficiency. Patients with secondary or tertiary adrenal insufficiency may also exhibit hyponatremia.34 ACTH deficiency in such patients is not expected to cause mineralocorticoid deficiency, as ACTH has only a minor role in aldosterone production.
It has been proposed that hyponatremia in secondary adrenal insufficiency is due to cortisol deficiency resulting in an increase of antidiuretic hormone secretion.35,36 The mechanisms for increased antidiuretic hormone include cortisol deficiency resulting in an increased corticotropin-releasing hormone level, which acts as an antidiuretic hormone secretagogue,37,38 and cortisol directly suppressing antidiuretic hormone secretion.39
In our patient, volume expansion and hyponatremia are expected due to increased antidiuretic hormone secretion as a result of corticosteroid insufficiency.
Hyperpigmentation
Hyperpigmentation of the skin is present only in long-standing primary adrenal insufficiency. This is due to chronic cortisol deficiency causing an increased secretion of pro-opiomelanocortin, a prohormone that is cleaved into ACTH, melanocyte-stimulating hormone, and other hormones. Melanocyte-stimulating hormone causes skin hyperpigmentation due to increased melanin synthesis.40 The hyperpigmentation is seen in sun-exposed areas, pressure areas, palmar creases, nipples, and mucous membranes.
This patient has long-standing corticosteroid deficiency due to noncompliance and primary adrenal insufficiency, and as a result she is expected to have elevated serum ACTH and hyperpigmentation.
Normokalemia
Mineralocorticoid deficiency results in hyperkalemia and metabolic acidosis by impairing renal excretion of potassium and acid.41 This patient is compliant with her mineralocorticoid replacement regimen; thus, potassium levels and pH are expected to be normal.
TAKE-HOME POINTS
- Suspect adrenal crisis in any patient who presents with shock.
- Acute abdomen or unexplained fever could be among the manifestations.
- Initial management requires liberal normal saline intravenous fluid administration to replete the intravascular space.
- Draw blood samples for serum chemistry, cortisol, and ACTH, followed immediately by intravenous hydrocortisone supplementation.
- In critically ill patients, evaluate adrenal function with random serum cortisol; in a nonacute setting use the ACTH stimulation test.
- Chronic management of primary adrenal insufficiency requires corticosteroid and mineralocorticoid therapy.
- Mani S, Rutecki GW. A patient with altered mental status and an acid-base disturbance. Cleve Clin J Med 2017; 84(1):27–34. doi:10.3949/ccjm.84a.16042
- Almiani M, Gorthi J, Subbiah S, Firoz M. Quiz page November 2012: an unusual case of acute hyponatremia and normal anion gap metabolic acidosis. Am J Kidney Dis 2012; 60(5):xxxiii–xxxvi. doi:10.1053/j.ajkd.2012.05.026
- Migeon CJ, Kenny FM, Hung W, Voorhess ML. Study of adrenal function in children with meningitis. Pediatrics 1967; 40(2):163–183.
- Rao RH, Vagnucci AH, Amico JA. Bilateral massive adrenal hemorrhage: early recognition and treatment. Ann Intern Med 1989; 110(3):227–235.
- Shimizu S, Tahara Y, Atsumi T, et al. Waterhouse-Friderichsen syndrome caused by invasive Haemophilus influenzae type B infection in a previously healthy young man. Anaesth Intensive Care 2010; 38(1):214–215.
- Castaldo ET, Guillamondegui OD, Greco JA 3rd, Feurer ID, Miller RS, Morris JA Jr. Are adrenal injuries predictive of adrenal insufficiency in patients sustaining blunt trauma? Am Surg 2008; 74(3):262–266.
- Xarli VP, Steele AA, Davis PJ, Buescher ES, Rios CN, Garcia-Bunuel R. Adrenal hemorrhage in the adult. Medicine (Baltimore) 1978; 57(3):211–221.
- Takeuchi K, Tsuzuki Y, Ando T, et al. Clinical studies of strangulating small bowel obstruction. Am Surg 2004; 70(1):40–44.
- MacKenzie IM. The haemodynamics of human septic shock. Anaesthesia 2001; 56(2):130–144.
- Bornstein SR, Allolio B, Arlt W, et al. Diagnosis and treatment of primary adrenal insufficiency: an Endocrine Society clinical practice guideline. J Clin Endocrinol Metab 2016; 101(2):364–389. doi:10.1210/jc.2015-1710
- Hamrahian AH, Fleseriu M; AACE Adrenal Scientific Committee. Evaluation and management of adrenal insufficiency in critically ill patients: disease state review. Endocr Pract 2017; 23(6):716–725. doi:10.4158/EP161720.RA
- Saenger P, Levine LS, Irvine WJ, et al. Progressive adrenal failure in polyglandular autoimmune disease. J Clin Endocrinol Metab 1982; 54(4):863–867.
- Coco G, Dal Pra C, Presotto F, et al. Estimated risk for developing autoimmune Addison's disease in patients with adrenal cortex autoantibodies. J Clin Endocrinol Metab 2006; 91(5):1637–1645. doi:10.1210/jc.2005-0860
- Ullian ME. The role of corticosteroids in the regulation of vascular tone. Cardiovasc Res 1999; 41(1):55–64.
- Yang S, Zhang L. Glucocorticoids and vascular reactivity. Curr Vasc Pharmacol 2004; 2(1):1–12.
- Yanase T, Tajima T, Katabami T, et al. Diagnosis and treatment of adrenal insufficiency including adrenal crisis: a Japan Endocrine Society clinical practice guideline [Opinion]. Endocr J 2016; 63(9):765–784. doi:10.1507/endocrj.EJ16-0242
- Taylor RL, Grebe SK, Singh RJ. Quantitative, highly sensitive liquid chromatography-tandem mass spectrometry method for detection of synthetic corticosteroids. Clin Chem 2004; 50(10):2345–2352. doi:10.1373/clinchem.2004.033605
- Goldfien A, Laidlaw JC, Haydar NA, Renold AE, Thorn GW. Fluorohydrocortisone and chlorohydrocortisone, highly potent derivatives of compound F. N Engl J Med 1955; 252(11):415–421. doi:10.1056/NEJM195503172521101
- Jenkins D, Forsham PH, Laidlaw JC, Reddy WJ, Thorn GW. Use of ACTH in the diagnosis of adrenal cortical insufficiency. Am J Med 1955; 18(1):3–14.
- Hägg E, Asplund K, Lithner F. Value of basal plasma cortisol assays in the assessment of pituitary-adrenal insufficiency. Clin Endocrinol (Oxf) 1987; 26(2):221–226.
- Deutschbein T, Unger N, Mann K, Petersenn S. Diagnosis of secondary adrenal insufficiency: unstimulated early morning cortisol in saliva and serum in comparison with the insulin tolerance test. Horm Metab Res 2009; 41(4):834–839. doi:10.1055/s-0029-1225630
- Erturk E, Jaffe CA, Barkan AL. Evaluation of the integrity of the hypothalamic-pituitary-adrenal axis by insulin hypoglycemia test. J Clin Endocrinol Metab 1998; 83(7):2350–2354.
- Cooper MS, Stewart PM. Corticosteroid insufficiency in acutely ill patients. N Engl J Med 2003; 348(8):727–734. doi:10.1056/NEJMra020529
- Dellinger RP, Levy MM, Rhodes A, et al; Surviving Sepsis Campaign Guidelines Committee including the Pediatric Subgroup. Surviving Sepsis Campaign: international guidelines for management of severe sepsis and septic shock, 2012. Intensive Care Med 2013; 39(2):165–228. doi:10.1007/s00134-012-2769-8
- Ceccato F, Barbot M, Zilio M, et al. Performance of salivary cortisol in the diagnosis of Cushing's syndrome, adrenal incidentaloma, and adrenal insufficiency. Eur J Endocrinol 2013; 169(1):31–36. doi:10.1530/EJE-13-0159
- Dickstein G, Shechner C, Nicholson WE, et al. Adrenocorticotropin stimulation test: effects of basal cortisol level, time of day, and suggested new sensitive low dose test. J Clin Endocrinol Metab 1991; 72(4):773–778. doi:10.1210/jcem-72-4-773
- May ME, Carey RM. Rapid adrenocorticotropic hormone test in practice. Retrospective review. Am J Med 1985; 79(6):679–884.
- Speckart PF, Nicoloff JT, Bethune JE. Screening for adrenocortical insufficiency with cosyntropin (synthetic ACTH). Arch Intern Med 1971; 128(5):761–763.
- Peechakara S, Bena J, Clarke NJ, et al. Total and free cortisol levels during 1 μg, 25 μg, and 250 μg cosyntropin stimulation tests compared to insulin tolerance test: results of a randomized, prospective, pilot study. Endocrine 2017; 57(3):388–393. doi:10.1007/s12020-017-1371-9
- Finucane FM, Liew A, Thornton E, Rogers B, Tormey W, Agha A. Clinical insights into the safety and utility of the insulin tolerance test (ITT) in the assessment of the hypothalamo-pituitary-adrenal axis. Clin Endocrinol (Oxf) 2008; 69(4):603–607. doi:10.1111/j.1365-2265.2008.03240.x
- Lindholm J, Kehlet H. Re-evaluation of the clinical value of the 30 min ACTH test in assessing the hypothalamic-pituitary-adrenocortical function. Clin Endocrinol (Oxf) 1987; 26(1):53–59.
- Charmandari E, Nicolaides NC, Chrousos GP. Adrenal insufficiency. Lancet 2014; 383(9935):2152–2167. doi:10.1016/S0140-6736(13)61684-0
- Burke CW. Adrenocortical insufficiency. Clin Endocrinol Metab 1985; 14(4):947–976.
- Jessani N, Jehangir W, Behman D, Yousif A, Spiler IJ. Secondary adrenal insufficiency: an overlooked cause of hyponatremia. J Clin Med Res 2015; 7(4):286–288. doi:10.14740/jocmr2041w
- Oelkers W. Hyponatremia and inappropriate secretion of vasopressin (antidiuretic hormone) in patients with hypopituitarism. N Engl J Med 1989; 321(8):492–496. doi:10.1056/NEJM198908243210802
- Ishikawa S, Schrier RW. Effect of arginine vasopressin antagonist on renal water excretion in glucocorticoid and mineralocorticoid deficient rats. Kidney Int 1982; 22(6):587–593.
- Wolfson B, Manning RW, Davis LG, Arentzen R, Baldino F Jr. Co-localization of corticotropin releasing factor and vasopressin mRNA in neurones after adrenalectomy. Nature 1985; 315(6014):59–61.
- Kalogeras KT, Nieman LK, Friedman TC, et al. Inferior petrosal sinus sampling in healthy subjects reveals a unilateral corticotropin-releasing hormone-induced arginine vasopressin release associated with ipsilateral adrenocorticotropin secretion. J Clin Invest 1996; 97:2045–2050.
- Kovacs KJ, Foldes A, Sawchenko PE. Glucocorticoid negative feedback selectively targets vasopressin transcription in parvocellular neurosecretory neurons. J Neurosci 2000; 20:3843–3852.
- Sarkar SB, Sarkar S, Ghosh S, Bandyopadhyay S. Addison's disease. Contemp Clin Dent 2012; 3(4):484–486. doi:10.4103/0976-237X.107450
- Szylman P, Better OS, Chaimowitz C, Rosler A. Role of hyperkalemia in the metabolic acidosis of isolated hypoaldosteronism. N Engl J Med 1976; 294(7):361–365. doi:10.1056/NEJM197602122940703
- Mani S, Rutecki GW. A patient with altered mental status and an acid-base disturbance. Cleve Clin J Med 2017; 84(1):27–34. doi:10.3949/ccjm.84a.16042
- Almiani M, Gorthi J, Subbiah S, Firoz M. Quiz page November 2012: an unusual case of acute hyponatremia and normal anion gap metabolic acidosis. Am J Kidney Dis 2012; 60(5):xxxiii–xxxvi. doi:10.1053/j.ajkd.2012.05.026
- Migeon CJ, Kenny FM, Hung W, Voorhess ML. Study of adrenal function in children with meningitis. Pediatrics 1967; 40(2):163–183.
- Rao RH, Vagnucci AH, Amico JA. Bilateral massive adrenal hemorrhage: early recognition and treatment. Ann Intern Med 1989; 110(3):227–235.
- Shimizu S, Tahara Y, Atsumi T, et al. Waterhouse-Friderichsen syndrome caused by invasive Haemophilus influenzae type B infection in a previously healthy young man. Anaesth Intensive Care 2010; 38(1):214–215.
- Castaldo ET, Guillamondegui OD, Greco JA 3rd, Feurer ID, Miller RS, Morris JA Jr. Are adrenal injuries predictive of adrenal insufficiency in patients sustaining blunt trauma? Am Surg 2008; 74(3):262–266.
- Xarli VP, Steele AA, Davis PJ, Buescher ES, Rios CN, Garcia-Bunuel R. Adrenal hemorrhage in the adult. Medicine (Baltimore) 1978; 57(3):211–221.
- Takeuchi K, Tsuzuki Y, Ando T, et al. Clinical studies of strangulating small bowel obstruction. Am Surg 2004; 70(1):40–44.
- MacKenzie IM. The haemodynamics of human septic shock. Anaesthesia 2001; 56(2):130–144.
- Bornstein SR, Allolio B, Arlt W, et al. Diagnosis and treatment of primary adrenal insufficiency: an Endocrine Society clinical practice guideline. J Clin Endocrinol Metab 2016; 101(2):364–389. doi:10.1210/jc.2015-1710
- Hamrahian AH, Fleseriu M; AACE Adrenal Scientific Committee. Evaluation and management of adrenal insufficiency in critically ill patients: disease state review. Endocr Pract 2017; 23(6):716–725. doi:10.4158/EP161720.RA
- Saenger P, Levine LS, Irvine WJ, et al. Progressive adrenal failure in polyglandular autoimmune disease. J Clin Endocrinol Metab 1982; 54(4):863–867.
- Coco G, Dal Pra C, Presotto F, et al. Estimated risk for developing autoimmune Addison's disease in patients with adrenal cortex autoantibodies. J Clin Endocrinol Metab 2006; 91(5):1637–1645. doi:10.1210/jc.2005-0860
- Ullian ME. The role of corticosteroids in the regulation of vascular tone. Cardiovasc Res 1999; 41(1):55–64.
- Yang S, Zhang L. Glucocorticoids and vascular reactivity. Curr Vasc Pharmacol 2004; 2(1):1–12.
- Yanase T, Tajima T, Katabami T, et al. Diagnosis and treatment of adrenal insufficiency including adrenal crisis: a Japan Endocrine Society clinical practice guideline [Opinion]. Endocr J 2016; 63(9):765–784. doi:10.1507/endocrj.EJ16-0242
- Taylor RL, Grebe SK, Singh RJ. Quantitative, highly sensitive liquid chromatography-tandem mass spectrometry method for detection of synthetic corticosteroids. Clin Chem 2004; 50(10):2345–2352. doi:10.1373/clinchem.2004.033605
- Goldfien A, Laidlaw JC, Haydar NA, Renold AE, Thorn GW. Fluorohydrocortisone and chlorohydrocortisone, highly potent derivatives of compound F. N Engl J Med 1955; 252(11):415–421. doi:10.1056/NEJM195503172521101
- Jenkins D, Forsham PH, Laidlaw JC, Reddy WJ, Thorn GW. Use of ACTH in the diagnosis of adrenal cortical insufficiency. Am J Med 1955; 18(1):3–14.
- Hägg E, Asplund K, Lithner F. Value of basal plasma cortisol assays in the assessment of pituitary-adrenal insufficiency. Clin Endocrinol (Oxf) 1987; 26(2):221–226.
- Deutschbein T, Unger N, Mann K, Petersenn S. Diagnosis of secondary adrenal insufficiency: unstimulated early morning cortisol in saliva and serum in comparison with the insulin tolerance test. Horm Metab Res 2009; 41(4):834–839. doi:10.1055/s-0029-1225630
- Erturk E, Jaffe CA, Barkan AL. Evaluation of the integrity of the hypothalamic-pituitary-adrenal axis by insulin hypoglycemia test. J Clin Endocrinol Metab 1998; 83(7):2350–2354.
- Cooper MS, Stewart PM. Corticosteroid insufficiency in acutely ill patients. N Engl J Med 2003; 348(8):727–734. doi:10.1056/NEJMra020529
- Dellinger RP, Levy MM, Rhodes A, et al; Surviving Sepsis Campaign Guidelines Committee including the Pediatric Subgroup. Surviving Sepsis Campaign: international guidelines for management of severe sepsis and septic shock, 2012. Intensive Care Med 2013; 39(2):165–228. doi:10.1007/s00134-012-2769-8
- Ceccato F, Barbot M, Zilio M, et al. Performance of salivary cortisol in the diagnosis of Cushing's syndrome, adrenal incidentaloma, and adrenal insufficiency. Eur J Endocrinol 2013; 169(1):31–36. doi:10.1530/EJE-13-0159
- Dickstein G, Shechner C, Nicholson WE, et al. Adrenocorticotropin stimulation test: effects of basal cortisol level, time of day, and suggested new sensitive low dose test. J Clin Endocrinol Metab 1991; 72(4):773–778. doi:10.1210/jcem-72-4-773
- May ME, Carey RM. Rapid adrenocorticotropic hormone test in practice. Retrospective review. Am J Med 1985; 79(6):679–884.
- Speckart PF, Nicoloff JT, Bethune JE. Screening for adrenocortical insufficiency with cosyntropin (synthetic ACTH). Arch Intern Med 1971; 128(5):761–763.
- Peechakara S, Bena J, Clarke NJ, et al. Total and free cortisol levels during 1 μg, 25 μg, and 250 μg cosyntropin stimulation tests compared to insulin tolerance test: results of a randomized, prospective, pilot study. Endocrine 2017; 57(3):388–393. doi:10.1007/s12020-017-1371-9
- Finucane FM, Liew A, Thornton E, Rogers B, Tormey W, Agha A. Clinical insights into the safety and utility of the insulin tolerance test (ITT) in the assessment of the hypothalamo-pituitary-adrenal axis. Clin Endocrinol (Oxf) 2008; 69(4):603–607. doi:10.1111/j.1365-2265.2008.03240.x
- Lindholm J, Kehlet H. Re-evaluation of the clinical value of the 30 min ACTH test in assessing the hypothalamic-pituitary-adrenocortical function. Clin Endocrinol (Oxf) 1987; 26(1):53–59.
- Charmandari E, Nicolaides NC, Chrousos GP. Adrenal insufficiency. Lancet 2014; 383(9935):2152–2167. doi:10.1016/S0140-6736(13)61684-0
- Burke CW. Adrenocortical insufficiency. Clin Endocrinol Metab 1985; 14(4):947–976.
- Jessani N, Jehangir W, Behman D, Yousif A, Spiler IJ. Secondary adrenal insufficiency: an overlooked cause of hyponatremia. J Clin Med Res 2015; 7(4):286–288. doi:10.14740/jocmr2041w
- Oelkers W. Hyponatremia and inappropriate secretion of vasopressin (antidiuretic hormone) in patients with hypopituitarism. N Engl J Med 1989; 321(8):492–496. doi:10.1056/NEJM198908243210802
- Ishikawa S, Schrier RW. Effect of arginine vasopressin antagonist on renal water excretion in glucocorticoid and mineralocorticoid deficient rats. Kidney Int 1982; 22(6):587–593.
- Wolfson B, Manning RW, Davis LG, Arentzen R, Baldino F Jr. Co-localization of corticotropin releasing factor and vasopressin mRNA in neurones after adrenalectomy. Nature 1985; 315(6014):59–61.
- Kalogeras KT, Nieman LK, Friedman TC, et al. Inferior petrosal sinus sampling in healthy subjects reveals a unilateral corticotropin-releasing hormone-induced arginine vasopressin release associated with ipsilateral adrenocorticotropin secretion. J Clin Invest 1996; 97:2045–2050.
- Kovacs KJ, Foldes A, Sawchenko PE. Glucocorticoid negative feedback selectively targets vasopressin transcription in parvocellular neurosecretory neurons. J Neurosci 2000; 20:3843–3852.
- Sarkar SB, Sarkar S, Ghosh S, Bandyopadhyay S. Addison's disease. Contemp Clin Dent 2012; 3(4):484–486. doi:10.4103/0976-237X.107450
- Szylman P, Better OS, Chaimowitz C, Rosler A. Role of hyperkalemia in the metabolic acidosis of isolated hypoaldosteronism. N Engl J Med 1976; 294(7):361–365. doi:10.1056/NEJM197602122940703
A 67-year-old woman with bilateral hand numbness
A 67-year-old woman presents to the emergency department after 8 weeks of progressive numbness and tingling in both hands, involving all fingers. The numbness has increased in severity in the last 3 days. She also has occasional numbness around her mouth. She reports no numbness in her feet.
She says she underwent thyroid surgery twice for thyroid cancer 10 years ago. Her medical history also includes type 2 diabetes mellitus (diagnosed 1 year ago), hypertension, dyslipidemia, and diastolic heart failure (diagnosed 5 years ago).
Her current medications are:
- Metformin 1 g twice a day
- Candesartan 16 mg once a day
- Atorvastatin 20 mg once a day
- Furosemide 40 mg twice a day
- Levothyroxine 100 μg per day
- Calcium carbonate 1,500 mg twice a day
- A vitamin D tablet twice a day, which she has not taken for the last 2 months.
She admits she has not been taking her medications regularly because she has been feeling depressed.
On physical examination, she is alert and oriented but appears anxious. She is not in respiratory distress. Her blood pressure is 150/90 mm Hg and her pulse is 92 beats per minute and regular. There is a thyroidectomy scar on the anterior neck. Her jugular venous pressure is not elevated. Her heart sounds are normal without extra sounds. She has no pulmonary rales and no lower-extremity edema.
The Phalen test and Tinel test for carpal tunnel syndrome are negative in both hands. Using a Katz hand diagram, the patient reports tingling and numbness in all fingers, both palms, and the dorsum of both hands. Tapping the area over the facial nerve does not elicit twitching of the facial muscles (ie, no Chvostek sign), but compression of the upper arm elicits carpal spasm (ie, positive Trousseau sign). There is no evidence of motor weakness in her hands. The rest of the physical examination is unremarkable.
POSSIBLE CAUSES OF NUMBNESS
1. Based on the initial evaluation, which of the following is the most likely cause of our patient’s bilateral hand numbness?
- Hypocalcemia due to primary hypoparathyroidism
- Carpal tunnel syndrome due to primary hypothyroidism
- Diabetic peripheral neuropathy
- Vitamin B12 deficiency due to metformin
- Hypocalcemia due to low serum calcitonin
All the conditions above except low serum calcitonin can cause bilateral hand paresthesia. Our patient most likely has hypocalcemia due to primary hypoparathyroidism.
Hypocalcemia
In our patient, bilateral hand numbness and perioral numbness after stopping vitamin D and a positive Trousseau sign strongly suggest hypocalcemia. The classic physical findings in patients with hypocalcemia are the Trousseau sign and the Chvostek sign. The Trousseau sign is elicited by inflating a blood pressure cuff above the systolic blood pressure for 3 minutes and observing for ischemia-induced carpopedal spasm, wrist and metacarpophalangeal joint flexion, thumb adduction, and interphalangeal joint extension. The Chvostek sign is elicited by tapping over the area of the facial nerve below the zygoma in front of the tragus, resulting in ipsilateral twitching of facial muscles.
Although the Trousseau sign is more sensitive and specific than the Chvostek sign, neither is pathognomonic for hypocalcemia.1 The Chvostek sign has been reported to be negative in 30% of patients with hypocalcemia and positive in 10% of normocalcemic individuals.1 The Trousseau sign, however, is present in 94% of hypocalcemic patients vs 1% of normocalcemic individuals.2
Primary hypoparathyroidism secondary to thyroidectomy. Postsurgical hypoparathyroidism is the most common cause of primary hypoparathyroidism. It results from ischemic injury or accidental removal of the parathyroid glands during anterior neck surgery.3,4 The consequent hypocalcemia can be transient, intermittent, or permanent. Permanent postsurgical hypoparathyroidism is defined as persistent hypocalcemia with insufficient parathyroid hormone (PTH) for more than 12 months after neck surgery; however, some consider 6 months to be enough to define the condition.5–7
The incidence of postsurgical hypoparathyroidism varies considerably with the extent of thyroid surgery and the experience of the surgeon.6,8 In the hands of experienced surgeons, permanent hypoparathyroidism occurs in fewer than 1% of patients after total thyroidectomy, whereas the rate may be higher than 6% with less-experienced surgeons.5,9 Other risk factors for postsurgical hypoparathyroidism include female sex, autoimmune thyroid disease, pregnancy, and lactation.5
Pseudohypoparathyroidism is a group of disorders characterized by renal resistance to PTH, leading to hypocalcemia, hyperphosphatemia, and elevated serum PTH. It is also associated with phenotypic features such as short stature and short fourth metacarpal bones.
Calcitonin deficiency. Calcitonin is a polypeptide hormone secreted from the parafollicular (C) cells of the thyroid gland. After total thyroidectomy, calcitonin levels are expected to be reduced. However, the role of calcitonin in humans is unclear. One study has shown that calcitonin is possibly a vestigial hormone, given that no calcitonin-related disorders (excess or deficiency) have been reported in humans.10
Carpal tunnel syndrome due to hypothyroidism
Our patient also could have primary hypothyroidism as a result of thyroidectomy. Hypothyroidism can cause bilateral hand numbness due to carpal tunnel syndrome, which is mediated by mucopolysaccharide deposition and synovial membrane swelling.11 One study reported that 29% of patients with hypothyroidism had carpal tunnel syndrome.12 Symptoms of carpal tunnel syndrome in hypothyroid patients may occur despite thyroid replacement therapy.13
Carpal tunnel syndrome is a clinical diagnosis. Patients usually experience hand paresthesia in the distribution of the median nerve. Provocative physical tests for carpal tunnel syndrome include the Tinel test, the Phalen test, and the Katz hand diagram, which is considered the best of the 3 tests.14,15 Based on how the patient marks the location and type of symptoms on the diagram, carpal tunnel syndrome is rated as classic, probable, possible, or unlikely (Table 1).14,16,17 The sensitivity of a classic or probable diagram ranges from 64% to 80%, while the specificity ranges from 73% to 90%.14,15
Carpal tunnel syndrome is less likely to be the cause of our patient’s symptoms, as her Katz hand diagram indicates only “possible” carpal tunnel syndrome. Her perioral numbness and positive Trousseau sign make hypocalcemia a more likely cause.
Diabetic peripheral neuropathy
Sensory peripheral neuropathy is a recognized complication of diabetes mellitus. However, neuropathy in diabetic patients most commonly manifests initially as distal symmetrical ascending neuropathy starting in the lower extremities.18 Therefore, diabetic peripheral neuropathy is less likely in this patient since her symptoms are limited to her hands.
Vitamin B12 deficiency
Metformin-induced vitamin B12 deficiency is another possible cause of peripheral neuropathy. It might be secondary to metformin-induced changes in intrinsic factor levels and small-intestine motility with resultant bacterial overgrowth, as well as inhibition of vitamin B12 absorption in the terminal ileum.19
However, metformin-induced vitamin B12 deficiency is not the most likely cause of our patient’s neuropathy, since she has been taking this drug for only 1 year. Vitamin B12 deficiency with consequent peripheral neuropathy is more likely in patients taking metformin in high doses for 10 or more years.20
Laboratory results and electrocardiography
Table 2 shows the patient’s initial laboratory results. Of note, her serum calcium level is 5.7 mg/dL (reference range 8.9–10.1). Electrocardiography in the emergency department shows:
- Prolonged PR interval (23 msec)
- Wide QRS complexes (13 msec)
- Flat T waves
- Prolonged corrected QT interval (475 msec)
- Occasional premature ventricular complexes.
CLINICAL MANIFESTATIONS OF HYPOCALCEMIA
2. Which of the following is not a manifestation of hypocalcemia?
- Tonic-clonic seizures
- Cyanosis
- Cardiac ventricular arrhythmias
- Acute pancreatitis
- Depression
Hypocalcemia can cause a wide range of clinical manifestations (Table 3), the extent and severity of which depend on the severity of hypocalcemia and how quickly it develops. The more acute the hypocalcemia, the more severe the manifestations.21
Tetany can cause seizures
Hypocalcemia is characterized by neuromuscular hyperexcitability, manifested clinically by tetany.22 Manifestations of tetany are numerous and include acral paresthesia, perioral numbness, muscle cramps, carpopedal spasm, and seizures. Tetany is the hallmark of hypocalcemia regardless of etiology. However, certain causes are associated with peculiar clinical manifestations. For example, chronic primary hypoparathyroidism may be associated with basal ganglia calcifications that can result in parkinsonism, other extrapyramidal disorders, and dementia (Table 4).6
Airway spasm can be fatal
A serious manifestation of acute severe hypocalcemia is spasm of the glottis muscles, which may cause cyanosis and, if untreated, death.21
Ventricular arrhythmias
Another potential fatal complication of acute severe hypocalcemia is polymorphic ventricular tachycardia due to prolongation of the QT interval, which is readily identified with electrocardiography.23
Hypocalcemia does not cause pancreatitis
Hypercalcemia, rather than hypocalcemia, may cause acute pancreatitis.24 Conversely, acute pancreatitis may cause hypocalcemia due to precipitation of calcium in the abdominal cavity.25
Psychiatric manifestations
In addition to depression, hypocalcemia is associated with psychiatric manifestations including anxiety, confusion, and emotional instability.
STEPS TO DIAGNOSIS OF HYPOCALCEMIA
First step: Confirm true hypocalcemia
Calcium circulates in the blood in 3 forms: bound to albumin (40% to 45%), bound to anions (10% to 15%), and free (ionized) (45%). Although ionized calcium is the active form, most laboratories report total serum calcium.
Since changes in serum albumin concentration affect the total serum calcium level, it is imperative to correct the measured serum calcium to the serum albumin concentration. Each 1-g/dL decrease in serum albumin lowers the total serum calcium by 0.8 mg/dL. Thus:
Corrected serum calcium (mg/dL) =
measured total serum calcium (mg/dL) +
0.8 (4 − serum albumin [g/dL]).
If the patient’s serum calcium level remains low when corrected for serum albumin, he or she has true hypocalcemia, which implies a low ionized serum calcium. Conversely, pseudohypocalcemia means that the measured calcium level is low but the corrected serum calcium is normal.
Using this formula, our patient’s corrected calcium level is calculated as 5.7 + 0.8 (4 – 3.2) = 6.3 mg/dL, indicating true hypocalcemia.
PHOSPHATE IS OFTEN HIGH WHEN CALCIUM IS LOW
In addition to hypocalcemia, our patient has an elevated phosphate level (Table 2).
3. Which of the following hypocalcemic disorders is not associated with hyperphosphatemia?
- End-stage renal disease
- Primary hypoparathyroidism
- Pseudohypoparathyroidism
- Vitamin D3 deficiency
- Rhabdomyolysis
Vitamin D deficiency is not associated with hyperphosphatemia.
Second step in evaluating hypocalcemia: Check phosphate, magnesium, creatinine
The major causes of hypocalcemia can be categorized according to the serum phosphate level: high vs normal or low (Table 5).
High-phosphate, low-calcium states. In the absence of concurrent end-stage renal disease and an excessive phosphate load, primary hypoparathyroidism is the most likely cause of hypocalcemia associated with hyperphosphatemia.
PTH increases serum ionized calcium by26,27:
- Increasing bone resorption
- Increasing reabsorption of calcium from the distal renal tubules
- Increasing the activity of 1-alpha-hydroxylase, responsible for conversion of 25-hydroxyvitamin D3 to 1,25-dihydroxyvitamin D3 (the most biologically active vitamin D metabolite); 1,25-dihydroxyvitamin D increases the absorption of calcium and phosphate from the intestine.
Conversely, PTH decreases reabsorption of phosphate from proximal renal tubules, resulting in hypophosphatemia. Therefore, low serum PTH (primary hypoparathyroidism) or a PTH-resistant state (pseudohypoparathyroidism) results in hypocalcemia and hyperphosphatemia.26,27
Both end-stage renal disease and rhabdomyolysis are associated with high serum phosphate levels. The kidney normally excretes excess dietary phosphate to maintain phosphate homeostasis; however, this is impaired in end-stage renal disease, leading to hyperphosphatemia. In rhabdomyolysis, it is mainly the transcellular shift of phosphate into the extracellular space from myocyte injury that raises phosphate levels.
Normal- or low-phosphate, low calcium states. Hypocalcemia can also result from vitamin D deficiency, but this cause is associated with a low or normal serum phosphate level. In such cases, hypocalcemia causes secondary hyperparathyroidism with consequent renal phosphate loss and, thus, hypophosphatemia.27
Third step: Check serum intact PTH and 25-hydroxyvitamin D levels
Hypocalcemia stimulates secretion of PTH. Therefore, hypocalcemia with elevated serum PTH is caused by disorders that do not impair PTH secretion, including chronic renal failure and vitamin D deficiency (Table 5). Conversely, hypocalcemia with low or normal serum PTH levels suggests primary hypoparathyroidism.
Our patient’s serum PTH level is 20 ng/mL, which is within the reference range. This does not discount the diagnosis of primary hypoparathyroidism. Although most patients with primary hypoparathyroidism have low or undetectable serum PTH levels, some have normal PTH levels if some degree of PTH production is preserved.5,7,28–30 In these patients, the remaining functioning parathyroid tissue is not enough to maintain a normal serum calcium level, resulting in hypocalcemia. As a result, hypocalcemia stimulates the remaining parathyroid tissue to its maximum output, producing PTH levels usually within the lower or middle-normal range.30 In such patients, the terms parathyroid insufficiency and relative primary hypoparathyroidism are more precise than primary hypoparathyroidism.
Postsurgical hypoparathyroidism with an inappropriately normal PTH level is usually seen in patients with disorders that impair intestinal calcium absorption or bone resorption.31 In our patient’s case, the “normal” serum PTH level is likely due to maximal stimulation of remaining functioning parathyroid tissue by severe hypocalcemia, which is a result of her discontinuation of calcium and calcitriol therapy and her vitamin D deficiency.
CASE RESUMED: NO RESPONSE TO INTRAVENOUS CALCIUM GLUCONATE
The patient is given 2 10-mL ampules of 10% calcium gluconate diluted in 100 mL of 5% dextrose in water over 20 minutes intravenously. Electrocardiographic monitoring is continued. Two hours later, her measured serum calcium is only 5.8 mg/dL, with no improvement in her symptoms.
A continuous infusion of calcium gluconate is started: 12 ampules of calcium gluconate are added to 380 mL of 5% dextrose in water and infused at 40 mL/hour (infused rate of elemental calcium = 1.3 mg/kg/hour); 3 hours later, her measured serum calcium level is still only 5.8 mg//dL; at 6 hours it is 5.9 mg/dL, and her symptoms have not improved.
4. Which of the following is the most appropriate next step?
- Change the calcium gluconate to calcium chloride
- Increase the infusion rate to 1.5 mg of elemental calcium/kg/hour
- Give a bolus of 2 10-mL ampules of 10% calcium gluconate intravenously over 1 minute
- Give additional oral calcium tablets
- Check the serum magnesium level
Treatment of hypocalcemia can involve intravenous or oral calcium therapy.
Intravenous calcium is indicated for patients with any of the following6,32:
- Moderate to severe neuromuscular irritability (eg, acral paresthesia, carpopedal spasm, prolonged QT interval, seizures, laryngospasm, bronchospasm)
- Acute hypocalcemia with corrected serum calcium level less than 7.6 mg/dL, even if the patient is asymptomatic
- Cardiac failure.
One 10-mL ampule of 10% calcium gluconate contains 93 mg of elemental calcium; 1 or 2 ampules are typically diluted in 50 to 100 mL of 5% dextrose in water and infused slowly over 15 to 20 minutes. Rapid administration of intravenous calcium is contraindicated, as it may produce cardiac arrhythmias and possibly cardiac arrest. Therefore, intravenous calcium should be given slowly while continuing electrocardiographic monitoring.33
Since the effect of 1 ampule of calcium gluconate lasts only 2 to 3 hours, most patients with symptomatic hypocalcemia require continuous intravenous calcium infusion. The recommended dose of infused elemental calcium is 0.5 to 1.5 mg/kg/hour.34 Several ampules are added to 500 to 1,000 mL of 5% dextrose in water or 0.9% normal saline and infused at a rate appropriate for the patient’s corrected calcium and symptoms.
Oral calcium and vitamin D supplements can be given initially to patients with a corrected serum calcium level of 7.6 mg/dL or greater, with or without mild symptoms, if they can tolerate oral intake. However, this is not the treatment of choice for resistant acute hypocalcemia, as in this case.
Calcium chloride has no advantages over calcium gluconate. Further, it can be associated with local irritation and may result in tissue necrosis if extravasation occurs.35
Increasing the infusion rate of calcium gluconate to the maximum recommended dose may improve the patient’s ionized calcium level and symptoms somewhat. However, it is not the best option for this patient, given that she did not respond to 2 ampules of calcium gluconate followed by continuous infusion of 1.3 mg/kg/hour for 6 hours.
Calcium gluconate bolus. Similarly, giving the patient an additional 2 ampules of calcium gluconate over 1 minute would not be recommended, as rapid administration of intravenous calcium gluconate (eg, over 1 minute) is contraindicated.
Check magnesium
If hypocalcemia persists despite intravenous calcium therapy, as in our patient, further investigation or action is required. An important cause of persistent hypocalcemia is severe hypomagnesemia. Severe hypomagnesemia (serum magnesium < 0.8 mg/dL) causes resistant hypocalcemia by several mechanisms:
- Inducing PTH resistance32,36,37
- Decreasing PTH secretion32,36
- Decreasing calcitriol production.
The decrease in calcitriol production is a direct effect of hypomagnesemia, but it is also an indirect effect of low PTH secretion, which inhibits the enzyme 1-alpha-hydroxylase. Thus, conversion of 25-hydroxyvitamin D3 to calcitriol is impaired, leading to low calcitriol production.
Our patient could have hypomagnesemia due to furosemide use and uncontrolled diabetes mellitus. Hypocalcemia resistant to calcium therapy may occasionally respond to magnesium therapy even if the serum magnesium level is normal. This may be due to depleted intracellular magnesium salt levels.6,38 Rarely, severe hypermagnesemia can also be associated with hypocalcemia due to inhibition of PTH secretion.37,39
CASE RESUMED
Our patient’s serum magnesium level is 0.6 mg/dL (reference range 1.7–2.4 mg/dL). She is given 2 g of magnesium sulfate in 60 mL of 0.9% normal saline infused over 1 hour, followed by a continuous infusion of magnesium sulfate (12 g diluted in 250 mL of 0.9% normal saline, infused over 24 hours). On repeat testing 4 hours later, her serum magnesium level is 0.7 mg/dL, and at 8 hours later it is 0.9 mg/dL. She is subsequently started on oral magnesium oxide 600 mg per day. The magnesium sulfate infusion is continued for another 24 hours.
PREVENTING HYPERCALCIURIA
Patients with low PTH (primary hypoparathyroidism) may have hypercalciuria due to decreased renal tubular calcium reabsorption. Two important measures can minimize hypercalciuria in such patients:
- Keeping the serum calcium level in the low-normal range4,5,40
- Giving a thiazide diuretic (eg, hydrochlorothiazide 12.5–50 mg daily) with a low-salt diet.41,42
A thiazide diuretic is usually started once the 24-hour urine calcium reaches 250 mg.6 Thiazides are thought to enhance both proximal and distal renal tubular calcium reabsorption.43,44
PRIMARY HYPOPARATHYROIDISM: LONG-TERM MANAGEMENT
Long-term management of primary hypoparathyroidism requires calcium and vitamin D supplementation.
Calcium supplements. The most commonly prescribed calcium preparations are calcium carbonate and calcium citrate (containing 40% and 20% elemental calcium, respectively). Calcium carbonate, which is less expensive than calcium citrate, binds with phosphate intake and requires an acidic environment for absorption, and so it is better absorbed when taken with meals. Because calcium citrate does not require an acidic environment for absorption, it is the calcium preparation of choice for patients on proton pump inhibitors, or patients with achlorhydria or constipation.45 Calcium doses vary widely, with most hypoparathyroid patients requiring 1 to 2 g of elemental calcium daily.6
Vitamin D supplements. To promote intestinal absorption, calcium is combined with vitamin D in a fixed-dose preparation given in divided doses.46 Calcitriol (1,25-dihydroxyvitamin D3) is the most active metabolite of vitamin D, with rapid onset and offset of action, and it is the preferred form of vitamin D therapy for patients with hypoparathyroidism. If calcitriol is not available or is not affordable, alphacalcidol (1-alpha-hydroxyvitamin D3) is another option. This is a synthetic analogue of vitamin D that is already hyroxylated at the C1 position. After oral intake, it is hydroxylated in the liver to form calcitriol.
Since renal production of calcitriol is PTH-dependent, in hypoparathyroidism the conversion of 25-hydroxyvitamin D3 to calcitriol is limited. Therefore, vitamin D3 (cholecalciferol) and vitamin D2 (ergocalciferol) are not the preferred forms of vitamin D for such patients. However, either can be added to calcitriol, as they may have extraskeletal benefits.7
CASE CONCLUDED
Our patient presented with primary parathyroid insufficiency associated with vitamin D deficiency. Therefore, in addition to calcitriol and calcium combined with vitamin D in a fixed-dose preparation, her management included vitamin D3 for her vitamin D deficiency.
She was discharged on these medications. At a follow-up visit 3 weeks later, her measured serum calcium level was 8.6 mg/dL. She reported gradual resolution of her symptoms. She was also referred to a psychiatrist for her depression.
TAKE-HOME POINTS
- Hypocalcemia causes neuromuscular excitability, manifested clinically by tetany.
- Common causes of hypocalcemia include vitamin D deficiency, hypomagnesemia, renal failure, and primary hypoparathyroidism.
- The first step in evaluating hypocalcemia is to correct the measured serum calcium to the serum albumin concentration.
- Laboratory testing for hypocalcemia should include serum phosphorus, magnesium, creatinine, PTH, and 25-hydroxyvitamin D3.
- Primary hypoparathyroidism is characterized by hypocalcemia, hyperphosphatemia, and low serum PTH.
- Moderate to severe manifestations of hypo-
calcemia and acute hypocalcemia (< 7.6 mg/dL), even if asymptomatic, warrant intravenous calcium therapy. - Correction of hypomagnesemia is essential to treat hypocalcemia, especially if resistant to intravenous calcium therapy.
- The goal of chronic management of primary hypoparathyroidism includes correcting the serum calcium level to a low-normal range, the serum phosphorus level to an upper-normal range, and prevention of hypercalciuria.
Acknowledgments: The authors wish to thank Mr. Michael Edward Tierney of the School of Medicine, University of Sydney, Australia, for his linguistic editing of the manuscript.
- Jesus JE, Landry A. Images in clinical medicine. Chvostek’s and Trousseau’s signs. N Engl J Med 2012; 367:e15.
- Urbano FL. Signs of hypocalcemia: Chvostek’s and Trousseau’s. Hosp Physician 2000; 36:43–45.
- Chisthi MM, Nair RS, Kuttanchettiyar KG, Yadev I. Mechanisms behind post-thyroidectomy hypocalcemia: interplay of calcitonin, parathormone, and albumin—a prospective study. J Invest Surg 2017; 30:217–225.
- Shoback DM, Bilezikian JP, Costa AG, et al. Presentation of hypoparathyroidism: etiologies and clinical features. J Clin Endocrinol Metab 2016; 101:2300–2312.
- Stack BC Jr, Bimston DN, Bodenner DL, et al. American Association of Clinical Endocrinologists and American College of Endocrinology disease state clinical review: postoperative hypoparathyroidism—definitions and management. Endocr Pract 2015; 21:674–685.
- Shoback D. Clinical practice. Hypoparathyroidism. N Engl J Med 2008; 359:391–403.
- Abate EG, Clarke BL. Review of hypoparathyroidism. Front Endocrinol (Lausanne) 2017; 7:172.
- Coimbra C, Monteiro F, Oliveira P, Ribeiro L, de Almeida MG, Condé A. Hypoparathyroidism following thyroidectomy: predictive factors. Acta Otorrinolaringol Esp 2017; 68:106–111.
- Thomusch O, Machens A, Sekulla C, Ukkat J, Brauckhoff M, Dralle H. The impact of surgical technique on postoperative hypoparathyroidism in bilateral thyroid surgery: a multivariate analysis of 5846 consecutive patients. Surgery 2003; 133:180–185.
- Hirsch PF, Lester GE, Talmage RV. Calcitonin, an enigmatic hormone: does it have a function? J Musculoskelet Neuronal Interact 2001; 1:299–305.
- Karne SS, Bhalerao NS. Carpal tunnel syndrome in hypothyroidism. J Clin Diagn Res 2016; 10:OC36–OC38.
- Duyff RF, Van den Bosch J, Laman DM, van Loon BJ, Linssen WH. Neuromuscular findings in thyroid dysfunction: a prospective clinical and electrodiagnostic study. J Neurol Neurosurg Psychiatry 2000; 68:750–755.
- Palumbo CF, Szabo RM, Olmsted SL. The effects of hypothyroidism and thyroid replacement on the development of carpal tunnel syndrome. J Hand Surg Am 2000; 25:734–739.
- Katz JN, Stirrat CR, Larson MG, Fossel AH, Eaton HM, Liang MH. A self-administered hand symptom diagram for the diagnosis and epidemiologic study of carpal tunnel syndrome. J Rheumatol 1990; 17:1495–1498.
- Katz JN, Stirrat CR. A self-administered hand diagram for the diagnosis of carpal tunnel syndrome. J Hand Surg Am 1990; 15:360–363.
- Calfee RP, Dale AM, Ryan D, Descatha A, Franzblau A, Evanoff B. Performance of simplified scoring systems for hand diagrams in carpal tunnel syndrome screening. J Hand Surg Am 2012; 37:10–17.
- D’Arcy CA, McGee S. The rational clinical examination. Does this patient have carpal tunnel syndrome? JAMA 2000; 283:3110–3117.
- Marchettini P, Lacerenza M, Mauri E, Marangoni C. Painful peripheral neuropathies. Curr Neuropharmacol 2006; 4:175–181.
- Kibirige D, Mwebaze R. Vitamin B12 deficiency among patients with diabetes mellitus: is routine screening and supplementation justified? J Diabetes Metab Disord 2013;12:17.
- Akinlade KS, Agbebaku SO, Rahamon SK, Balogun WO. Vitamin B12 levels in patients with type 2 diabetes mellitus on metformin. Ann Ib Postgrad Med 2015; 13:79–83.
- Tohme JF, Bilezikian JP. Hypocalcemic emergencies. Endocrinol Metab Clin North Am 1993; 22:363–375.
- Macefield G, Burke D. Paraesthesiae and tetany induced by voluntary hyperventilation. Increased excitability of human cutaneous and motor axons. Brain 1991; 114:527–540.
- Benoit SR, Mendelsohn AB, Nourjah P, Staffa JA, Graham DJ. Risk factors for prolonged QTc among US adults: Third National Health and Nutrition Examination Survey. Eur J Cardiovasc Prev Rehabil 2005; 12:363–368.
- Khoo TK, Vege SS, Abu-Lebdeh HS, Ryu E, Nadeem S, Wermers RA. Acute pancreatitis in primary hyperparathyroidism: a population-based study. J Clin Endocrinol Metab 2009; 94:2115–2118.
- McKay C, Beastall GH, Imrie CW, Baxter JN. Circulating intact parathyroid hormone levels in acute pancreatitis. Br J Surg 1994; 81:357–360.
- Talmage RV, Mobley HT. Calcium homeostasis: reassessment of the actions of parathyroid hormone. Gen Comp Endocrinol 2008; 156:1–8.
- Friedman PA, Gesek FA. Calcium transport in renal epithelial cells. Am J Physiol 1993; 264:F181–F198.
- Jensen PV, Jelstrup SM, Homøe P. Long-term outcomes after total thyroidectomy. Dan Med J 2015; 62:A5156.
- Ritter K, Elfenbein D, Schneider DF, Chen H, Sippel RS. Hypoparathyroidism after total thyroidectomy: incidence and resolution. J Surg Res 2015; 197:348–353.
- Promberger R, Ott J, Kober F, Karik M, Freissmuth M, Hermann M. Normal parathyroid hormone levels do not exclude permanent hypoparathyroidism after thyroidectomy. Thyroid 2011; 21:145–150.
- Lorente-Poch L, Sancho JJ, Muñoz-Nova JL, Sánchez-Velázquez P, Sitges-Serra A. Defining the syndromes of parathyroid failure after total thyroidectomy. Gland Surgery 2015; 4:82–90.
- Cooper MS, Gittoes NJ. Diagnosis and management of hypocalcaemia. BMJ 2008; 336:1298–1302.
- Tohme JF, Bilezikian JP. Diagnosis and treatment of hypocalcemic emergencies. Endocrinologist 1996; 6:10–18.
- Carroll R, Matfin G. Endocrine and metabolic emergencies: hypocalcaemia. Ther Adv Endocrinol Metab 2010; 1:29–33.
- Kim MP, Raho VJ, Mak J, Kaynar AM. Skin and soft tissue necrosis from calcium chloride in a deicer. J Emerg Med 2007; 32:41–44.
- Tong GM, Rude RK. Magnesium deficiency in critical illness. J Intensive Care Med 2005; 20:3–17.
- Cholst IN, Steinberg SF, Tropper PJ, Fox HE, Segre GV, Bilezikian JP. The influence of hypermagnesemia on serum calcium and parathyroid hormone levels in human subjects. N Engl J Med 1984; 310:1221–1225.
- Ryzen E, Nelson TA, Rude RK. Low blood mononuclear cell magnesium content and hypocalcemia in normomagnesemic patients. West J Med 1987; 147:549–553.
- Koontz SL, Friedman SA, Schwartz ML. Symptomatic hypocalcemia after tocolytic therapy with magnesium sulfate and nifedipine. Am J Obstet Gynecol 2004; 190:1773–1776.
- Brandi ML, Bilezikian JP, Shoback D, et al. Management of hypoparathyroidism: summary statement and guidelines. J Clin Endocrinol Metab 2016; 101:2273–2283.
- Porter RH, Cox BG, Heaney D, Hostetter TH, Stinebaugh BJ, Suki WN. Treatment of hypoparathyroid patients with chlorthalidone. N Engl J Med 1978; 298:577–581.
- Clarke BL, Brown EM, Collins MT, et al. Epidemiology and diagnosis of hypoparathyroidism. J Clin Endocrinol Metab 2016; 101:2284–2299.
- Nijenhuis T, Vallon V, van der Kemp AW, Loffing J, Hoenderop JG, Bindels RJ. Enhanced passive Ca2+ reabsorption and reduced Mg2+ channel abundance explains thiazide-induced hypocalciuria and hypomagnesemia. J Clin Invest 2005; 115:1651–1658.
- Costanzo LS. Localization of diuretic action in microperfused rat distal tubules: Ca and Na transport. Am J Physiol 1985; 248:F527–F535.
- Brandi ML, Bilezikian JP, Shoback D, et al. Management of hypoparathyroidism: summary statement and guidelines. J Clin Endocrinol Metab 2016; 101:2273–2283.
- Scotti A, Bianchini C, Abbiati G, Marzo A. Absorption of calcium administered alone or in fixed combination with vitamin D to healthy volunteers. Arzneimittelforschung 2001; 51:493–500.
A 67-year-old woman presents to the emergency department after 8 weeks of progressive numbness and tingling in both hands, involving all fingers. The numbness has increased in severity in the last 3 days. She also has occasional numbness around her mouth. She reports no numbness in her feet.
She says she underwent thyroid surgery twice for thyroid cancer 10 years ago. Her medical history also includes type 2 diabetes mellitus (diagnosed 1 year ago), hypertension, dyslipidemia, and diastolic heart failure (diagnosed 5 years ago).
Her current medications are:
- Metformin 1 g twice a day
- Candesartan 16 mg once a day
- Atorvastatin 20 mg once a day
- Furosemide 40 mg twice a day
- Levothyroxine 100 μg per day
- Calcium carbonate 1,500 mg twice a day
- A vitamin D tablet twice a day, which she has not taken for the last 2 months.
She admits she has not been taking her medications regularly because she has been feeling depressed.
On physical examination, she is alert and oriented but appears anxious. She is not in respiratory distress. Her blood pressure is 150/90 mm Hg and her pulse is 92 beats per minute and regular. There is a thyroidectomy scar on the anterior neck. Her jugular venous pressure is not elevated. Her heart sounds are normal without extra sounds. She has no pulmonary rales and no lower-extremity edema.
The Phalen test and Tinel test for carpal tunnel syndrome are negative in both hands. Using a Katz hand diagram, the patient reports tingling and numbness in all fingers, both palms, and the dorsum of both hands. Tapping the area over the facial nerve does not elicit twitching of the facial muscles (ie, no Chvostek sign), but compression of the upper arm elicits carpal spasm (ie, positive Trousseau sign). There is no evidence of motor weakness in her hands. The rest of the physical examination is unremarkable.
POSSIBLE CAUSES OF NUMBNESS
1. Based on the initial evaluation, which of the following is the most likely cause of our patient’s bilateral hand numbness?
- Hypocalcemia due to primary hypoparathyroidism
- Carpal tunnel syndrome due to primary hypothyroidism
- Diabetic peripheral neuropathy
- Vitamin B12 deficiency due to metformin
- Hypocalcemia due to low serum calcitonin
All the conditions above except low serum calcitonin can cause bilateral hand paresthesia. Our patient most likely has hypocalcemia due to primary hypoparathyroidism.
Hypocalcemia
In our patient, bilateral hand numbness and perioral numbness after stopping vitamin D and a positive Trousseau sign strongly suggest hypocalcemia. The classic physical findings in patients with hypocalcemia are the Trousseau sign and the Chvostek sign. The Trousseau sign is elicited by inflating a blood pressure cuff above the systolic blood pressure for 3 minutes and observing for ischemia-induced carpopedal spasm, wrist and metacarpophalangeal joint flexion, thumb adduction, and interphalangeal joint extension. The Chvostek sign is elicited by tapping over the area of the facial nerve below the zygoma in front of the tragus, resulting in ipsilateral twitching of facial muscles.
Although the Trousseau sign is more sensitive and specific than the Chvostek sign, neither is pathognomonic for hypocalcemia.1 The Chvostek sign has been reported to be negative in 30% of patients with hypocalcemia and positive in 10% of normocalcemic individuals.1 The Trousseau sign, however, is present in 94% of hypocalcemic patients vs 1% of normocalcemic individuals.2
Primary hypoparathyroidism secondary to thyroidectomy. Postsurgical hypoparathyroidism is the most common cause of primary hypoparathyroidism. It results from ischemic injury or accidental removal of the parathyroid glands during anterior neck surgery.3,4 The consequent hypocalcemia can be transient, intermittent, or permanent. Permanent postsurgical hypoparathyroidism is defined as persistent hypocalcemia with insufficient parathyroid hormone (PTH) for more than 12 months after neck surgery; however, some consider 6 months to be enough to define the condition.5–7
The incidence of postsurgical hypoparathyroidism varies considerably with the extent of thyroid surgery and the experience of the surgeon.6,8 In the hands of experienced surgeons, permanent hypoparathyroidism occurs in fewer than 1% of patients after total thyroidectomy, whereas the rate may be higher than 6% with less-experienced surgeons.5,9 Other risk factors for postsurgical hypoparathyroidism include female sex, autoimmune thyroid disease, pregnancy, and lactation.5
Pseudohypoparathyroidism is a group of disorders characterized by renal resistance to PTH, leading to hypocalcemia, hyperphosphatemia, and elevated serum PTH. It is also associated with phenotypic features such as short stature and short fourth metacarpal bones.
Calcitonin deficiency. Calcitonin is a polypeptide hormone secreted from the parafollicular (C) cells of the thyroid gland. After total thyroidectomy, calcitonin levels are expected to be reduced. However, the role of calcitonin in humans is unclear. One study has shown that calcitonin is possibly a vestigial hormone, given that no calcitonin-related disorders (excess or deficiency) have been reported in humans.10
Carpal tunnel syndrome due to hypothyroidism
Our patient also could have primary hypothyroidism as a result of thyroidectomy. Hypothyroidism can cause bilateral hand numbness due to carpal tunnel syndrome, which is mediated by mucopolysaccharide deposition and synovial membrane swelling.11 One study reported that 29% of patients with hypothyroidism had carpal tunnel syndrome.12 Symptoms of carpal tunnel syndrome in hypothyroid patients may occur despite thyroid replacement therapy.13
Carpal tunnel syndrome is a clinical diagnosis. Patients usually experience hand paresthesia in the distribution of the median nerve. Provocative physical tests for carpal tunnel syndrome include the Tinel test, the Phalen test, and the Katz hand diagram, which is considered the best of the 3 tests.14,15 Based on how the patient marks the location and type of symptoms on the diagram, carpal tunnel syndrome is rated as classic, probable, possible, or unlikely (Table 1).14,16,17 The sensitivity of a classic or probable diagram ranges from 64% to 80%, while the specificity ranges from 73% to 90%.14,15
Carpal tunnel syndrome is less likely to be the cause of our patient’s symptoms, as her Katz hand diagram indicates only “possible” carpal tunnel syndrome. Her perioral numbness and positive Trousseau sign make hypocalcemia a more likely cause.
Diabetic peripheral neuropathy
Sensory peripheral neuropathy is a recognized complication of diabetes mellitus. However, neuropathy in diabetic patients most commonly manifests initially as distal symmetrical ascending neuropathy starting in the lower extremities.18 Therefore, diabetic peripheral neuropathy is less likely in this patient since her symptoms are limited to her hands.
Vitamin B12 deficiency
Metformin-induced vitamin B12 deficiency is another possible cause of peripheral neuropathy. It might be secondary to metformin-induced changes in intrinsic factor levels and small-intestine motility with resultant bacterial overgrowth, as well as inhibition of vitamin B12 absorption in the terminal ileum.19
However, metformin-induced vitamin B12 deficiency is not the most likely cause of our patient’s neuropathy, since she has been taking this drug for only 1 year. Vitamin B12 deficiency with consequent peripheral neuropathy is more likely in patients taking metformin in high doses for 10 or more years.20
Laboratory results and electrocardiography
Table 2 shows the patient’s initial laboratory results. Of note, her serum calcium level is 5.7 mg/dL (reference range 8.9–10.1). Electrocardiography in the emergency department shows:
- Prolonged PR interval (23 msec)
- Wide QRS complexes (13 msec)
- Flat T waves
- Prolonged corrected QT interval (475 msec)
- Occasional premature ventricular complexes.
CLINICAL MANIFESTATIONS OF HYPOCALCEMIA
2. Which of the following is not a manifestation of hypocalcemia?
- Tonic-clonic seizures
- Cyanosis
- Cardiac ventricular arrhythmias
- Acute pancreatitis
- Depression
Hypocalcemia can cause a wide range of clinical manifestations (Table 3), the extent and severity of which depend on the severity of hypocalcemia and how quickly it develops. The more acute the hypocalcemia, the more severe the manifestations.21
Tetany can cause seizures
Hypocalcemia is characterized by neuromuscular hyperexcitability, manifested clinically by tetany.22 Manifestations of tetany are numerous and include acral paresthesia, perioral numbness, muscle cramps, carpopedal spasm, and seizures. Tetany is the hallmark of hypocalcemia regardless of etiology. However, certain causes are associated with peculiar clinical manifestations. For example, chronic primary hypoparathyroidism may be associated with basal ganglia calcifications that can result in parkinsonism, other extrapyramidal disorders, and dementia (Table 4).6
Airway spasm can be fatal
A serious manifestation of acute severe hypocalcemia is spasm of the glottis muscles, which may cause cyanosis and, if untreated, death.21
Ventricular arrhythmias
Another potential fatal complication of acute severe hypocalcemia is polymorphic ventricular tachycardia due to prolongation of the QT interval, which is readily identified with electrocardiography.23
Hypocalcemia does not cause pancreatitis
Hypercalcemia, rather than hypocalcemia, may cause acute pancreatitis.24 Conversely, acute pancreatitis may cause hypocalcemia due to precipitation of calcium in the abdominal cavity.25
Psychiatric manifestations
In addition to depression, hypocalcemia is associated with psychiatric manifestations including anxiety, confusion, and emotional instability.
STEPS TO DIAGNOSIS OF HYPOCALCEMIA
First step: Confirm true hypocalcemia
Calcium circulates in the blood in 3 forms: bound to albumin (40% to 45%), bound to anions (10% to 15%), and free (ionized) (45%). Although ionized calcium is the active form, most laboratories report total serum calcium.
Since changes in serum albumin concentration affect the total serum calcium level, it is imperative to correct the measured serum calcium to the serum albumin concentration. Each 1-g/dL decrease in serum albumin lowers the total serum calcium by 0.8 mg/dL. Thus:
Corrected serum calcium (mg/dL) =
measured total serum calcium (mg/dL) +
0.8 (4 − serum albumin [g/dL]).
If the patient’s serum calcium level remains low when corrected for serum albumin, he or she has true hypocalcemia, which implies a low ionized serum calcium. Conversely, pseudohypocalcemia means that the measured calcium level is low but the corrected serum calcium is normal.
Using this formula, our patient’s corrected calcium level is calculated as 5.7 + 0.8 (4 – 3.2) = 6.3 mg/dL, indicating true hypocalcemia.
PHOSPHATE IS OFTEN HIGH WHEN CALCIUM IS LOW
In addition to hypocalcemia, our patient has an elevated phosphate level (Table 2).
3. Which of the following hypocalcemic disorders is not associated with hyperphosphatemia?
- End-stage renal disease
- Primary hypoparathyroidism
- Pseudohypoparathyroidism
- Vitamin D3 deficiency
- Rhabdomyolysis
Vitamin D deficiency is not associated with hyperphosphatemia.
Second step in evaluating hypocalcemia: Check phosphate, magnesium, creatinine
The major causes of hypocalcemia can be categorized according to the serum phosphate level: high vs normal or low (Table 5).
High-phosphate, low-calcium states. In the absence of concurrent end-stage renal disease and an excessive phosphate load, primary hypoparathyroidism is the most likely cause of hypocalcemia associated with hyperphosphatemia.
PTH increases serum ionized calcium by26,27:
- Increasing bone resorption
- Increasing reabsorption of calcium from the distal renal tubules
- Increasing the activity of 1-alpha-hydroxylase, responsible for conversion of 25-hydroxyvitamin D3 to 1,25-dihydroxyvitamin D3 (the most biologically active vitamin D metabolite); 1,25-dihydroxyvitamin D increases the absorption of calcium and phosphate from the intestine.
Conversely, PTH decreases reabsorption of phosphate from proximal renal tubules, resulting in hypophosphatemia. Therefore, low serum PTH (primary hypoparathyroidism) or a PTH-resistant state (pseudohypoparathyroidism) results in hypocalcemia and hyperphosphatemia.26,27
Both end-stage renal disease and rhabdomyolysis are associated with high serum phosphate levels. The kidney normally excretes excess dietary phosphate to maintain phosphate homeostasis; however, this is impaired in end-stage renal disease, leading to hyperphosphatemia. In rhabdomyolysis, it is mainly the transcellular shift of phosphate into the extracellular space from myocyte injury that raises phosphate levels.
Normal- or low-phosphate, low calcium states. Hypocalcemia can also result from vitamin D deficiency, but this cause is associated with a low or normal serum phosphate level. In such cases, hypocalcemia causes secondary hyperparathyroidism with consequent renal phosphate loss and, thus, hypophosphatemia.27
Third step: Check serum intact PTH and 25-hydroxyvitamin D levels
Hypocalcemia stimulates secretion of PTH. Therefore, hypocalcemia with elevated serum PTH is caused by disorders that do not impair PTH secretion, including chronic renal failure and vitamin D deficiency (Table 5). Conversely, hypocalcemia with low or normal serum PTH levels suggests primary hypoparathyroidism.
Our patient’s serum PTH level is 20 ng/mL, which is within the reference range. This does not discount the diagnosis of primary hypoparathyroidism. Although most patients with primary hypoparathyroidism have low or undetectable serum PTH levels, some have normal PTH levels if some degree of PTH production is preserved.5,7,28–30 In these patients, the remaining functioning parathyroid tissue is not enough to maintain a normal serum calcium level, resulting in hypocalcemia. As a result, hypocalcemia stimulates the remaining parathyroid tissue to its maximum output, producing PTH levels usually within the lower or middle-normal range.30 In such patients, the terms parathyroid insufficiency and relative primary hypoparathyroidism are more precise than primary hypoparathyroidism.
Postsurgical hypoparathyroidism with an inappropriately normal PTH level is usually seen in patients with disorders that impair intestinal calcium absorption or bone resorption.31 In our patient’s case, the “normal” serum PTH level is likely due to maximal stimulation of remaining functioning parathyroid tissue by severe hypocalcemia, which is a result of her discontinuation of calcium and calcitriol therapy and her vitamin D deficiency.
CASE RESUMED: NO RESPONSE TO INTRAVENOUS CALCIUM GLUCONATE
The patient is given 2 10-mL ampules of 10% calcium gluconate diluted in 100 mL of 5% dextrose in water over 20 minutes intravenously. Electrocardiographic monitoring is continued. Two hours later, her measured serum calcium is only 5.8 mg/dL, with no improvement in her symptoms.
A continuous infusion of calcium gluconate is started: 12 ampules of calcium gluconate are added to 380 mL of 5% dextrose in water and infused at 40 mL/hour (infused rate of elemental calcium = 1.3 mg/kg/hour); 3 hours later, her measured serum calcium level is still only 5.8 mg//dL; at 6 hours it is 5.9 mg/dL, and her symptoms have not improved.
4. Which of the following is the most appropriate next step?
- Change the calcium gluconate to calcium chloride
- Increase the infusion rate to 1.5 mg of elemental calcium/kg/hour
- Give a bolus of 2 10-mL ampules of 10% calcium gluconate intravenously over 1 minute
- Give additional oral calcium tablets
- Check the serum magnesium level
Treatment of hypocalcemia can involve intravenous or oral calcium therapy.
Intravenous calcium is indicated for patients with any of the following6,32:
- Moderate to severe neuromuscular irritability (eg, acral paresthesia, carpopedal spasm, prolonged QT interval, seizures, laryngospasm, bronchospasm)
- Acute hypocalcemia with corrected serum calcium level less than 7.6 mg/dL, even if the patient is asymptomatic
- Cardiac failure.
One 10-mL ampule of 10% calcium gluconate contains 93 mg of elemental calcium; 1 or 2 ampules are typically diluted in 50 to 100 mL of 5% dextrose in water and infused slowly over 15 to 20 minutes. Rapid administration of intravenous calcium is contraindicated, as it may produce cardiac arrhythmias and possibly cardiac arrest. Therefore, intravenous calcium should be given slowly while continuing electrocardiographic monitoring.33
Since the effect of 1 ampule of calcium gluconate lasts only 2 to 3 hours, most patients with symptomatic hypocalcemia require continuous intravenous calcium infusion. The recommended dose of infused elemental calcium is 0.5 to 1.5 mg/kg/hour.34 Several ampules are added to 500 to 1,000 mL of 5% dextrose in water or 0.9% normal saline and infused at a rate appropriate for the patient’s corrected calcium and symptoms.
Oral calcium and vitamin D supplements can be given initially to patients with a corrected serum calcium level of 7.6 mg/dL or greater, with or without mild symptoms, if they can tolerate oral intake. However, this is not the treatment of choice for resistant acute hypocalcemia, as in this case.
Calcium chloride has no advantages over calcium gluconate. Further, it can be associated with local irritation and may result in tissue necrosis if extravasation occurs.35
Increasing the infusion rate of calcium gluconate to the maximum recommended dose may improve the patient’s ionized calcium level and symptoms somewhat. However, it is not the best option for this patient, given that she did not respond to 2 ampules of calcium gluconate followed by continuous infusion of 1.3 mg/kg/hour for 6 hours.
Calcium gluconate bolus. Similarly, giving the patient an additional 2 ampules of calcium gluconate over 1 minute would not be recommended, as rapid administration of intravenous calcium gluconate (eg, over 1 minute) is contraindicated.
Check magnesium
If hypocalcemia persists despite intravenous calcium therapy, as in our patient, further investigation or action is required. An important cause of persistent hypocalcemia is severe hypomagnesemia. Severe hypomagnesemia (serum magnesium < 0.8 mg/dL) causes resistant hypocalcemia by several mechanisms:
- Inducing PTH resistance32,36,37
- Decreasing PTH secretion32,36
- Decreasing calcitriol production.
The decrease in calcitriol production is a direct effect of hypomagnesemia, but it is also an indirect effect of low PTH secretion, which inhibits the enzyme 1-alpha-hydroxylase. Thus, conversion of 25-hydroxyvitamin D3 to calcitriol is impaired, leading to low calcitriol production.
Our patient could have hypomagnesemia due to furosemide use and uncontrolled diabetes mellitus. Hypocalcemia resistant to calcium therapy may occasionally respond to magnesium therapy even if the serum magnesium level is normal. This may be due to depleted intracellular magnesium salt levels.6,38 Rarely, severe hypermagnesemia can also be associated with hypocalcemia due to inhibition of PTH secretion.37,39
CASE RESUMED
Our patient’s serum magnesium level is 0.6 mg/dL (reference range 1.7–2.4 mg/dL). She is given 2 g of magnesium sulfate in 60 mL of 0.9% normal saline infused over 1 hour, followed by a continuous infusion of magnesium sulfate (12 g diluted in 250 mL of 0.9% normal saline, infused over 24 hours). On repeat testing 4 hours later, her serum magnesium level is 0.7 mg/dL, and at 8 hours later it is 0.9 mg/dL. She is subsequently started on oral magnesium oxide 600 mg per day. The magnesium sulfate infusion is continued for another 24 hours.
PREVENTING HYPERCALCIURIA
Patients with low PTH (primary hypoparathyroidism) may have hypercalciuria due to decreased renal tubular calcium reabsorption. Two important measures can minimize hypercalciuria in such patients:
- Keeping the serum calcium level in the low-normal range4,5,40
- Giving a thiazide diuretic (eg, hydrochlorothiazide 12.5–50 mg daily) with a low-salt diet.41,42
A thiazide diuretic is usually started once the 24-hour urine calcium reaches 250 mg.6 Thiazides are thought to enhance both proximal and distal renal tubular calcium reabsorption.43,44
PRIMARY HYPOPARATHYROIDISM: LONG-TERM MANAGEMENT
Long-term management of primary hypoparathyroidism requires calcium and vitamin D supplementation.
Calcium supplements. The most commonly prescribed calcium preparations are calcium carbonate and calcium citrate (containing 40% and 20% elemental calcium, respectively). Calcium carbonate, which is less expensive than calcium citrate, binds with phosphate intake and requires an acidic environment for absorption, and so it is better absorbed when taken with meals. Because calcium citrate does not require an acidic environment for absorption, it is the calcium preparation of choice for patients on proton pump inhibitors, or patients with achlorhydria or constipation.45 Calcium doses vary widely, with most hypoparathyroid patients requiring 1 to 2 g of elemental calcium daily.6
Vitamin D supplements. To promote intestinal absorption, calcium is combined with vitamin D in a fixed-dose preparation given in divided doses.46 Calcitriol (1,25-dihydroxyvitamin D3) is the most active metabolite of vitamin D, with rapid onset and offset of action, and it is the preferred form of vitamin D therapy for patients with hypoparathyroidism. If calcitriol is not available or is not affordable, alphacalcidol (1-alpha-hydroxyvitamin D3) is another option. This is a synthetic analogue of vitamin D that is already hyroxylated at the C1 position. After oral intake, it is hydroxylated in the liver to form calcitriol.
Since renal production of calcitriol is PTH-dependent, in hypoparathyroidism the conversion of 25-hydroxyvitamin D3 to calcitriol is limited. Therefore, vitamin D3 (cholecalciferol) and vitamin D2 (ergocalciferol) are not the preferred forms of vitamin D for such patients. However, either can be added to calcitriol, as they may have extraskeletal benefits.7
CASE CONCLUDED
Our patient presented with primary parathyroid insufficiency associated with vitamin D deficiency. Therefore, in addition to calcitriol and calcium combined with vitamin D in a fixed-dose preparation, her management included vitamin D3 for her vitamin D deficiency.
She was discharged on these medications. At a follow-up visit 3 weeks later, her measured serum calcium level was 8.6 mg/dL. She reported gradual resolution of her symptoms. She was also referred to a psychiatrist for her depression.
TAKE-HOME POINTS
- Hypocalcemia causes neuromuscular excitability, manifested clinically by tetany.
- Common causes of hypocalcemia include vitamin D deficiency, hypomagnesemia, renal failure, and primary hypoparathyroidism.
- The first step in evaluating hypocalcemia is to correct the measured serum calcium to the serum albumin concentration.
- Laboratory testing for hypocalcemia should include serum phosphorus, magnesium, creatinine, PTH, and 25-hydroxyvitamin D3.
- Primary hypoparathyroidism is characterized by hypocalcemia, hyperphosphatemia, and low serum PTH.
- Moderate to severe manifestations of hypo-
calcemia and acute hypocalcemia (< 7.6 mg/dL), even if asymptomatic, warrant intravenous calcium therapy. - Correction of hypomagnesemia is essential to treat hypocalcemia, especially if resistant to intravenous calcium therapy.
- The goal of chronic management of primary hypoparathyroidism includes correcting the serum calcium level to a low-normal range, the serum phosphorus level to an upper-normal range, and prevention of hypercalciuria.
Acknowledgments: The authors wish to thank Mr. Michael Edward Tierney of the School of Medicine, University of Sydney, Australia, for his linguistic editing of the manuscript.
A 67-year-old woman presents to the emergency department after 8 weeks of progressive numbness and tingling in both hands, involving all fingers. The numbness has increased in severity in the last 3 days. She also has occasional numbness around her mouth. She reports no numbness in her feet.
She says she underwent thyroid surgery twice for thyroid cancer 10 years ago. Her medical history also includes type 2 diabetes mellitus (diagnosed 1 year ago), hypertension, dyslipidemia, and diastolic heart failure (diagnosed 5 years ago).
Her current medications are:
- Metformin 1 g twice a day
- Candesartan 16 mg once a day
- Atorvastatin 20 mg once a day
- Furosemide 40 mg twice a day
- Levothyroxine 100 μg per day
- Calcium carbonate 1,500 mg twice a day
- A vitamin D tablet twice a day, which she has not taken for the last 2 months.
She admits she has not been taking her medications regularly because she has been feeling depressed.
On physical examination, she is alert and oriented but appears anxious. She is not in respiratory distress. Her blood pressure is 150/90 mm Hg and her pulse is 92 beats per minute and regular. There is a thyroidectomy scar on the anterior neck. Her jugular venous pressure is not elevated. Her heart sounds are normal without extra sounds. She has no pulmonary rales and no lower-extremity edema.
The Phalen test and Tinel test for carpal tunnel syndrome are negative in both hands. Using a Katz hand diagram, the patient reports tingling and numbness in all fingers, both palms, and the dorsum of both hands. Tapping the area over the facial nerve does not elicit twitching of the facial muscles (ie, no Chvostek sign), but compression of the upper arm elicits carpal spasm (ie, positive Trousseau sign). There is no evidence of motor weakness in her hands. The rest of the physical examination is unremarkable.
POSSIBLE CAUSES OF NUMBNESS
1. Based on the initial evaluation, which of the following is the most likely cause of our patient’s bilateral hand numbness?
- Hypocalcemia due to primary hypoparathyroidism
- Carpal tunnel syndrome due to primary hypothyroidism
- Diabetic peripheral neuropathy
- Vitamin B12 deficiency due to metformin
- Hypocalcemia due to low serum calcitonin
All the conditions above except low serum calcitonin can cause bilateral hand paresthesia. Our patient most likely has hypocalcemia due to primary hypoparathyroidism.
Hypocalcemia
In our patient, bilateral hand numbness and perioral numbness after stopping vitamin D and a positive Trousseau sign strongly suggest hypocalcemia. The classic physical findings in patients with hypocalcemia are the Trousseau sign and the Chvostek sign. The Trousseau sign is elicited by inflating a blood pressure cuff above the systolic blood pressure for 3 minutes and observing for ischemia-induced carpopedal spasm, wrist and metacarpophalangeal joint flexion, thumb adduction, and interphalangeal joint extension. The Chvostek sign is elicited by tapping over the area of the facial nerve below the zygoma in front of the tragus, resulting in ipsilateral twitching of facial muscles.
Although the Trousseau sign is more sensitive and specific than the Chvostek sign, neither is pathognomonic for hypocalcemia.1 The Chvostek sign has been reported to be negative in 30% of patients with hypocalcemia and positive in 10% of normocalcemic individuals.1 The Trousseau sign, however, is present in 94% of hypocalcemic patients vs 1% of normocalcemic individuals.2
Primary hypoparathyroidism secondary to thyroidectomy. Postsurgical hypoparathyroidism is the most common cause of primary hypoparathyroidism. It results from ischemic injury or accidental removal of the parathyroid glands during anterior neck surgery.3,4 The consequent hypocalcemia can be transient, intermittent, or permanent. Permanent postsurgical hypoparathyroidism is defined as persistent hypocalcemia with insufficient parathyroid hormone (PTH) for more than 12 months after neck surgery; however, some consider 6 months to be enough to define the condition.5–7
The incidence of postsurgical hypoparathyroidism varies considerably with the extent of thyroid surgery and the experience of the surgeon.6,8 In the hands of experienced surgeons, permanent hypoparathyroidism occurs in fewer than 1% of patients after total thyroidectomy, whereas the rate may be higher than 6% with less-experienced surgeons.5,9 Other risk factors for postsurgical hypoparathyroidism include female sex, autoimmune thyroid disease, pregnancy, and lactation.5
Pseudohypoparathyroidism is a group of disorders characterized by renal resistance to PTH, leading to hypocalcemia, hyperphosphatemia, and elevated serum PTH. It is also associated with phenotypic features such as short stature and short fourth metacarpal bones.
Calcitonin deficiency. Calcitonin is a polypeptide hormone secreted from the parafollicular (C) cells of the thyroid gland. After total thyroidectomy, calcitonin levels are expected to be reduced. However, the role of calcitonin in humans is unclear. One study has shown that calcitonin is possibly a vestigial hormone, given that no calcitonin-related disorders (excess or deficiency) have been reported in humans.10
Carpal tunnel syndrome due to hypothyroidism
Our patient also could have primary hypothyroidism as a result of thyroidectomy. Hypothyroidism can cause bilateral hand numbness due to carpal tunnel syndrome, which is mediated by mucopolysaccharide deposition and synovial membrane swelling.11 One study reported that 29% of patients with hypothyroidism had carpal tunnel syndrome.12 Symptoms of carpal tunnel syndrome in hypothyroid patients may occur despite thyroid replacement therapy.13
Carpal tunnel syndrome is a clinical diagnosis. Patients usually experience hand paresthesia in the distribution of the median nerve. Provocative physical tests for carpal tunnel syndrome include the Tinel test, the Phalen test, and the Katz hand diagram, which is considered the best of the 3 tests.14,15 Based on how the patient marks the location and type of symptoms on the diagram, carpal tunnel syndrome is rated as classic, probable, possible, or unlikely (Table 1).14,16,17 The sensitivity of a classic or probable diagram ranges from 64% to 80%, while the specificity ranges from 73% to 90%.14,15
Carpal tunnel syndrome is less likely to be the cause of our patient’s symptoms, as her Katz hand diagram indicates only “possible” carpal tunnel syndrome. Her perioral numbness and positive Trousseau sign make hypocalcemia a more likely cause.
Diabetic peripheral neuropathy
Sensory peripheral neuropathy is a recognized complication of diabetes mellitus. However, neuropathy in diabetic patients most commonly manifests initially as distal symmetrical ascending neuropathy starting in the lower extremities.18 Therefore, diabetic peripheral neuropathy is less likely in this patient since her symptoms are limited to her hands.
Vitamin B12 deficiency
Metformin-induced vitamin B12 deficiency is another possible cause of peripheral neuropathy. It might be secondary to metformin-induced changes in intrinsic factor levels and small-intestine motility with resultant bacterial overgrowth, as well as inhibition of vitamin B12 absorption in the terminal ileum.19
However, metformin-induced vitamin B12 deficiency is not the most likely cause of our patient’s neuropathy, since she has been taking this drug for only 1 year. Vitamin B12 deficiency with consequent peripheral neuropathy is more likely in patients taking metformin in high doses for 10 or more years.20
Laboratory results and electrocardiography
Table 2 shows the patient’s initial laboratory results. Of note, her serum calcium level is 5.7 mg/dL (reference range 8.9–10.1). Electrocardiography in the emergency department shows:
- Prolonged PR interval (23 msec)
- Wide QRS complexes (13 msec)
- Flat T waves
- Prolonged corrected QT interval (475 msec)
- Occasional premature ventricular complexes.
CLINICAL MANIFESTATIONS OF HYPOCALCEMIA
2. Which of the following is not a manifestation of hypocalcemia?
- Tonic-clonic seizures
- Cyanosis
- Cardiac ventricular arrhythmias
- Acute pancreatitis
- Depression
Hypocalcemia can cause a wide range of clinical manifestations (Table 3), the extent and severity of which depend on the severity of hypocalcemia and how quickly it develops. The more acute the hypocalcemia, the more severe the manifestations.21
Tetany can cause seizures
Hypocalcemia is characterized by neuromuscular hyperexcitability, manifested clinically by tetany.22 Manifestations of tetany are numerous and include acral paresthesia, perioral numbness, muscle cramps, carpopedal spasm, and seizures. Tetany is the hallmark of hypocalcemia regardless of etiology. However, certain causes are associated with peculiar clinical manifestations. For example, chronic primary hypoparathyroidism may be associated with basal ganglia calcifications that can result in parkinsonism, other extrapyramidal disorders, and dementia (Table 4).6
Airway spasm can be fatal
A serious manifestation of acute severe hypocalcemia is spasm of the glottis muscles, which may cause cyanosis and, if untreated, death.21
Ventricular arrhythmias
Another potential fatal complication of acute severe hypocalcemia is polymorphic ventricular tachycardia due to prolongation of the QT interval, which is readily identified with electrocardiography.23
Hypocalcemia does not cause pancreatitis
Hypercalcemia, rather than hypocalcemia, may cause acute pancreatitis.24 Conversely, acute pancreatitis may cause hypocalcemia due to precipitation of calcium in the abdominal cavity.25
Psychiatric manifestations
In addition to depression, hypocalcemia is associated with psychiatric manifestations including anxiety, confusion, and emotional instability.
STEPS TO DIAGNOSIS OF HYPOCALCEMIA
First step: Confirm true hypocalcemia
Calcium circulates in the blood in 3 forms: bound to albumin (40% to 45%), bound to anions (10% to 15%), and free (ionized) (45%). Although ionized calcium is the active form, most laboratories report total serum calcium.
Since changes in serum albumin concentration affect the total serum calcium level, it is imperative to correct the measured serum calcium to the serum albumin concentration. Each 1-g/dL decrease in serum albumin lowers the total serum calcium by 0.8 mg/dL. Thus:
Corrected serum calcium (mg/dL) =
measured total serum calcium (mg/dL) +
0.8 (4 − serum albumin [g/dL]).
If the patient’s serum calcium level remains low when corrected for serum albumin, he or she has true hypocalcemia, which implies a low ionized serum calcium. Conversely, pseudohypocalcemia means that the measured calcium level is low but the corrected serum calcium is normal.
Using this formula, our patient’s corrected calcium level is calculated as 5.7 + 0.8 (4 – 3.2) = 6.3 mg/dL, indicating true hypocalcemia.
PHOSPHATE IS OFTEN HIGH WHEN CALCIUM IS LOW
In addition to hypocalcemia, our patient has an elevated phosphate level (Table 2).
3. Which of the following hypocalcemic disorders is not associated with hyperphosphatemia?
- End-stage renal disease
- Primary hypoparathyroidism
- Pseudohypoparathyroidism
- Vitamin D3 deficiency
- Rhabdomyolysis
Vitamin D deficiency is not associated with hyperphosphatemia.
Second step in evaluating hypocalcemia: Check phosphate, magnesium, creatinine
The major causes of hypocalcemia can be categorized according to the serum phosphate level: high vs normal or low (Table 5).
High-phosphate, low-calcium states. In the absence of concurrent end-stage renal disease and an excessive phosphate load, primary hypoparathyroidism is the most likely cause of hypocalcemia associated with hyperphosphatemia.
PTH increases serum ionized calcium by26,27:
- Increasing bone resorption
- Increasing reabsorption of calcium from the distal renal tubules
- Increasing the activity of 1-alpha-hydroxylase, responsible for conversion of 25-hydroxyvitamin D3 to 1,25-dihydroxyvitamin D3 (the most biologically active vitamin D metabolite); 1,25-dihydroxyvitamin D increases the absorption of calcium and phosphate from the intestine.
Conversely, PTH decreases reabsorption of phosphate from proximal renal tubules, resulting in hypophosphatemia. Therefore, low serum PTH (primary hypoparathyroidism) or a PTH-resistant state (pseudohypoparathyroidism) results in hypocalcemia and hyperphosphatemia.26,27
Both end-stage renal disease and rhabdomyolysis are associated with high serum phosphate levels. The kidney normally excretes excess dietary phosphate to maintain phosphate homeostasis; however, this is impaired in end-stage renal disease, leading to hyperphosphatemia. In rhabdomyolysis, it is mainly the transcellular shift of phosphate into the extracellular space from myocyte injury that raises phosphate levels.
Normal- or low-phosphate, low calcium states. Hypocalcemia can also result from vitamin D deficiency, but this cause is associated with a low or normal serum phosphate level. In such cases, hypocalcemia causes secondary hyperparathyroidism with consequent renal phosphate loss and, thus, hypophosphatemia.27
Third step: Check serum intact PTH and 25-hydroxyvitamin D levels
Hypocalcemia stimulates secretion of PTH. Therefore, hypocalcemia with elevated serum PTH is caused by disorders that do not impair PTH secretion, including chronic renal failure and vitamin D deficiency (Table 5). Conversely, hypocalcemia with low or normal serum PTH levels suggests primary hypoparathyroidism.
Our patient’s serum PTH level is 20 ng/mL, which is within the reference range. This does not discount the diagnosis of primary hypoparathyroidism. Although most patients with primary hypoparathyroidism have low or undetectable serum PTH levels, some have normal PTH levels if some degree of PTH production is preserved.5,7,28–30 In these patients, the remaining functioning parathyroid tissue is not enough to maintain a normal serum calcium level, resulting in hypocalcemia. As a result, hypocalcemia stimulates the remaining parathyroid tissue to its maximum output, producing PTH levels usually within the lower or middle-normal range.30 In such patients, the terms parathyroid insufficiency and relative primary hypoparathyroidism are more precise than primary hypoparathyroidism.
Postsurgical hypoparathyroidism with an inappropriately normal PTH level is usually seen in patients with disorders that impair intestinal calcium absorption or bone resorption.31 In our patient’s case, the “normal” serum PTH level is likely due to maximal stimulation of remaining functioning parathyroid tissue by severe hypocalcemia, which is a result of her discontinuation of calcium and calcitriol therapy and her vitamin D deficiency.
CASE RESUMED: NO RESPONSE TO INTRAVENOUS CALCIUM GLUCONATE
The patient is given 2 10-mL ampules of 10% calcium gluconate diluted in 100 mL of 5% dextrose in water over 20 minutes intravenously. Electrocardiographic monitoring is continued. Two hours later, her measured serum calcium is only 5.8 mg/dL, with no improvement in her symptoms.
A continuous infusion of calcium gluconate is started: 12 ampules of calcium gluconate are added to 380 mL of 5% dextrose in water and infused at 40 mL/hour (infused rate of elemental calcium = 1.3 mg/kg/hour); 3 hours later, her measured serum calcium level is still only 5.8 mg//dL; at 6 hours it is 5.9 mg/dL, and her symptoms have not improved.
4. Which of the following is the most appropriate next step?
- Change the calcium gluconate to calcium chloride
- Increase the infusion rate to 1.5 mg of elemental calcium/kg/hour
- Give a bolus of 2 10-mL ampules of 10% calcium gluconate intravenously over 1 minute
- Give additional oral calcium tablets
- Check the serum magnesium level
Treatment of hypocalcemia can involve intravenous or oral calcium therapy.
Intravenous calcium is indicated for patients with any of the following6,32:
- Moderate to severe neuromuscular irritability (eg, acral paresthesia, carpopedal spasm, prolonged QT interval, seizures, laryngospasm, bronchospasm)
- Acute hypocalcemia with corrected serum calcium level less than 7.6 mg/dL, even if the patient is asymptomatic
- Cardiac failure.
One 10-mL ampule of 10% calcium gluconate contains 93 mg of elemental calcium; 1 or 2 ampules are typically diluted in 50 to 100 mL of 5% dextrose in water and infused slowly over 15 to 20 minutes. Rapid administration of intravenous calcium is contraindicated, as it may produce cardiac arrhythmias and possibly cardiac arrest. Therefore, intravenous calcium should be given slowly while continuing electrocardiographic monitoring.33
Since the effect of 1 ampule of calcium gluconate lasts only 2 to 3 hours, most patients with symptomatic hypocalcemia require continuous intravenous calcium infusion. The recommended dose of infused elemental calcium is 0.5 to 1.5 mg/kg/hour.34 Several ampules are added to 500 to 1,000 mL of 5% dextrose in water or 0.9% normal saline and infused at a rate appropriate for the patient’s corrected calcium and symptoms.
Oral calcium and vitamin D supplements can be given initially to patients with a corrected serum calcium level of 7.6 mg/dL or greater, with or without mild symptoms, if they can tolerate oral intake. However, this is not the treatment of choice for resistant acute hypocalcemia, as in this case.
Calcium chloride has no advantages over calcium gluconate. Further, it can be associated with local irritation and may result in tissue necrosis if extravasation occurs.35
Increasing the infusion rate of calcium gluconate to the maximum recommended dose may improve the patient’s ionized calcium level and symptoms somewhat. However, it is not the best option for this patient, given that she did not respond to 2 ampules of calcium gluconate followed by continuous infusion of 1.3 mg/kg/hour for 6 hours.
Calcium gluconate bolus. Similarly, giving the patient an additional 2 ampules of calcium gluconate over 1 minute would not be recommended, as rapid administration of intravenous calcium gluconate (eg, over 1 minute) is contraindicated.
Check magnesium
If hypocalcemia persists despite intravenous calcium therapy, as in our patient, further investigation or action is required. An important cause of persistent hypocalcemia is severe hypomagnesemia. Severe hypomagnesemia (serum magnesium < 0.8 mg/dL) causes resistant hypocalcemia by several mechanisms:
- Inducing PTH resistance32,36,37
- Decreasing PTH secretion32,36
- Decreasing calcitriol production.
The decrease in calcitriol production is a direct effect of hypomagnesemia, but it is also an indirect effect of low PTH secretion, which inhibits the enzyme 1-alpha-hydroxylase. Thus, conversion of 25-hydroxyvitamin D3 to calcitriol is impaired, leading to low calcitriol production.
Our patient could have hypomagnesemia due to furosemide use and uncontrolled diabetes mellitus. Hypocalcemia resistant to calcium therapy may occasionally respond to magnesium therapy even if the serum magnesium level is normal. This may be due to depleted intracellular magnesium salt levels.6,38 Rarely, severe hypermagnesemia can also be associated with hypocalcemia due to inhibition of PTH secretion.37,39
CASE RESUMED
Our patient’s serum magnesium level is 0.6 mg/dL (reference range 1.7–2.4 mg/dL). She is given 2 g of magnesium sulfate in 60 mL of 0.9% normal saline infused over 1 hour, followed by a continuous infusion of magnesium sulfate (12 g diluted in 250 mL of 0.9% normal saline, infused over 24 hours). On repeat testing 4 hours later, her serum magnesium level is 0.7 mg/dL, and at 8 hours later it is 0.9 mg/dL. She is subsequently started on oral magnesium oxide 600 mg per day. The magnesium sulfate infusion is continued for another 24 hours.
PREVENTING HYPERCALCIURIA
Patients with low PTH (primary hypoparathyroidism) may have hypercalciuria due to decreased renal tubular calcium reabsorption. Two important measures can minimize hypercalciuria in such patients:
- Keeping the serum calcium level in the low-normal range4,5,40
- Giving a thiazide diuretic (eg, hydrochlorothiazide 12.5–50 mg daily) with a low-salt diet.41,42
A thiazide diuretic is usually started once the 24-hour urine calcium reaches 250 mg.6 Thiazides are thought to enhance both proximal and distal renal tubular calcium reabsorption.43,44
PRIMARY HYPOPARATHYROIDISM: LONG-TERM MANAGEMENT
Long-term management of primary hypoparathyroidism requires calcium and vitamin D supplementation.
Calcium supplements. The most commonly prescribed calcium preparations are calcium carbonate and calcium citrate (containing 40% and 20% elemental calcium, respectively). Calcium carbonate, which is less expensive than calcium citrate, binds with phosphate intake and requires an acidic environment for absorption, and so it is better absorbed when taken with meals. Because calcium citrate does not require an acidic environment for absorption, it is the calcium preparation of choice for patients on proton pump inhibitors, or patients with achlorhydria or constipation.45 Calcium doses vary widely, with most hypoparathyroid patients requiring 1 to 2 g of elemental calcium daily.6
Vitamin D supplements. To promote intestinal absorption, calcium is combined with vitamin D in a fixed-dose preparation given in divided doses.46 Calcitriol (1,25-dihydroxyvitamin D3) is the most active metabolite of vitamin D, with rapid onset and offset of action, and it is the preferred form of vitamin D therapy for patients with hypoparathyroidism. If calcitriol is not available or is not affordable, alphacalcidol (1-alpha-hydroxyvitamin D3) is another option. This is a synthetic analogue of vitamin D that is already hyroxylated at the C1 position. After oral intake, it is hydroxylated in the liver to form calcitriol.
Since renal production of calcitriol is PTH-dependent, in hypoparathyroidism the conversion of 25-hydroxyvitamin D3 to calcitriol is limited. Therefore, vitamin D3 (cholecalciferol) and vitamin D2 (ergocalciferol) are not the preferred forms of vitamin D for such patients. However, either can be added to calcitriol, as they may have extraskeletal benefits.7
CASE CONCLUDED
Our patient presented with primary parathyroid insufficiency associated with vitamin D deficiency. Therefore, in addition to calcitriol and calcium combined with vitamin D in a fixed-dose preparation, her management included vitamin D3 for her vitamin D deficiency.
She was discharged on these medications. At a follow-up visit 3 weeks later, her measured serum calcium level was 8.6 mg/dL. She reported gradual resolution of her symptoms. She was also referred to a psychiatrist for her depression.
TAKE-HOME POINTS
- Hypocalcemia causes neuromuscular excitability, manifested clinically by tetany.
- Common causes of hypocalcemia include vitamin D deficiency, hypomagnesemia, renal failure, and primary hypoparathyroidism.
- The first step in evaluating hypocalcemia is to correct the measured serum calcium to the serum albumin concentration.
- Laboratory testing for hypocalcemia should include serum phosphorus, magnesium, creatinine, PTH, and 25-hydroxyvitamin D3.
- Primary hypoparathyroidism is characterized by hypocalcemia, hyperphosphatemia, and low serum PTH.
- Moderate to severe manifestations of hypo-
calcemia and acute hypocalcemia (< 7.6 mg/dL), even if asymptomatic, warrant intravenous calcium therapy. - Correction of hypomagnesemia is essential to treat hypocalcemia, especially if resistant to intravenous calcium therapy.
- The goal of chronic management of primary hypoparathyroidism includes correcting the serum calcium level to a low-normal range, the serum phosphorus level to an upper-normal range, and prevention of hypercalciuria.
Acknowledgments: The authors wish to thank Mr. Michael Edward Tierney of the School of Medicine, University of Sydney, Australia, for his linguistic editing of the manuscript.
- Jesus JE, Landry A. Images in clinical medicine. Chvostek’s and Trousseau’s signs. N Engl J Med 2012; 367:e15.
- Urbano FL. Signs of hypocalcemia: Chvostek’s and Trousseau’s. Hosp Physician 2000; 36:43–45.
- Chisthi MM, Nair RS, Kuttanchettiyar KG, Yadev I. Mechanisms behind post-thyroidectomy hypocalcemia: interplay of calcitonin, parathormone, and albumin—a prospective study. J Invest Surg 2017; 30:217–225.
- Shoback DM, Bilezikian JP, Costa AG, et al. Presentation of hypoparathyroidism: etiologies and clinical features. J Clin Endocrinol Metab 2016; 101:2300–2312.
- Stack BC Jr, Bimston DN, Bodenner DL, et al. American Association of Clinical Endocrinologists and American College of Endocrinology disease state clinical review: postoperative hypoparathyroidism—definitions and management. Endocr Pract 2015; 21:674–685.
- Shoback D. Clinical practice. Hypoparathyroidism. N Engl J Med 2008; 359:391–403.
- Abate EG, Clarke BL. Review of hypoparathyroidism. Front Endocrinol (Lausanne) 2017; 7:172.
- Coimbra C, Monteiro F, Oliveira P, Ribeiro L, de Almeida MG, Condé A. Hypoparathyroidism following thyroidectomy: predictive factors. Acta Otorrinolaringol Esp 2017; 68:106–111.
- Thomusch O, Machens A, Sekulla C, Ukkat J, Brauckhoff M, Dralle H. The impact of surgical technique on postoperative hypoparathyroidism in bilateral thyroid surgery: a multivariate analysis of 5846 consecutive patients. Surgery 2003; 133:180–185.
- Hirsch PF, Lester GE, Talmage RV. Calcitonin, an enigmatic hormone: does it have a function? J Musculoskelet Neuronal Interact 2001; 1:299–305.
- Karne SS, Bhalerao NS. Carpal tunnel syndrome in hypothyroidism. J Clin Diagn Res 2016; 10:OC36–OC38.
- Duyff RF, Van den Bosch J, Laman DM, van Loon BJ, Linssen WH. Neuromuscular findings in thyroid dysfunction: a prospective clinical and electrodiagnostic study. J Neurol Neurosurg Psychiatry 2000; 68:750–755.
- Palumbo CF, Szabo RM, Olmsted SL. The effects of hypothyroidism and thyroid replacement on the development of carpal tunnel syndrome. J Hand Surg Am 2000; 25:734–739.
- Katz JN, Stirrat CR, Larson MG, Fossel AH, Eaton HM, Liang MH. A self-administered hand symptom diagram for the diagnosis and epidemiologic study of carpal tunnel syndrome. J Rheumatol 1990; 17:1495–1498.
- Katz JN, Stirrat CR. A self-administered hand diagram for the diagnosis of carpal tunnel syndrome. J Hand Surg Am 1990; 15:360–363.
- Calfee RP, Dale AM, Ryan D, Descatha A, Franzblau A, Evanoff B. Performance of simplified scoring systems for hand diagrams in carpal tunnel syndrome screening. J Hand Surg Am 2012; 37:10–17.
- D’Arcy CA, McGee S. The rational clinical examination. Does this patient have carpal tunnel syndrome? JAMA 2000; 283:3110–3117.
- Marchettini P, Lacerenza M, Mauri E, Marangoni C. Painful peripheral neuropathies. Curr Neuropharmacol 2006; 4:175–181.
- Kibirige D, Mwebaze R. Vitamin B12 deficiency among patients with diabetes mellitus: is routine screening and supplementation justified? J Diabetes Metab Disord 2013;12:17.
- Akinlade KS, Agbebaku SO, Rahamon SK, Balogun WO. Vitamin B12 levels in patients with type 2 diabetes mellitus on metformin. Ann Ib Postgrad Med 2015; 13:79–83.
- Tohme JF, Bilezikian JP. Hypocalcemic emergencies. Endocrinol Metab Clin North Am 1993; 22:363–375.
- Macefield G, Burke D. Paraesthesiae and tetany induced by voluntary hyperventilation. Increased excitability of human cutaneous and motor axons. Brain 1991; 114:527–540.
- Benoit SR, Mendelsohn AB, Nourjah P, Staffa JA, Graham DJ. Risk factors for prolonged QTc among US adults: Third National Health and Nutrition Examination Survey. Eur J Cardiovasc Prev Rehabil 2005; 12:363–368.
- Khoo TK, Vege SS, Abu-Lebdeh HS, Ryu E, Nadeem S, Wermers RA. Acute pancreatitis in primary hyperparathyroidism: a population-based study. J Clin Endocrinol Metab 2009; 94:2115–2118.
- McKay C, Beastall GH, Imrie CW, Baxter JN. Circulating intact parathyroid hormone levels in acute pancreatitis. Br J Surg 1994; 81:357–360.
- Talmage RV, Mobley HT. Calcium homeostasis: reassessment of the actions of parathyroid hormone. Gen Comp Endocrinol 2008; 156:1–8.
- Friedman PA, Gesek FA. Calcium transport in renal epithelial cells. Am J Physiol 1993; 264:F181–F198.
- Jensen PV, Jelstrup SM, Homøe P. Long-term outcomes after total thyroidectomy. Dan Med J 2015; 62:A5156.
- Ritter K, Elfenbein D, Schneider DF, Chen H, Sippel RS. Hypoparathyroidism after total thyroidectomy: incidence and resolution. J Surg Res 2015; 197:348–353.
- Promberger R, Ott J, Kober F, Karik M, Freissmuth M, Hermann M. Normal parathyroid hormone levels do not exclude permanent hypoparathyroidism after thyroidectomy. Thyroid 2011; 21:145–150.
- Lorente-Poch L, Sancho JJ, Muñoz-Nova JL, Sánchez-Velázquez P, Sitges-Serra A. Defining the syndromes of parathyroid failure after total thyroidectomy. Gland Surgery 2015; 4:82–90.
- Cooper MS, Gittoes NJ. Diagnosis and management of hypocalcaemia. BMJ 2008; 336:1298–1302.
- Tohme JF, Bilezikian JP. Diagnosis and treatment of hypocalcemic emergencies. Endocrinologist 1996; 6:10–18.
- Carroll R, Matfin G. Endocrine and metabolic emergencies: hypocalcaemia. Ther Adv Endocrinol Metab 2010; 1:29–33.
- Kim MP, Raho VJ, Mak J, Kaynar AM. Skin and soft tissue necrosis from calcium chloride in a deicer. J Emerg Med 2007; 32:41–44.
- Tong GM, Rude RK. Magnesium deficiency in critical illness. J Intensive Care Med 2005; 20:3–17.
- Cholst IN, Steinberg SF, Tropper PJ, Fox HE, Segre GV, Bilezikian JP. The influence of hypermagnesemia on serum calcium and parathyroid hormone levels in human subjects. N Engl J Med 1984; 310:1221–1225.
- Ryzen E, Nelson TA, Rude RK. Low blood mononuclear cell magnesium content and hypocalcemia in normomagnesemic patients. West J Med 1987; 147:549–553.
- Koontz SL, Friedman SA, Schwartz ML. Symptomatic hypocalcemia after tocolytic therapy with magnesium sulfate and nifedipine. Am J Obstet Gynecol 2004; 190:1773–1776.
- Brandi ML, Bilezikian JP, Shoback D, et al. Management of hypoparathyroidism: summary statement and guidelines. J Clin Endocrinol Metab 2016; 101:2273–2283.
- Porter RH, Cox BG, Heaney D, Hostetter TH, Stinebaugh BJ, Suki WN. Treatment of hypoparathyroid patients with chlorthalidone. N Engl J Med 1978; 298:577–581.
- Clarke BL, Brown EM, Collins MT, et al. Epidemiology and diagnosis of hypoparathyroidism. J Clin Endocrinol Metab 2016; 101:2284–2299.
- Nijenhuis T, Vallon V, van der Kemp AW, Loffing J, Hoenderop JG, Bindels RJ. Enhanced passive Ca2+ reabsorption and reduced Mg2+ channel abundance explains thiazide-induced hypocalciuria and hypomagnesemia. J Clin Invest 2005; 115:1651–1658.
- Costanzo LS. Localization of diuretic action in microperfused rat distal tubules: Ca and Na transport. Am J Physiol 1985; 248:F527–F535.
- Brandi ML, Bilezikian JP, Shoback D, et al. Management of hypoparathyroidism: summary statement and guidelines. J Clin Endocrinol Metab 2016; 101:2273–2283.
- Scotti A, Bianchini C, Abbiati G, Marzo A. Absorption of calcium administered alone or in fixed combination with vitamin D to healthy volunteers. Arzneimittelforschung 2001; 51:493–500.
- Jesus JE, Landry A. Images in clinical medicine. Chvostek’s and Trousseau’s signs. N Engl J Med 2012; 367:e15.
- Urbano FL. Signs of hypocalcemia: Chvostek’s and Trousseau’s. Hosp Physician 2000; 36:43–45.
- Chisthi MM, Nair RS, Kuttanchettiyar KG, Yadev I. Mechanisms behind post-thyroidectomy hypocalcemia: interplay of calcitonin, parathormone, and albumin—a prospective study. J Invest Surg 2017; 30:217–225.
- Shoback DM, Bilezikian JP, Costa AG, et al. Presentation of hypoparathyroidism: etiologies and clinical features. J Clin Endocrinol Metab 2016; 101:2300–2312.
- Stack BC Jr, Bimston DN, Bodenner DL, et al. American Association of Clinical Endocrinologists and American College of Endocrinology disease state clinical review: postoperative hypoparathyroidism—definitions and management. Endocr Pract 2015; 21:674–685.
- Shoback D. Clinical practice. Hypoparathyroidism. N Engl J Med 2008; 359:391–403.
- Abate EG, Clarke BL. Review of hypoparathyroidism. Front Endocrinol (Lausanne) 2017; 7:172.
- Coimbra C, Monteiro F, Oliveira P, Ribeiro L, de Almeida MG, Condé A. Hypoparathyroidism following thyroidectomy: predictive factors. Acta Otorrinolaringol Esp 2017; 68:106–111.
- Thomusch O, Machens A, Sekulla C, Ukkat J, Brauckhoff M, Dralle H. The impact of surgical technique on postoperative hypoparathyroidism in bilateral thyroid surgery: a multivariate analysis of 5846 consecutive patients. Surgery 2003; 133:180–185.
- Hirsch PF, Lester GE, Talmage RV. Calcitonin, an enigmatic hormone: does it have a function? J Musculoskelet Neuronal Interact 2001; 1:299–305.
- Karne SS, Bhalerao NS. Carpal tunnel syndrome in hypothyroidism. J Clin Diagn Res 2016; 10:OC36–OC38.
- Duyff RF, Van den Bosch J, Laman DM, van Loon BJ, Linssen WH. Neuromuscular findings in thyroid dysfunction: a prospective clinical and electrodiagnostic study. J Neurol Neurosurg Psychiatry 2000; 68:750–755.
- Palumbo CF, Szabo RM, Olmsted SL. The effects of hypothyroidism and thyroid replacement on the development of carpal tunnel syndrome. J Hand Surg Am 2000; 25:734–739.
- Katz JN, Stirrat CR, Larson MG, Fossel AH, Eaton HM, Liang MH. A self-administered hand symptom diagram for the diagnosis and epidemiologic study of carpal tunnel syndrome. J Rheumatol 1990; 17:1495–1498.
- Katz JN, Stirrat CR. A self-administered hand diagram for the diagnosis of carpal tunnel syndrome. J Hand Surg Am 1990; 15:360–363.
- Calfee RP, Dale AM, Ryan D, Descatha A, Franzblau A, Evanoff B. Performance of simplified scoring systems for hand diagrams in carpal tunnel syndrome screening. J Hand Surg Am 2012; 37:10–17.
- D’Arcy CA, McGee S. The rational clinical examination. Does this patient have carpal tunnel syndrome? JAMA 2000; 283:3110–3117.
- Marchettini P, Lacerenza M, Mauri E, Marangoni C. Painful peripheral neuropathies. Curr Neuropharmacol 2006; 4:175–181.
- Kibirige D, Mwebaze R. Vitamin B12 deficiency among patients with diabetes mellitus: is routine screening and supplementation justified? J Diabetes Metab Disord 2013;12:17.
- Akinlade KS, Agbebaku SO, Rahamon SK, Balogun WO. Vitamin B12 levels in patients with type 2 diabetes mellitus on metformin. Ann Ib Postgrad Med 2015; 13:79–83.
- Tohme JF, Bilezikian JP. Hypocalcemic emergencies. Endocrinol Metab Clin North Am 1993; 22:363–375.
- Macefield G, Burke D. Paraesthesiae and tetany induced by voluntary hyperventilation. Increased excitability of human cutaneous and motor axons. Brain 1991; 114:527–540.
- Benoit SR, Mendelsohn AB, Nourjah P, Staffa JA, Graham DJ. Risk factors for prolonged QTc among US adults: Third National Health and Nutrition Examination Survey. Eur J Cardiovasc Prev Rehabil 2005; 12:363–368.
- Khoo TK, Vege SS, Abu-Lebdeh HS, Ryu E, Nadeem S, Wermers RA. Acute pancreatitis in primary hyperparathyroidism: a population-based study. J Clin Endocrinol Metab 2009; 94:2115–2118.
- McKay C, Beastall GH, Imrie CW, Baxter JN. Circulating intact parathyroid hormone levels in acute pancreatitis. Br J Surg 1994; 81:357–360.
- Talmage RV, Mobley HT. Calcium homeostasis: reassessment of the actions of parathyroid hormone. Gen Comp Endocrinol 2008; 156:1–8.
- Friedman PA, Gesek FA. Calcium transport in renal epithelial cells. Am J Physiol 1993; 264:F181–F198.
- Jensen PV, Jelstrup SM, Homøe P. Long-term outcomes after total thyroidectomy. Dan Med J 2015; 62:A5156.
- Ritter K, Elfenbein D, Schneider DF, Chen H, Sippel RS. Hypoparathyroidism after total thyroidectomy: incidence and resolution. J Surg Res 2015; 197:348–353.
- Promberger R, Ott J, Kober F, Karik M, Freissmuth M, Hermann M. Normal parathyroid hormone levels do not exclude permanent hypoparathyroidism after thyroidectomy. Thyroid 2011; 21:145–150.
- Lorente-Poch L, Sancho JJ, Muñoz-Nova JL, Sánchez-Velázquez P, Sitges-Serra A. Defining the syndromes of parathyroid failure after total thyroidectomy. Gland Surgery 2015; 4:82–90.
- Cooper MS, Gittoes NJ. Diagnosis and management of hypocalcaemia. BMJ 2008; 336:1298–1302.
- Tohme JF, Bilezikian JP. Diagnosis and treatment of hypocalcemic emergencies. Endocrinologist 1996; 6:10–18.
- Carroll R, Matfin G. Endocrine and metabolic emergencies: hypocalcaemia. Ther Adv Endocrinol Metab 2010; 1:29–33.
- Kim MP, Raho VJ, Mak J, Kaynar AM. Skin and soft tissue necrosis from calcium chloride in a deicer. J Emerg Med 2007; 32:41–44.
- Tong GM, Rude RK. Magnesium deficiency in critical illness. J Intensive Care Med 2005; 20:3–17.
- Cholst IN, Steinberg SF, Tropper PJ, Fox HE, Segre GV, Bilezikian JP. The influence of hypermagnesemia on serum calcium and parathyroid hormone levels in human subjects. N Engl J Med 1984; 310:1221–1225.
- Ryzen E, Nelson TA, Rude RK. Low blood mononuclear cell magnesium content and hypocalcemia in normomagnesemic patients. West J Med 1987; 147:549–553.
- Koontz SL, Friedman SA, Schwartz ML. Symptomatic hypocalcemia after tocolytic therapy with magnesium sulfate and nifedipine. Am J Obstet Gynecol 2004; 190:1773–1776.
- Brandi ML, Bilezikian JP, Shoback D, et al. Management of hypoparathyroidism: summary statement and guidelines. J Clin Endocrinol Metab 2016; 101:2273–2283.
- Porter RH, Cox BG, Heaney D, Hostetter TH, Stinebaugh BJ, Suki WN. Treatment of hypoparathyroid patients with chlorthalidone. N Engl J Med 1978; 298:577–581.
- Clarke BL, Brown EM, Collins MT, et al. Epidemiology and diagnosis of hypoparathyroidism. J Clin Endocrinol Metab 2016; 101:2284–2299.
- Nijenhuis T, Vallon V, van der Kemp AW, Loffing J, Hoenderop JG, Bindels RJ. Enhanced passive Ca2+ reabsorption and reduced Mg2+ channel abundance explains thiazide-induced hypocalciuria and hypomagnesemia. J Clin Invest 2005; 115:1651–1658.
- Costanzo LS. Localization of diuretic action in microperfused rat distal tubules: Ca and Na transport. Am J Physiol 1985; 248:F527–F535.
- Brandi ML, Bilezikian JP, Shoback D, et al. Management of hypoparathyroidism: summary statement and guidelines. J Clin Endocrinol Metab 2016; 101:2273–2283.
- Scotti A, Bianchini C, Abbiati G, Marzo A. Absorption of calcium administered alone or in fixed combination with vitamin D to healthy volunteers. Arzneimittelforschung 2001; 51:493–500.